10 класс

Ответы по географии контурная карта 10 класс: ГДЗ контурные карты по географии 10 класс Приваловский Дрофа

Содержание

ГДЗ контурные карты по географии 10 класс Приваловский Дрофа

Многие школьники в качестве предмета на ЕГЭ выбирают географию. Хорошо подготовиться и сдать экзамен на отлично поможет регулярная, ответственная проработка с подходящим учебником. В момент выбор пособия нужно ориентироваться на уровень знаний ребенка, наличие дополнительных помощников и качество подготовки в рамках школьного обучения. Одним из эффективных и полезных навыков по предмету эксперты и педагоги считают работу с картами. С помощью гдз по географии контурные карты за 9 класс Приваловский школьник без труда усвоит всю нужную информацию, и подготовиться к экзамену по географии.

Для кого онлайн справочник служит эффективным помощником в ходе обучения?

Среди активных пользователей подробных ответов по географии 9 класс к контурным картам Приваловского можно выделить:

  • папы и мамы учеников выпускных классов, которые хотят убедиться в том, что у ребенка достаточно знаний, чтобы сдать ЕГЭ хорошо. В силу различных обстоятельств, у родителей не всегда есть время для глубокого погружения в суть школьной дисциплины. В такие моменты решебник поможет не ослабить контроль и быстро отслеживать динамику;
  • школьные предметники, которые преподают географию в десятом классе. В период экзаменов они не успевают параллельно проверять выполненные школьниками упражнения. Воспользовавшись справочником, можно сэкономить время, при этом качественно проверить все задания, не переживая за результат контроля;
  • ребята, которые систематически пропускают уроки, в связи с болезными или разными спортивными мероприятиями и творческими конкурсами. С помощью еуроки ГДЗ ученик быстро догонит программу, и напишет экзамен на высокий бал;
  • дети, которые вынуждены осваивать школьную программу в пределах дома. Подготовка даст возможность найти альтернативное решение некоторым упражнениям, научиться самостоятельно выполнять задания с высоким уровнем сложности;
  • десятиклассники, которые принимают активное участие в олимпиадах и других состязаниях по географии. Пособие станет особенно полезно для школьников, которые не учатся в школах с углубленным изучением предмета, но они хотят повысить уровень знаний и быть не хуже тех школьников, которые учатся в специализированных учебных заведениях.

Какими достоинствами обладает онлайн решебник?

Из большого количества преимуществ готовые решения к контурным картам по географии 9 класс автор Приваловский можно выделить основные:

  • доступность в любое время и любой день;
  • возможность регулярного самоконтроля;
  • стандартизированные ответы;
  • структура в полном соответствии с учебным пособием;
  • экономия материальных средств, за счет того, что нет необходимости нанимать репетиторов.

Старшеклассники в скором времени перейдут на новую ступень обучения. В связи с этим, очень важно уметь работать с практическим и теоретическим материалом, систематически осуществлять самоконтроль, выделять основную информацию, структурировать ее. С помощью еуроки ГДЗ ребенок получит все нужные навыки, хорошо подготовиться к экзаменам и поступить в среднее или высшее заведение своей мечты.

Страница не найдена

Новости


24 авг

Депутат Госдумы, академик РАН Геннадий Онищенко отметил, что в российских школах важно создавать благоприятную среду для профилактики заболеваний.

24 авг

В рамках подготовки к новому учебному году школы на Ямале обследуют специальные комиссии, особое внимание уделяется противопожарной безопасности и антитеррористической защищённости учреждений.

24 авг

Праздничные линейки в школах Оренбурга пройдут 1 сентября в очном формате, заявила начальник управления образования городской администрации Лариса Бебешко.

24 авг

Заслуженный учитель России, доктор педагогических наук Евгений Ямбург назвал логичной и замечательной идею мэра Москвы Сергея Собянина об отмене обязательного ношения масок преподавателями на уроках.

24 авг

Мэр Москвы Сергей Собянин заявил, что всего в столичном регионе привились от коронавирусной инфекции COVID-19 около 80% учителей.

24 авг

Роспотребнадзор в преддверии 1 сентября дал рекомендации по выбору школьного рюкзака.

23 авг

В Тюменской области по итогам 2021 года введут в эксплуатацию восемь школ более чем на 4 тыс. мест, заявил заместитель губернатора, директор департамента образования и науки Алексей Райдер на онлайн-заседании регионального правительства.



Страница не найдена

Новости


24 авг

Депутат Госдумы, академик РАН Геннадий Онищенко отметил, что в российских школах важно создавать благоприятную среду для профилактики заболеваний.

24 авг

В рамках подготовки к новому учебному году школы на Ямале обследуют специальные комиссии, особое внимание уделяется противопожарной безопасности и антитеррористической защищённости учреждений.

24 авг

Праздничные линейки в школах Оренбурга пройдут 1 сентября в очном формате, заявила начальник управления образования городской администрации Лариса Бебешко.

24 авг

Заслуженный учитель России, доктор педагогических наук Евгений Ямбург назвал логичной и замечательной идею мэра Москвы Сергея Собянина об отмене обязательного ношения масок преподавателями на уроках.

24 авг

Мэр Москвы Сергей Собянин заявил, что всего в столичном регионе привились от коронавирусной инфекции COVID-19 около 80% учителей.

24 авг

Роспотребнадзор в преддверии 1 сентября дал рекомендации по выбору школьного рюкзака.

23 авг

В Тюменской области по итогам 2021 года введут в эксплуатацию восемь школ более чем на 4 тыс. мест, заявил заместитель губернатора, директор департамента образования и науки Алексей Райдер на онлайн-заседании регионального правительства.



▶▷▶▷ гдз контурные карты по географии 10 11 класс максаковский

▶▷▶▷ гдз контурные карты по географии 10 11 класс максаковский

ИнтерфейсРусский/Английский
Тип лицензияFree
Кол-во просмотров257
Кол-во загрузок132 раз
Обновление:24-11-2019

гдз контурные карты по географии 10 11 класс максаковский — ГДЗ по географии для 1011 класса Базовый уровень ВП reshebamegdzgeografija 10 -klassmaksakovskij Cached Качественные решения и подробные гдз по географии для учеников 10 11 класса Базовый уровень, авторы учебника:ВП Максаковский ГДЗ по географии 10-11 класс рабочая тетрадь Максаковский gdz-putinainfo 10 -klassgeografiya- 10 gdz Cached ГДЗ ответы на вопросы к рабочей тетради и контурные карты по географии 10 — 11 класс Максаковский ФГОС решебник от Путина 365 (71) Гдз Контурные Карты По Географии 10 11 Класс Максаковский — Image Results More Гдз Контурные Карты По Географии 10 11 Класс Максаковский images ГДЗ решебник по географии 10-11 класс рабочая тетрадь gdzputinaco 10 -klass-onlajngeografiya- 10 gdz Cached ГДЗ решебник по географии 10 — 11 класс рабочая тетрадь Максаковский Здесь представлены ответы к рабочей тетради и контурные карты по географии 10 — 11 класс Максаковский Решебник Контурные карты по географии Дик и Дрофа 10 класс гдз gndakorggdz 10 32 Cached Контурные карты по географии Дик и Дрофа 10 класс Задание не найдено Карта мира ГДЗ решебники по географии 10 класс: ответы к учебникам gdz-georuall-gdz 10 -klassgeografiya- 10 Cached ГДЗ решебники по географии 10 класс : онлайн ответы к учебникам, рабочие тетради, тесты, контрольные и самостоятельные работы, контурные карты ГДЗ рабочая тетрадь по географии 10 класс Максаковский gdz-georuall-gdz 10 -klassgeografiya- 10 Cached Рабочая тетрадь по географии за 10 класс автора Максаковского ВП 2012 года издания Данное пособие по своему наполнению состоит из одного блока и насчитывает 60 страниц ГДЗ решебник география 10 класс Дрофа и Дик контурные карты gdz-putinabizgdz-i-reshebniki-po-geografiigdz Cached ГДЗ решебник география 10 класс Дрофа и Дик контурные карты Предлагаем Вам списать готовые ответы на вопросы к страницам контурных карт по географии за 10 класс Дрофа и Дик 10-11 класс География, учебники онлайн ГДЗ newgdzcomgeografiya-5- 11 -klass-uchebniki-gdz забейте на все выходные садитесь за компьютер и начинайте сегодня! хотя бы повторять, а кто и изучать начинать (пройденный) материал по географии за 11 класс ГДЗ (решебник) по географии 10 класс Максаковский globuss24rugdzrabocaa-tetrad-po-geografii Cached ГДЗ (решебник) по географии 10 класс Максаковский Рабочая тетрадь по географии для 10 класса под редакцией В П Максаковского создавалась специально для проведения практических и ГДЗ решебник по географии 10 класс контурные карты Сиротин gdzputinaco 10 -klass-onlajngeografiya- 10 gdz Cached Контурные карты по географии за 10 класс автора Сиротин ВИ, 2016 года издания Готовые ответы состоят из 56 страниц и содержат географические, экономические характеристики страны, топографические карты со всеми 1 2 3 4 5 Next 30,800

  • 10-11 класс География Ким и Кузнецов Базовый уровень. Если субъективное право связано с субъективным
  • и интересами лица, то субъективная обязанность связана география 10 11 класс максаковский возможностью. Латинская Америка; Тема 11.
    Рабочая тетрадь по географии, 10 класс. ГДЗ по другим предметам gt;
  • ью. Латинская Америка; Тема 11.
    Рабочая тетрадь по географии, 10 класс. ГДЗ по другим предметам gt;gt;. Ответы к рабочим тетрадям, контурным картам и вопросам учебника.
    РАБОЧАЯ ТЕТРАДЬ ПО ГЕОГРАФИИ 10-11 КЛАСС МАКСАКОВСКИЙ ГДЗ. Содержание контурных карт состоит из тех стран мира, которые изучаются учащимися согласно Федеральному стандарту.
    Учебник для 10 кл. 7 класс (с контурными … Скачать бесплатно Контурные карты 6 класс. ГДЗ по английскому языку. ГДЗ по географии (2)
    ГДЗ по географии 10 класс контурные карты Аст-пресс. ГДЗ от Путина ру — онлайн решебники (ГДЗ) к учебникам и рабочим тетрадям.
    Нанесите штриховкой на контурную карту (см. Определите, какие из них относятся к экономически развитым, а какие к развивающимся странам. Контрольная работа по теме динамика 10 класс. Смежные документы, касающиеся геогрaфия мaксaковский 10 клaсс гдз:
    География 10 класс — бесплатные онлайн решебники, ответы и ГДЗ. Контурные карты по географии 10 класс.
    Сайты классов, групп, кружков… Самостоятельная работа по географии 10 класс. Политическая карта мира, 10 класс, тестирование. Выполнить задания в контурной карте….
    География 11 класс практические работы. Авторы: Витченко А.Н., Обух Г.Г., Станкевич Н.Г. Официальные ГДЗ России.
    В соответствии с программой этого курса все задания в тетради подразделяются на 11 тем. Основой для выполнения большинства из них должны послужить школьный учебник и атлас для 10 класса.

10 класс. ГДЗ по другим предметам gt;gt;. Ответы к рабочим тетрадям

10 класс

  • экономические характеристики страны
  • экономические характеристики страны
  • а кто и изучать начинать (пройденный) материал по географии за 11 класс ГДЗ (решебник) по географии 10 класс Максаковский globuss24rugdzrabocaa-tetrad-po-geografii Cached ГДЗ (решебник) по географии 10 класс Максаковский Рабочая тетрадь по географии для 10 класса под редакцией В П Максаковского создавалась специально для проведения практических и ГДЗ решебник по географии 10 класс контурные карты Сиротин gdzputinaco 10 -klass-onlajngeografiya- 10 gdz Cached Контурные карты по географии за 10 класс автора Сиротин ВИ

Нажмите здесь , если переадресация не будет выполнена в течение нескольких секунд гдз контурные карты по географии класс максаковский Поиск в Все Картинки Ещё Видео Новости Покупки Карты Книги Все продукты ГДЗ по географии класс рабочая тетрадь Максаковский https gdz putinainfo klass gdz ГДЗ готовые домашние задания к рабочей тетради и контурные карты по географии класс Максаковский ГДЗ решебник по географии класс рабочая тетрадь https gdz goorg klass Представляем решенную рабочую тетрадь по географии для класса , автор Максаковский ВП Решебник ГДЗ решебник по географии класс Gdz center https gdz center klass gdz CENTER вы найдете ответы к рабочей тетради и контурные карты по географии класс Максаковский Вы ГДЗ по географии класс рабочая тетрадь Максаковский eurokiorg gdz _ klass Рабочая тетрадь с комплектом контурных карт по географии класс ФГОС Сиротин Дрофа Контурные карты ГДЗ решебник по географии класс рабочая тетрадь https gdz popme klass gdz тетради и контурные карты по географии класс Максаковский Вы можете смотреть и читать онлайн без Контурные карты по географии Дик и Дрофа класс gdz Контурные карты по географии Дик и Дрофа класс Контурные карты по географии Дик и Дрофа Задание не География класс Максаковский ГДЗ контурные и worldluxrealtycomgeografiya klass дек класс , решебник ГДЗ , контурные карты География класс , решебник ГДЗ контурные карты Контурные карты класс максаковский ГДЗ решебник по miryvhildingandersotzyvyrukonturni сен Здесь представлены ответы к рабочей тетради и контурные карты по географии класс Рабочая тетрадь по географии , класс Максаковский geo gdz ru rabochayatetradpo Ответы к рабочей тетради по географии класса Политическая карта мира География мировых природных ресурсов География Латинская Америка Глобальные проблемы человечества ГДЗ по географии Решебник ГДЗ по географии за класс gdz _ klass Подробный решебник гдз по Географии за класс к учебнику школьной География класс контурные карты География класс Базовый уровень автор ВП Максаковский Рабочая тетрадь Максаковский география классы kartgeoburorurabochayatetrad Содержит вопросы, графики, контурные карты , тесты Читать рабочую тетрадь по географии для классов года Решебник ГДЗ рабочая тетрадь география классы Максаковский онлайн Читать учебник география класс гдз карта adlerhudozhkarugeografiia klass окт география класс гдз карта ГДЗ География класс Румянцев Контурная карта Гдз по географии контурные карты класс максаковский levelapru gdz pogeografiikonturnie Я думаю, что он простит нас когда узнает, что мы преданы и пытались по география контурные карты Гдз по географии класс максаковский крассворд по pinterestcom Гдз по географии класс максаковский крассворд по японии Гдз погеометрии класс погорелов Решебник контурные карты начальный курс по географии класс авторыип галай Сделанные Контурные Карты По Географии Класс ГДЗ контурных карт географии класс содержат краткие сведения про основные страны Европы, Вам так же будут интересны География класс Рабочая тетрадь Максаковский Гдз контурные карты по географии класс максаковский ziehaethoowerxaamnltjly gdz konturny Волновые свойства микрочастиц Старцев Я, контроль, самооценка Сушку проводят на воздухе после гдз по контурным картам класс максаковский октября gojovgirruphp окт Гдз решебник по географии класс контурные карты рабочая тетрадь максаковский анализ Гдз решебник по географии класс рабочая тетрадь максаковский гдз география максаковский контурная карта tbmmovaby gdz geografiia сен ГДЗ решебник по географии класс контурные карты Сиротин Автор Максаковский ВП География класс рабочая тетрадь Максаковский Самые подробные ГДЗ и гдз рт по географии класс ten t окт гдз рт по географии класс ГДЗ по географии класс Максаковский Гдз по географии за класс, год ГДЗ решебник по географии класс контурные карты решебник по рабочей тетради класс география сиротин dushkzrureshebnikporabochei ноя ГДЗ рабочая тетрадь по географии для классов ГДЗ по географии класс контурные карты Сиротин по географии класса максаковского Суббота Картинки география рабочая тетрадь гдз Mosvagru mosvagrugeografiia rabochaia окт тетради и контурные карты по географии класс Максаковский ГДЗ по географии для География классы География Книги для школы labirintrugenres Контурные карты к УМК В География классы Контурные карты к УМК В П Максаковского ФГОС гдз контурная карта класс природные ресурсы xwhppsa emfucomerleklablogcom a гдз контурная карта класс природные ресурсы Картинки по запросу гдз контурная карта класс автора тетради и контурные карты по географии класс Максаковский ГДЗ карта гдз по географии класс arkaimavtoru gdz pogeografii kla сен гдз по географии класс класс рабочая тетрадь Максаковский gdz putinainfo тетради и контурные карты по географии класс Максаковский ФГОС География класс максаковский контурные карты гдз shiyaiku ojagaeyiekaarhusworkscomgegva География класс максаковский контурные карты гдз Кузовлев Activity Book английский язык класс Это подметил СМСоловьев, хотя он и не ГДЗ класс География Рабочая тетрадь Максаковский klass ГДЗ Рабочая тетрадь по географии Максаковский класс ОГЛАВЛЕНИЕ Тема Политическая карта мира Тема География отраслей мирового хозяйства ГДЗ и klass gdz География отраслей мирового хозяйства Рабочая тетрадь класс Максаковский Ответы и ГДЗ Задание гдз рабочая тетрадь максаковский класс aquamaniaru gdz окт задания к рабочей тетради и контурные карты по географии класс Максаковский ФГОС Книга География классы Контурные карты К Ozon ozonrucontext Рейтинг отзыва Книга География классы Контурные карты К учебнику В П Максаковского характеристики, фото и решебник онлайн по географии класс максаковский musicprobarriecomreshebnikonlain ноя тетради и контурные карты по географии класс Максаковский Гдз по географии класс гдз география класс задания DomNouta gdz geografiia сен гдз география класс задания ГДЗ География класс Максаковский Рабочая задания к рабочей тетради и контурные карты по географии класс Максаковский ФГОС гдз по географии класс рабочая тетрадь дрофа nokemkurganschoolruphp окт Рабочая тетрадь с контурными картами Контурные карты по географии за класс автора сиротин в тетрадь по географии для класса , автор максаковский в гдз по географии рабочий тетрадь класс максаковский roskinoorg gdz pogeografiirabochii Максаковский ГДЗ готовые домашние задания к рабочей тетради и контурные карты по географии класс география класс блок добывания знаний и умений тема bypaasjruphp окт Гдз по географии класс максаковский блок добывания знаний и умений как гдз по контурной ГДЗ География класс Niglus gdz ГДЗ География класс Рабочая тетрадь по географии Максаковский класс Контурные карты по географии контурные карты гдз alttransrukonturnyekarty gdz xm окт Контурные карты по географии за класс ответы сверяйся Org Безкоштовні Гдз з по клас Онлайн Без ГДЗ по географии класс рабочая тетрадь Максаковский гдз рабочая тетрадь по географии максаковский asppermrucon ten t gdz rabochaiatetrad ноя рабочей тетради и контурные карты по географии класс Максаковский ГДЗ по географии География класс максаковский рабочая тетрадь гдз keyru?jdx klass tetrad gdz Гдз готовые домашние задания к рабочей тетради и контурные карты по географии класс максаковский География Рабочая тетрадь классы Максаковский ВП Максаковский ВП, Заяц ДВ , с Рабочая тетрадь по курсу географии классы предназначена для тесты, анализ графиков, составление таблиц, работу с контурными картами, Политическая карта мира ГДЗ рабочая тетрадь по географии класс Максаковский https gdz georu gdz klass Рабочая тетрадь по географии за класс автора Максаковского ВП года издания Данное пособие по контурная карта по географии класс решебник Аккуратная сделанная контурная карта по географии за класс с ответами Тема Окна в окружающий мир Тема Многогранный мир чувств географии класс Максаковский Решебник ГДЗ рабочая тетрадь по гдз по географии класс тетрадь максаковский University wwwugmclubcom gdz pogeografii сен и контурные карты по географии класс Максаковский ФГОС решебник от Путина ГДЗ гдз география тетрадь класс максаковский denprokhorovru gdz geografiiatetrad сен задания к рабочей тетради и контурные карты по географии класс Максаковский ФГОС гдз онлайн класс география ALTC altcru gdz onlain klass geografiia сен Гдз решебник контурные карты класс издат; ельство дик и дрофа год ответы на вопросы к рабочей тетради и контурные карты по географии класс Максаковский гдз по географии максаковский dolgoprudnyconditionerru gdz po сен гдз по географии максаковский и контурные карты по географии класс Максаковский ГДЗ гдз контурные карты класс ресурсы La Mensola wwwvalledelmensolait gdz konturny сен гдз контурные карты класс ресурсы Контурные карты по географии класс Максаковский ГДЗ iphone Готовая контурные карты география класс украина Контурные карты по географии класс ФГОС gdz netsubject?sk класс ГДЗ География класс Рабочая тетрадь по географии класс Максаковский Просвещение гдз география класс максаковский cstransitru gdz geografiia klass сен ГДЗ География класс Максаковский Рабочая Аноним on гдз по географии класс максаковский к учебнику и контурные карты по географии класс Максаковский Картинки по запросу гдз контурные карты по географии класс максаковский гдз по географии класса максаковский wwwglasskoreaorg сен и контурные карты по географии класс Максаковский ФГОС решебник от Путина ГДЗ по Запросы, похожие на гдз контурные карты по географии класс максаковский гдз по географии учебник класс максаковский учебник гдз по географии класс максаковский ответы на задания рабочая тетрадь по географии класс максаковский онлайн гдз по географии класс максаковский блок самостоятельных решений гдз по географии класс максаковский учебник гдз по географии класс рабочая тетрадь кузнецов гдз по географии класс контурные карты география класс бахчиева гдз След Войти Версия Поиска Мобильная Полная Конфиденциальность Условия Настройки Отзыв Справка

10-11 класс География Ким и Кузнецов Базовый уровень. Если субъективное право связано с субъективными интересами лица, то субъективная обязанность связана география 10 11 класс максаковский возможностью. Латинская Америка; Тема 11.
Рабочая тетрадь по географии, 10 класс. ГДЗ по другим предметам gt;gt;. Ответы к рабочим тетрадям, контурным картам и вопросам учебника.
РАБОЧАЯ ТЕТРАДЬ ПО ГЕОГРАФИИ 10-11 КЛАСС МАКСАКОВСКИЙ ГДЗ. Содержание контурных карт состоит из тех стран мира, которые изучаются учащимися согласно Федеральному стандарту.
Учебник для 10 кл. 7 класс (с контурными … Скачать бесплатно Контурные карты 6 класс. ГДЗ по английскому языку. ГДЗ по географии (2)
ГДЗ по географии 10 класс контурные карты Аст-пресс. ГДЗ от Путина ру — онлайн решебники (ГДЗ) к учебникам и рабочим тетрадям.
Нанесите штриховкой на контурную карту (см. Определите, какие из них относятся к экономически развитым, а какие к развивающимся странам. Контрольная работа по теме динамика 10 класс. Смежные документы, касающиеся геогрaфия мaксaковский 10 клaсс гдз:
География 10 класс — бесплатные онлайн решебники, ответы и ГДЗ. Контурные карты по географии 10 класс.
Сайты классов, групп, кружков… Самостоятельная работа по географии 10 класс. Политическая карта мира, 10 класс, тестирование. Выполнить задания в контурной карте….
География 11 класс практические работы. Авторы: Витченко А.Н., Обух Г.Г., Станкевич Н.Г. Официальные ГДЗ России.
В соответствии с программой этого курса все задания в тетради подразделяются на 11 тем. Основой для выполнения большинства из них должны послужить школьный учебник и атлас для 10 класса.

▶▷▶ контурный карты по географии 10 класс дрофа гдз

▶▷▶ контурный карты по географии 10 класс дрофа гдз

ИнтерфейсРусский/Английский
Тип лицензияFree
Кол-во просмотров257
Кол-во загрузок132 раз
Обновление:06-01-2019

контурный карты по географии 10 класс дрофа гдз — Yahoo Search Results Yahoo Web Search Sign in Mail Go to Mail» data-nosubject=»[No Subject]» data-timestamp=’short’ Help Account Info Yahoo Home Settings Home News Mail Finance Tumblr Weather Sports Messenger Settings Want more to discover? Make Yahoo Your Home Page See breaking news more every time you open your browser Add it now No Thanks Yahoo Search query Web Images Video News Local Answers Shopping Recipes Sports Finance Dictionary More Anytime Past day Past week Past month Anytime Get beautiful photos on every new browser window Download ГДЗ по географии 9 класс контурные карты Дрофа Дик gdz-putinainfo › 9 класс › География ГДЗ по географии 9 класс контурные карты Дрофа Дик ГДЗ готовые домашние задания к контурным картам по географии 9 класс Дрофа и Дик ФГОС от Путина Готовые контурные карты по географии 10 класс — Решеба reshebanet//48- 1-0 -140 Cached Решебники → ГДЗ → География → Готовые контурные карты по географии 10 класс Готовые контурные карты по географии 10 класс ГДЗ География 5 класс Румянцев (контурные карты) gramoteycom/gdz-geografiya-5-klass-rumyantsev Cached Контурные карты География 5 класс Румянцев АВ Дрофа На этой странице школьного портала Грамотей размещены ответы на контурные карты Румянцева за 5 класс по предмету география ГДЗ по Географии 6 класс Контурные карты ответы gdz-na-5com › География › 6 класс ГДЗ по Географии 6 класс Контурные карты ответы Дик» и » Дрофа » контурные карты по ГДЗ контурные карты по географии 8 класс Котляр botanamnet › … › 8 класс › География Подробный решебник ГДЗ к контурным картам по географии 8 класс Котляр ОГ 2016, онлайн ответы на домашнюю работу Атласы и контурные карты – Скачать электронные книги бесплатно wwwchitalkinoru/atlasy Cached Вашему вниманию предлагается атлас по географии для 5 класса географии 6 класс С ГДЗ от Путина 10 класс География gdz-putinainfo/ 10 -klass/geografiya- 10 Cached ГДЗ по географии 10 -11 класс Домогацких рабочая тетрадь ГДЗ по географии 10 -11 класс рабочая тетрадь Максаковский ГДЗ Контурные карты по географии 9 класс Дик и Дрофа Решебник gitemco/gdz/9/8/4 Cached ГДЗ Контурные карты по географии 9 класс Дик и Дрофа Решебник карты по географии Дик и Решебник (ГДЗ) к контурной карте по географии 6 класс Дрофа megareshebaru//6_klass_drofa/39- 1-0 -2111 Cached / контурные карты Дрофа карте по географии за 6 класс с ГДЗ , он сможет подготовиться к Решебник Контурные карты по географии Дик и Дрофа 7 класс гдз sluverorg/gdz/7/18 Cached Для рационального использования свободного времени, столь важного для самообразования, создан решебник контурные карты по географии Дик и Дрофа 7 класс гдз , который поможет выявить Promotional Results For You Free Download | Mozilla Firefox ® Web Browser wwwmozillaorg Download Firefox — the faster, smarter, easier way to browse the web and all of Yahoo 1 2 3 4 5 Next 1,200 results Settings Help Suggestions Privacy (Updated) Terms (Updated) Advertise About ads About this page Powered by Bing™

  • ДиК» по географии за 10 класс Решены и готовы следующие контурные карты : В разработке Эти ответы предназначены родителям
  • ученик проверяет
  • горы

Рига

чтобы получать только отличные оценки Почему полезно использование контурных карт онлайн Применяя при подготовке пособия с контурными картами на нашем сайте

  • он сможет подготовиться к Решебник Контурные карты по географии Дик и Дрофа 7 класс гдз sluverorg/gdz/7/18 Cached Для рационального использования свободного времени
  • столь важного для самообразования
  • easier way to browse the web and all of Yahoo 1 2 3 4 5 Next 1

контурный карты по географии 10 класс дрофа гдз — Поиск в Google Специальные ссылки Перейти к основному контенту Справка по использованию специальных возможностей Оставить отзыв о специальных возможностях Нажмите здесь , если переадресация не будет выполнена в течение нескольких секунд Войти Удалить Пожаловаться на неприемлемые подсказки Режимы поиска Все Картинки Новости Покупки Видео Ещё Карты Книги Авиабилеты Финансы Настройки Настройки поиска Языки (Languages) Включить Безопасный поиск Расширенный поиск Ваши данные в Поиске История Поиск в справке Инструменты Результатов: примерно 25 (0,41 сек) Looking for results in English? Change to English Оставить русский Изменить язык Результаты поиска Все результаты ГДЗ готовые контурные карты по географии 10 класс — gdz-georu › Решебники › 10 класс › География Сохраненная копия Решебник ( ГДЗ ) ответы к готовым контурным картам по географии 10 класс Приваловский ДиК и Дрофа Решебник Контурные карты по географии Дик и Дрофа 10 класс гдз gndakorg/gdz/10/32 Сохраненная копия Отправь задание и получи ответ Получить решение Контурные карты по географии Дик и Дрофа 10 класс Контурные карты по географии Дик и Видео 0:41 гдз контурные карты 10 класс география дрофа Вера Федорова YouTube — 20 июл 2017 г 2:59 Видеоинструкция по работе с контурными картами Светлана Лаврентьева YouTube — 21 окт 2013 г Картинки по запросу контурный карты по географии 10 класс дрофа гдз «dhl»:1,»id»:»FMnT0IcYZ28mbM:»,»ml»:»600″:»bh»:90,»bw»:122,»oh»:3060,»ou»:» «,»ow»:4814,»pt»:»gdz-georu/10/images/cm_b_1_1_bjpg»,»rh»:»gdz-georu»,»rid»:»sistM7AArsKA2M»,»rt»:0,»ru»:» «,»sc»:1,»st»:»gdz-georu»,»th»:90,»tu»:» \u003dtbn:ANd9GcSRmGEA9jXSh2RUQEm91HMKvVzzQvx8QFFoEVXtOzzWaWuWVN5uZNVpdBI»,»tw»:142 «copy»:»gdz-georu\u2030 \u201d њK»,»id»:»GhWImGZM7EMZgM:»,»ml»:»600″:»bh»:90,»bw»:66,»oh»:1331,»ou»:» «,»ow»:968,»pt»:»gdz-georu/10/images/cm_b_2_1_bjpg»,»rh»:»gdz-georu»,»rid»:»sistM7AArsKA2M»,»rt»:0,»ru»:» «,»sc»:1,»st»:»gdz-georu»,»th»:98,»tu»:» \u003dtbn:ANd9GcQw_WYF7Z43ZOk_nhQGy0jZ4BTs_ukJ5CqIln7ZHyL781ZKJrM3Vo7G3g»,»tw»:71 «copy»:»gdz-georu»,»id»:»TjbR0GNpYu2fYM:»,»ml»:»600″:»bh»:90,»bw»:122,»oh»:2386,»ou»:» «,»ow»:3278,»pt»:»gdz-georu/10/images/cm_b_7_1_bjpg»,»rh»:»gdz-georu»,»rid»:»sistM7AArsKA2M»,»rt»:0,»ru»:» «,»sc»:1,»st»:»gdz-georu»,»th»:90,»tu»:» \u003dtbn:ANd9GcQ_h__iM87p1H9zqcZZIYOZp_ZmrX9D5aufB_s51WhebkWlrLEks17Vng»,»tw»:124 «id»:»2IZomsLQM_BAPM:»,»ml»:»600″:»bh»:90,»bw»:66,»oh»:1331,»ou»:» «,»ow»:968,»pt»:»gdz-georu/10/images/cm_b_13_1_bjpg»,»rh»:»gdz-georu»,»rid»:»sistM7AArsKA2M»,»rt»:0,»ru»:» «,»sc»:1,»st»:»gdz-georu»,»th»:98,»tu»:» \u003dtbn:ANd9GcQnxIkpbD-_2g78GPpN18EwQrku5eR6AJZ_38mUyEjrk9DgTC2aOuQX8Q»,»tw»:71 «id»:»L1j0bO2VkCCngM:»,»ml»:»600″:»bh»:90,»bw»:66,»oh»:1331,»ou»:» «,»ow»:968,»pt»:»gdz-georu/10/images/cm_b_12_1_bjpg»,»rh»:»gdz-georu»,»rid»:»sistM7AArsKA2M»,»rt»:0,»ru»:» «,»sc»:1,»st»:»gdz-georu»,»th»:98,»tu»:» \u003dtbn:ANd9GcQ3vdiAFZ_6dZSonjggKrgg8pdaEYqbNsx3LLAYiRf2QoesQ9TyfJI0Usk»,»tw»:71 «copy»:»gdz-georuАpАмБg°зєЈ\u2030*P»,»id»:»mgbrxGyU9m_qOM:»,»ml»:»600″:»bh»:90,»bw»:66,»oh»:1331,»ou»:» «,»ow»:968,»pt»:»gdz-georu/10/images/cm_b_4_1_bjpg»,»rh»:»gdz-georu»,»rid»:»sistM7AArsKA2M»,»rt»:0,»ru»:» «,»sc»:1,»st»:»gdz-georu»,»th»:98,»tu»:» \u003dtbn:ANd9GcTDZNoEOfkqcZaZa3VlsyCtLeIuf7U0Gy4wX4flnLEKSIRM_cG7pSpg-u0″,»tw»:71 «id»:»bJln5l1gEcxG0M:»,»ml»:»600″:»bh»:90,»bw»:68,»oh»:1331,»ou»:» «,»ow»:968,»pt»:»gdz-georu/10/images/cm_b_11_1_bjpg»,»rh»:»gdz-georu»,»rid»:»sistM7AArsKA2M»,»rt»:0,»ru»:» «,»st»:»gdz-georu»,»th»:98,»tu»:» \u003dtbn:ANd9GcSCY93mVymBisVvTXrKlTb41KHmdNsJEc3pTp6Ftxdj4R8B1QTUn773BQ»,»tw»:71 Другие картинки по запросу «контурный карты по географии 10 класс дрофа гдз» Жалоба отправлена Пожаловаться на картинки Благодарим за замечания Пожаловаться на другую картинку Пожаловаться на содержание картинки Отмена Пожаловаться Все результаты ГДЗ решебник по географии 10 класс контурные карты Дрофа, Дик › 10 класс › География Сохраненная копия Предлагаемый решебник задаваемых заданий по географии для 10 класса на контурных картах Дрофа , Дик, дает ГДЗ для учащихся За счет него ГДЗ решебник по географии 10 класс контурные карты Дрофа Дик › 10 класс › География Сохраненная копия На сайте GDZ CENTER вы найдете ответы к контурным картам по географии 10 класс Дрофа и Дик Вы можете смотреть и читать онлайн (без ГДЗ от Путина по географии 10 класс контурные карты Дрофа, Дик › 10 класс › География Сохраненная копия Ответы к контурным картам по географии для 10 класса Дрофа , Дик от Путина – это первоклассный сборник уже готовых, прорисованных карт Стр 2 контурные 10 класс Дрофа — Контурные карты konturmapru/10klas-gdz-history-drofa-p2html Сохраненная копия Готовые домашние задания по истории России 10 класс — контурная карта » Российская империя к 1914 году» атлас Дрофа 2015 год, страницы 2,3 Контурные карты по географии 10 класс Дрофа ДиК ФГОС | ISBN › › Контурные карты по географии 10 класс Сохраненная копия Купите контурные карты по географии для 10 класса издательства Дрофа ДиК по выгодной цене в интернет-магазине издательства На сайте гдз контурная карта по географии 5 класс дрофа ответы румянцев profcareerru//gdz-konturnaia-karta-po-geografii-5-klass-drofa-otvety-rumiantsevx Сохраненная копия гдз контурная карта по географии 5 класс дрофа ответы румянцев и Дрофа 10 класс Контурные карты по истории для 7 классаАтлас контурных карт география 10 класс контурные карты гдз — Houston Body Wrap hgcbodywrapcom/?geografiya_10_klass_konturnie_karti_gdzoption Сохраненная копия география 10 класс контурные карты гдз карты география 8 класс, здесь представлены ответы к контурным картам по географии 8 класс дрофа и дик ГДЗ | решебник по географии | ВКонтакте Сохраненная копия Похожие решебник по географии запись закреплена 6 сен 2014 Действия Пожаловаться ГДЗ решебник контурные карты по географии Дик и Дрофа 10 класс ▷ гдз по географии контурные карты 5 класс издательство дрофа wwwzstelceu/content//gdz-po-geografii-konturnye-karty-5-klass-izdatelstvo-drofa Сохраненная копия 4 нояб 2018 г — гдз по географии контурные карты 5 класс издательство дрофа и условные знаки ГДЗ география 6 класс контурные карты Дрофа ответы 2017 Дрофа География 5 — 6 класс Скрыть География 10 -11 классы ▷ гдз по географии контурные карты 10 класс учись быть первым wwwzstelceu/content//gdz-po-geografii-konturnye-karty-10-klass-uchis-byt-pervy Сохраненная копия 4 нояб 2018 г — гдз по географии контурные карты 10 класс учись быть первым ГДЗ решебник к контурным картам по географии 7 класс Дрофа ГДЗ по географии 8 класс Приваловский (контурные карты Учись Сохраненная копия ГДЗ на контурные карты по географии за 8 класс Учись быть первым Skip to content ☰ Главная · 1 класс · 2 класс · 3 класс · 4 класс · 5 класс · 6 класс · 7 класс · 8 класс · 9 класс · 10 класс · 11 класс Дрофа , Дик На этой странице вы найдете решебник к контурным картам по предмету география 8 класс ГДЗ к контурной карте по географии Беларуси 10 класс › › География › контурные карты Белкартография Сохраненная копия Похожие Подробные ответы и гдз к контурным картам по географии Беларуси для учащихся 10 класс , издательство Белкартография на 2016 учебный год ▷ гдз по географии контурная карта 5 класс дрофа 2016 wwwsrccalumniorg//gdz-po-geografii-konturnaia-karta-5-klass-drofa-2016xml Сохраненная копия 27 нояб 2018 г — гдз по географии контурная карта 5 класс дрофа 2016 материал ГДЗ решебник по географии 10 класс контурные карты Дрофа , Дик Контурные карты по географии 10 класс дрофа скачать wwwconnectxhealthwarecom/?Konturnie_karti_po10_klass_drofa Контурные карты по географии 10 класс дрофа скачать Ответы к рабочей тетради по географии Сиротин Экономическая и социальная география Контурная карта по географии 6 класс дрофа гдз » Популярные Контурная карта по географии 6 класс дрофа гдз Это фосфатное, что с целью Гдз по сканеру алгебра 10 класс кругловатый глава Из этично Атласы и контурные карты по географии География Книги для › › Книги для школы › География Сохраненная копия Дмитрий Заяц — География 10 -11 классы Контурные карты Базовый уровень обложка География 10 -11 классы Контурные карты Базовый уровень 1 Заполненная контурная карта 11 класса katiga7mpl/posts/20-zapolnennaja-konturnaja-karta-11-klassahtml Сохраненная копия 4 окт 2017 г — Тут отличные гдз по географии контурные карты для 5 класса от Путина Готовая контурные карты по географии 10 класс дрофа гдз на контурной карте отметить крайние точки, моря, границы › 5 — 9 классы › География Сохраненная копия Похожие 10 сент 2016 г — 5 — 9 классы · География ; 5 баллов На контурной карте отметить крайние точки, моря, границы, ( Карта российской Федерации) ГДЗ контурные карты по географии 5 класс Румянцев Дрофа › География › 5 класс Сохраненная копия Похожие Решебник по географии за 5 класс авторы Румянцев издательство Дрофа ГДЗ готовые контурные карты по географии Дик и Дрофа 8 класс Похожие ГДЗ готовые контурные карты по географии Дик и Дрофа 8 класс Гдз по русскому языку 10 -11 класс гольцова упр Алгебра, Рыбные Игры, Windows ГДЗ решебник география 10 класс Дрофа и Дик — контурные gdz-putinabiz/gdz/gdz-reshebnik-geografiya-10-klass-drofa-i-dik-konturnye-karty Сохраненная копия Предлагаем Вам списать готовые ответы на вопросы к страницам контурных карт по географии за 10 класс Дрофа и Дик Номера заданий к ГДЗ удобно ГДЗ и решебник по географии (контурные карты Дрофа Дик) для gdz-otvety-reshebnikicom//407_gdz_i_reshebnik_po_geografii_konturnye_karty_ Сохраненная копия Спиши готовую домашнюю работу ( ГДЗ ) из онлайн решебника по географии ( контурные карты Дрофа Дик) для 10 класса Смотри бесплатные ответы Пояснения к фильтрации результатов Мы скрыли некоторые результаты, которые очень похожи на уже представленные выше (33) Показать скрытые результаты Вместе с контурный карты по географии 10 класс дрофа гдз часто ищут гдз контурные карты по географии 10-11 класс дрофа гдз контурные карты 10-11 класс дрофа контурные карты по географии 10 класс дрофа онлайн гдз по географии 10 класс контурные карты аст пресс контурная карта по географии 10-11 класс изд картография гдз гдз контурные карты по географии 10-11 класс дрофа 2018 гдз контурные карты по география 10 класс экономическая и социальная гдз по географии 10 класс контурные карты дрофа 2018 Ссылки в нижнем колонтитуле Россия — Подробнее… Справка Отправить отзыв Конфиденциальность Условия Аккаунт Поиск Карты YouTube Play Новости Почта Контакты Диск Календарь Google+ Переводчик Фото Ещё Покупки Документы Blogger Hangouts Google Keep Jamboard Подборки Другие сервисы Google

Яндекс Яндекс Найти Поиск Поиск Картинки Видео Карты Маркет Новости ТВ онлайн Знатоки Коллекции Музыка Переводчик Диск Почта Все Ещё Дополнительная информация о запросе Добавлены результаты по запросу « контурны е карты по географии 10 класс дрофа гдз » Искать только « контурный карты по географии 10 класс дрофа гдз » Показаны результаты для Нижнего Новгорода Москва 1 ГДЗ решебник по географии 10 класс контурные карты gdzgoorg › 10…574-geografiya…konturye-karty-drofa… Сохранённая копия Показать ещё с сайта Пожаловаться Информация о сайте Предлагаемый решебник задаваемых заданий по географии для 10 класса на контурных картах Дрофа , Дик, дает ГДЗ для учащихся За счет него, ученик проверяет, правильно ли он выполнил задание 2 ГДЗ Контурные карты по географии 10 класс Дик Карта мира Зарубежная Азия Япония Индия Великобритания nigluscom › gdz/10/8/3 Сохранённая копия Показать ещё с сайта Пожаловаться Информация о сайте Решебник Контурные карты по географии Дик и Дрофа 10 класс гдз Читать ещё Решебник Контурные карты по географии Дик и Дрофа 10 класс гдз niglussp@gmailcom Скрыть 3 ГДЗ решебник география 10 класс Дрофа и Дик gdz-putinabiz › …geografii…10…drofa…konturnye-karty Сохранённая копия Показать ещё с сайта Пожаловаться Информация о сайте Предлагаем Вам списать готовые ответы на вопросы к страницам контурных карт по географии за 10 класс Дрофа и Дик Номера заданий к ГДЗ удобно читать и смотреть онлайн с телефонов (скачать нельзя) Читать ещё Предлагаем Вам списать готовые ответы на вопросы к страницам контурных карт по географии за 10 класс Дрофа и Дик Номера заданий к ГДЗ удобно читать и смотреть онлайн с телефонов (скачать нельзя) Все решебники к книге (к учебнику и рабочей тетради) постоянно обновляются и соответствуют домашней работе 2013, 2014, 2015 года Контурные карты с решениями — ФГОС Выберите номер страницы с упражнениями к параграфам и главам Скрыть 4 Гдз вип Гдз по Географии 10 класс Контурные карты gdz-vipru › …10-класс/география/контурные…дрофа… Сохранённая копия Показать ещё с сайта Пожаловаться Информация о сайте В 10 классе много самостоятельных заданий по географии нужно выполнить на контурных картах Наиболее удачные из них – контурные карты Дик Дрофа Использование именно этих контурных карт также предлагают учителя географии в школе Ведь они сами пользуются ими, при подготовке к урокам Читать ещё В 10 классе много самостоятельных заданий по географии нужно выполнить на контурных картах Наиболее удачные из них – контурные карты Дик Дрофа Использование именно этих контурных карт также предлагают учителя географии в школе Ведь они сами пользуются ими, при подготовке к урокам географии Скрыть 5 ГДЗ решебник по географии 10 класс контурные карты GdzPutinaco › 10-klass…geografii…konturnie…drofa… Сохранённая копия Показать ещё с сайта Пожаловаться Информация о сайте Здесь представлены ответы к контурным картам по географии 10 класс Дрофа и Дик Вы можете смотреть и читать онлайн (без скачивания) с компьютера и мобильных устройств Читать ещё Здесь представлены ответы к контурным картам по географии 10 класс Дрофа и Дик Вы можете смотреть и читать онлайн (без скачивания) с компьютера и мобильных устройств Быстрый поиск НАЙТИ Решебник на обновлении Другие решебники ГДЗ решебник по географии 10 класс рабочая тетрадь Домогацких Алексеевский ГДЗ решебник по географии 10 класс контурные карты Сиротин Скрыть 6 Контурные карты география 10 класс KartGeoBuroru › konturnye-karty/klass_10/ Сохранённая копия Показать ещё с сайта Пожаловаться Информация о сайте Все контурные карты и атласы по географии для 10 класса Высокое качество контурных карт по географии позволит ученикам хорошо подготовиться к школьным занятиям, чтобы получать только отличные оценки Читать ещё Все контурные карты и атласы по географии для 10 класса Высокое качество контурных карт по географии позволит ученикам хорошо подготовиться к школьным занятиям, чтобы получать только отличные оценки Почему полезно использование контурных карт онлайн Применяя при подготовке пособия с контурными картами на нашем сайте, ученик приобретает умение хорошо ориентироваться в пространстве, сможет легко находить реки, горы, континенты и государства Систематическая работа с контурными картами имеет очень важное значение в формировании прочных географических знаний у 10-классников 24 январь Скрыть 7 ГДЗ по географии 10 класс контурные карты eurokiorg › gdz…10_klass…geografii…konturnye-karty… Сохранённая копия Показать ещё с сайта Пожаловаться Информация о сайте Решебник по географии за 10 класс авторы Приваловский издательство Дрофа ГДЗ контурные карты по географии 10 класс Приваловский Дрофа Читать ещё Решебник по географии за 10 класс авторы Приваловский издательство Дрофа ГДЗ контурные карты по географии 10 класс Приваловский Дрофа стр 2 стр 3 стр 4 стр 5 стр 6 стр 7 стр 8 стр 9 стр 10 стр 11 стр 12 стр 13 стр 14 стр 15 стр 16 Новыe © Copyright Все права защищены Правообладатель SIA Ksenokss Адрес: 1073, Курземес проспект 106/45, Рига, Латвия Скрыть 8 Контурные карты по географии 10 класс Дрофа ДиК rosuchebnikru › …konturnye-karty…geografii…drofa… Сохранённая копия Показать ещё с сайта Пожаловаться Информация о сайте Задания к контурным картам переработаны с учетом обновленного содержания атласа Систематическое использование контурных карт на уроке и при выполнении домашних заданий способствует более прочному усвоению учебного материала, формирует как целостное Читать ещё Задания к контурным картам переработаны с учетом обновленного содержания атласа Систематическое использование контурных карт на уроке и при выполнении домашних заданий способствует более прочному усвоению учебного материала, формирует как целостное восприятие конкретной территории, так и умение выделять её специфические особенности Издание прошло рецензирование Комиссии Русского географического общества по географическому и экологическому образованию Содержание контурных карт разработано в соответствии с требованиями ФГОС Контурные карты являются универсальным продуктом и могут использоват Скрыть 9 Решебник, ГДЗ по географии 10 класс Дрофа , Дик vpr-klasscom › gdz…drofa…10kl…konturnye_kartyhtml Сохранённая копия Показать ещё с сайта Пожаловаться Информация о сайте Здесь можно списать готовые ответы и смотреть онлайн решебник по географии за 10 класс к контурным картам Дрофы и Дика Предлагаем списать готовые ответы на вопросы к параграфам и контурным картам 10 класса Дрофы и Дика Домашнюю работу с решенными заданиями к страницам и разделам Читать ещё Здесь можно списать готовые ответы и смотреть онлайн решебник по географии за 10 класс к контурным картам Дрофы и Дика Предлагаем списать готовые ответы на вопросы к параграфам и контурным картам 10 класса Дрофы и Дика Домашнюю работу с решенными заданиями к страницам и разделам легко смотреть онлайн даже на уроке Вернуться на главную страницу сайта vpr-klasscom Решебник и ГДЗ по географии 10 класс — Дрофа , Дик — контурные карты (к учебнику и рабочей тетради) — 2012, 2013, 2014 год (ФГОС) Чтобы смотреть готовые ответы на упражнения, выберите нужный раздел — снизу появится решение Плохо видно? Скрыть 10 Контурные карты по географии 10 класс Дрофа ГДЗ mnogo-reshebnikovcom › 10 класс › География › …-10-klass-drofa-gdz… Сохранённая копия Показать ещё с сайта Пожаловаться Информация о сайте На данной страничке можно просмотреть ГДЗ онлайн к контурным картам « Дрофа , ДиК» по географии за 10 класс Решены и готовы следующие контурные карты : В разработке Эти ответы предназначены родителям, которые смогут сверить правильность заполнения контурных карт , выявить недочеты в Читать ещё На данной страничке можно просмотреть ГДЗ онлайн к контурным картам « Дрофа , ДиК» по географии за 10 класс Решены и готовы следующие контурные карты : В разработке Эти ответы предназначены родителям, которые смогут сверить правильность заполнения контурных карт , выявить недочеты в знаниях, устраняя их, тем самым, оказать помощь ученику 10 класса в качественной подготовке к уроку географии Скрыть География 10 -11 классы Контурные карты / chitai-gorodru Доставка Акции Книги Канцтовары chitai-gorodru › География-10-11-клас Не подходит по запросу Спам или мошенничество Мешает видеть результаты Информация о сайте реклама Более 230 000 книг классической литературы, учебники для школьников и студентов Контактная информация +7 (495) 444-84-44 пн-вс 9:00-21:00 Магазин на Маркете 18+ Вместе с « контурный карты по географии 10 класс дрофа гдз » ищут: контурный карты по географии 7 класс контурный карты по географии 8 класс контурный карты по географии 9 класс гдз контурный карты по географии 6 класс контурный карты по географии 9 класс контурный карты контурный карты по истории 6 класс контурный карты по географии 10 класс контурный карты по истории 5 класс контурный карты по истории 7 класс 1 2 3 4 5 дальше Bing Google Mailru Нашлось 223 млн результатов Дать объявление Показать все Регистрация Войти 0+ Браузер с Алисой, которая на многое отвечает сразу Установить Закрыть Спасибо, что помогаете делать Яндекс лучше! Эта реклама отправилась на дополнительную проверку ОК ЯндексДирект Попробовать ещё раз Москва Настройки Клавиатура Помощь Обратная связь Для бизнеса Директ Метрика Касса Телефония Для души Музыка Погода ТВ онлайн Коллекции Яндекс О компании Вакансии Блог Контакты Мобильный поиск © 1997–2019 ООО «Яндекс» Лицензия на поиск Статистика Поиск защищён технологией Protect Алиса в ЯндексБраузере Выключит компьютер по голосовой команде 0+ Скачать Включить

ГДЗ География 10 класс Приваловский

Тонкости географии

Многие ученики 10 класса принимаю решение сдавать итоговый экзамен по дисциплине география. Для многих она наиболее легкая. Те, кто располагают необходимым объемом знания сдача не является трудностью. Но есть и другая категория учеников, которым необходима дополнительная подготовка. Помощником в этом деле станет «ГДЗ по географии 10 класс Контурные карты Приваловский (Дрофа)».

По мнению самих учеников самыми сложными темами являются:

  • география нематериальной сферы Зарубежной Европы;
  • экономико-географическое районирование США;
  • географические аспекты демографической и продовольственной проблем мира.

Знать эти темы важно. Все потому что они могут быть представлены в экзаменационных билетах.

Используя дополнительное пособие школьник сможет:

  1. заблаговременно подготовиться к написанию пробного экзамена и контрольному срезу;
  2. понять, по какому принципу разработаны разнообразные карты мира;
  3. понять смысл в самых сложных темах.

Полученные знания помогут ребенку стать увереннее в себе, что крайне важно в школе.

Отличительные черты ГДЗ

В 10 классе возраст ребенка осознанный. Преподаватели это понимают и не жалеют их предоставляя большой объем информации для изучения. В нем содержится много тем, которые могут вызвать трудности. В дополнительном пособии отражены все тонкости предстоящего курса.

Любой сможет найти в нем:

  • ответы как на основные, так и дополнительные вопросы от учителя;
  • подробный разбор всех встречаемых контурных карт и их правильное оформление;
  • правильные ответы на множество упражнений.

Пользоваться решебником легко. Все потому что его структура полностью схожа с главным изданием. Ребенок сможет не просто дублировать правильные ответы в рабочую тетрадь, но и понимать, что для него самое сложное. На этом основании можно сделать вывод – сдавать экзамен по дисциплине или сделать выбор в пользу другой.

Веские преимущества решебника

Дополнительное пособие служит отличным решением тем, кто не имеет возможности воспользоваться услугами репетитора. Стоит отметить, тот факт, что использование дополнительного пособия помогает наверстать упущенное. Порой ребенок пропускает уроки. Теперь изучение пропущенных тем не вызывает сложностей. Напрашивается один лишь вывод – «ГДЗ по географии 10 класс Контурные карты Приваловский А.Н. (Дрофа)» верный друг для будущего выпускника.

▶▷▶▷ гдз контурная карта по географии 10 класс максаковский учебник

▶▷▶▷ гдз контурная карта по географии 10 класс максаковский учебник

ИнтерфейсРусский/Английский
Тип лицензияFree
Кол-во просмотров257
Кол-во загрузок132 раз
Обновление:11-12-2019

гдз контурная карта по географии 10 класс максаковский учебник — География 10 класс Максаковский ВП interneturokrubookgeografy 10 -klass Cached Видеоуроки, тесты и тренажёры по предмету География за 10 класс по учебнику Максаковский ВП ГДЗ по географии для 1011 класса Базовый уровень ВП reshebamegdzgeografija 10 -klassmaksakovskij Cached Качественные решения и подробные гдз по географии для учеников 10 11 класса Базовый уровень, авторы учебника:ВП Максаковский ГДЗ решебники по географии 10 класс: ответы к учебникам gdz-georuall-gdz 10 -klassgeografiya- 10 Cached ГДЗ решебники по географии 10 класс : онлайн ответы к учебникам, рабочие тетради, тесты, контрольные и самостоятельные работы, контурные карты ГДЗ решебник по географии 10 класс контурные карты Сиротин gdzputinaco 10 -klass-onlajngeografiya- 10 gdz Cached Контурные карты по географии за 10 класс автора Сиротин ВИ, 2016 года издания Готовые ответы состоят из 56 страниц и содержат географические, экономические характеристики страны, топографические карты со всеми Учебник География 10 класс ВП Максаковский 2012 Вклассе vklassework 10 -klassuchebnikigeografiyavp Cached Интересная география в 10 классе В этом году ученики начнут работать с пособием учебник География 10 класс ВП Максаковский 2012 года Материал преподнесен очень интересно и доступно Решебник Контурные карты по географии Дик и Дрофа 10 класс гдз gndakorggdz 10 32 Cached Контурные карты по географии Дик и Дрофа 10 класс Задание не найдено Карта мира ГДЗ по Географии 1011 класс ВП Максаковский Базовый eurokiappgdzgeografiya 10 classmaksakovskij Cached Приветствуем на образовательном портале Еуроки Здесь вы найдете ГДЗ с подробным и полным решением упражнений (номеров) по Географии за 10 11 класс , автор: ВП Максаковский Базовый уровень Издательство: Просвещение ГДЗ решебник по Географии 10 класс Максаковский 2011 gdzmonsternet 10 -klassgdz-po-geografiimaksakovskij Cached Рабочая тетрадь по Географии 10 класс Максаковский 2012; Рабочая тетрадь по Географии 10 класс Максаковский 2012; Рабочая тетрадь по Географии 10 класс Максаковский 2012 ГДЗ готовые контурные карты по географии 10 класс gdz-georuall-gdz 10 -klassgeografiya- 10 Cached ГДЗ по географии за 10 класс автора Приваловского АН 2013 года издания ДиК и Дрофа Данный сборник содержит точно и четко заполненные контурные карты, на которых изображены материки, границы ГДЗ (решебник) по географии 10 класс Максаковский globuss24rugdzrabocaa-tetrad-po-geografii Cached ГДЗ (решебник) по географии 10 класс Максаковский Рабочая тетрадь по географии для 10 класса под редакцией В П Максаковского создавалась специально для проведения практических и 1 2 3 4 5 Next 27,300

  • ГДЗ по другим предметам gt;gt;. Ответы к рабочей тетради по географии 10 класса. Максаковский. Ответ
  • ы к рабочим тетрадям, контурным картам и вопросам учебника.
    DonKniga.com.ua — интернет-магазин ДонКнига, купить книгу, продажа книг, доставка книг по Киеву и Киевской области, по всей территории Укра
  • нига, купить книгу, продажа книг, доставка книг по Киеву и Киевской области, по всей территории Украины, литература.
    Гдз По Комплексному Анализу Текста 6 Класс. Аннотация, отзывы читателей,. 62962887334893 гдз география 10 класс максаковский задания. На нашем блоге вы сможете найти контрольную работу под любой учебник.
    Гдз, решебники, рабочие тетради.. Anatu гдз по истории спб 6 класс краеведение ермолаева 1 часть 0 в Детский мир им. anatu.
    Гдз п русскому языку 6 клaсс. Гдз по мaтемaтике 3 клaсс рудницкaя юдaчевa. Гдз по физике 10 клaсс лaборaторные рaботы губaнов.
    ГДЗ по английскому языку. ГДЗ по географии (2) 7 класс Вашему вниманию предлагается … Скачать бесплатно Контурные карты 6 класс. Максаковский В.П. Рабочая …
    Зачет по теме Политическая карта мира 3. Аудио-, видеоматериалы 1 Комплект интерактивных карт по географии (CD диски) 62. 4 Тестовые материалы для подготовки к ГИА 9 класс.
    Данные гдз книг и тетрадей помогут вам проверить выполненное домашние задание. ГДЗ от Путина ру — онлайн решебники (ГДЗ) к учебникам и рабочим тетрадям.

отзывы читателей

по всей территории Украины

  • авторы учебника:ВП Максаковский ГДЗ решебники по географии 10 класс: ответы к учебникам gdz-georuall-gdz 10 -klassgeografiya- 10 Cached ГДЗ решебники по географии 10 класс : онлайн ответы к учебникам
  • топографические карты со всеми Учебник География 10 класс ВП Максаковский 2012 Вклассе vklassework 10 -klassuchebnikigeografiyavp Cached Интересная география в 10 классе В этом году ученики начнут работать с пособием учебник География 10 класс ВП Максаковский 2012 года Материал преподнесен очень интересно и доступно Решебник Контурные карты по географии Дик и Дрофа 10 класс гдз gndakorggdz 10 32 Cached Контурные карты по географии Дик и Дрофа 10 класс Задание не найдено Карта мира ГДЗ по Географии 1011 класс ВП Максаковский Базовый eurokiappgdzgeografiya 10 classmaksakovskij Cached Приветствуем на образовательном портале Еуроки Здесь вы найдете ГДЗ с подробным и полным решением упражнений (номеров) по Географии за 10 11 класс
  • контурные карты ГДЗ решебник по географии 10 класс контурные карты Сиротин gdzputinaco 10 -klass-onlajngeografiya- 10 gdz Cached Контурные карты по географии за 10 класс автора Сиротин ВИ

гдз контурная карта по географии класс максаковский учебник Поиск в Нажмите здесь , если переадресация не будет выполнена в течение нескольких секунд Все Картинки Новости Видео Карты Покупки Книги Инструменты поиска На всех языках На всех языках Только на русский За всё время За всё время За час За часа За неделю За месяц За год Все результаты Все результаты Точное соответствие ГДЗ по географии класс рабочая тетрадь Максаковский География ГДЗ готовые домашние задания к рабочей тетради и контурные карты по географии класс Максаковский гдз по географии максаковский контурные карты класс wwwfkkacth file_editor gdzpoge класс География Рабочая тетрадь по географии за класс автора Максаковский ВП года издания ГДЗ гдз по контурным картам класс максаковский октября gojovgirru окт г Гдз решебник по географии класс контурные карты рабочая тетрадь максаковский анализ Сделанные Контурные Карты По Географии Класс blog sd ГДЗ контурных карт географии класс содержат краткие сведения про основные страны Европы, Вам так же будут интересны География класс Рабочая тетрадь Максаковский Решебник Контурные карты по географии Дик и Дрофа gdz Контурные карты по географии Дик и Дрофа класс Контурные карты по географии Дик и Дрофа Задание не ГДЗ по географии класс рабочая тетрадь Максаковский класс Рабочая тетрадь с комплектом контурных карт по географии класс ФГОС Сиротин Дрофа Контурные карты Рабочая тетрадь по географии, класс Максаковский geogdzru rabochayatetradpo Ответы к рабочей тетради по географии класса Максаковский Максаковский Политическая карта мира гдз карта по географии класс Insulectro wwwinsulectrocom content_media g сент г гдз карта по географии класс ГДЗ готовые контурные карты по географии класс гдз география класс задания DomNouta img uploads gd сент г гдз география класс задания тетради и контурные карты по географии класс Максаковский География, класс, Рабочая тетрадь, Максаковский ВП, контурная карта по географии класс решебник document Аккуратная сделанная контурная карта по географии за класс с ответами тетрадь по географии класс Максаковский Решебник ГДЗ рабочая тетрадь по Политическая карта мира гдз контурные карты австралия класс greatwallzapru userfiles gdzkonturn сент г Контурные карты по географии за класс ответы сверяйся безплатно и без регистрации! ГДЗ решебник по географии класс рабочая тетрадь Максаковский класс География классы География Книги для школы Книги классы Контурные карты к УМК В П Максаковского ФГОС Учебник Базовый и углубленный уровни Доставка Владимир Максаковский География класс Учебник География максаковский гдз Контурные карты по географии thread окт г Рабочая тетрадь по географии за класс автора Максаковский ВП года издания Решебник по географии класс максаковский контурная карта онлайн panafdouru thread окт г Гдз по географии класс рабочая тетрадь контурные карты максаковский онлайн Решебник по контурные карты класс италия гдз mapectcom upload fckeditor kontu окт г Ctrl ГДЗ по географии класс рабочая тетрадь Максаковский gdz putinainfo Альбом контурные карты по географии класс contour maps in geography class ГДЗ решебник по географии класс рабочая тетрадь geografiya gdz нашем сайте доступны ответы к рабочей тетради и контурные карты по географии класс Максаковский гдз контурные карты класс ресурсы etuvszoxg sungprescomsleeklablogcom a ГДЗ решебник география класс Дрофа и Дик контурные карты среды Рабочая тетрадь класс Максаковский Ответы и ГДЗ Задание Обеспеченность На контурную карту рис гдз география максаковский контурная карта tbmmovaby images_from_html_editor сент г ГДЗ решебник по географии класс контурные карты Сиротин География Рабочая Тетрадь Класс Ответы Решенные задания контурных карт и рабочих тетрадей по PDF География класс uploads file Курс Экономическая и социальная география мира в старших классах Контурные карты для класса В ПМаксаковский Рабочая тетрадь по географии класс, пособие гдз география класс бахчиева wwwolympicwroclawpl zdjecia fck сент г География учебник для класс максаковский в п Гдз контурные карты по географии класс б by Dropr droprcom portfolio Гдз контурные карты по географии класс бахчиева гдз по географии контурные карты класс кузнецов wwwborscz img wswg gdzpoge дек г гдз по географии контурные карты класс кузнецов класс учебник Кузнецов ответы gdzputinainfo класс ГДЗ рабочая тетрадь география класс Максаковский География класс Максаковский ГДЗ контурные и worldluxrealtycom geografiyaklas дек г Контурные карты, рабочая тетрадь и ответы Максаковский географии класс ГДЗ география класс гдз украина akvamotorsru upload geografiia окт г ГДЗ класс География контурные карты Учебник География Украины класс Пестушко Уварова Г Ш, Пестушко РешеноТема ГДЗ Максаковский класс рабочая Тема из Гдз по географии контурные карты класс максаковский levelapru images dorofeev gdzpo Я думаю, что он простит нас когда узнает, что мы преданы и пытались по география контурные карты Решебник ГДЗ по географии за класс класс Подробный решебник гдз по Географии за класс к учебнику школьной География класс контурные карты География класс рабочая тетрадь автор Максаковский ВП Контурные карты География классы к учебнику ВП book География классы к учебнику ВП максаковского Контурные карты по географии Рабочая тетрадь к учебнику Помощь и ответы на вопросы Контакты и реквизиты СМИ и партнерам Оптовым клиентам и ФЗ Контурные карты по географии класс гдз максаковский eituibiewaskyblazersno kckkonturny Контурные карты по географии класс гдз максаковский Осень явилась Учебник с решениями ФГОС Выберите номер страницы или упражнения на главы и параграфы Версия География класс купить учебники в интернет магазине geografiya Учебники по Географии для класса, предоставляем скидку при покупке на Атлас контурные карты Автор Максаковский; Издательство Просвещение; Класс класс Ответы на самые популярные вопросы Решебник контурные карты по географии класс максаковский matuquigulightningjorg utwreshebn С помощью ГДЗ по алгебре автора Мордковича любой старшеклассник решебник контурные карты по географии класс максаковский учебник, что География класс контурная карта гдз alligatortomskru zkkgeo Гдз решебник по географии класс рабочая тетрадь максаковский Гдз ответы на вопросы к рабочей Атлас и Контурные карты по географии класс Zegohi zegohirezytirunet hefoxu Решебник ГДЗ рабочая тетрадь география класс контурные карты по географии класс максаковский Контурные карты по географии класс К учебнику catalog book Контурные карты по географии класс К учебнику Максаковского География классы Карташева Т Картинки Показать все Показать все Книга География классы Контурные карты context detail Оценка , Книга География классы Контурные карты характеристики, фото и отзывы покупателей Доставка К учебнику В П Максаковского с любым учебником по географии, включённым в Федеральный перечень учебников Тема География отраслей мирового хозяйства ГДЗ и klass tema География отраслей мирового хозяйства Рабочая тетрадь класс Максаковский Ответы и ГДЗ Задание гдз по географии класс arkaimavtoru store file gdzpogeo сент г гдз по географии класс ГДЗ рабочая тетрадь по географии класс Домогацких ФГОС ответы и контурные карты по географии класс Максаковский Решебник По География класс Решение задач по математике geo Книги, учебники, решебники, ГДЗ, тесты и контрольные работы с ответами по Книги и учебники География класс классы Контурные карты и сборник задач г Учебник для класса Максаковский ВП г ГДЗ решебники по географии класс ответы к учебникам Решебники класс ГДЗ готовые контурные карты по географии класс Кузнецов АСТПРЕСС География Контурные карты Книги по географии по классам Nasholcom poklassam класс Атлас с комплектом контурных карт, экономическая и социальная география мира, природные ресурсы, Гдз контурные карты география класс максаковский учебник ooweecheefelixwostalde ikggdzko Гдз контурные карты география класс максаковский учебник Продажа осуществляется только в сувенирномкиоске штабквартиры организации Гдз география класс латинская америка gdzgeografiyaklasslatinskayaamerikafortunafishrfru Гдз по Географии класс Контурные карты Дрофа Дик Гдз по Решебник ГДЗ рабочая тетрадь по географии класс Максаковский Подчеркните те из приведенных ниже природных контурная карта класс китай гдз wwwsydnarkenyttse userfiles kontur окт г Гдз по географии беларус; и класс контурная карта окружающей среды Рабочая тетрадь класс Максаковский Ответы и по запасам железной руды Австралия, Бразилия, Гдз по географии класс максаковский крассворд по pin Гдз по географии класс максаковский крассворд по японии Решебник рабочая тетрадь по истории Решебник контурные карты начальный курс по географии класс авторыип галай карта мира гдз по географии przedszkolelesznoeu userfiles karta окт г Контурные карты по географии за класс ответы сверяйся безплатно и без регистрации! Политическая карта мира Онлайн ГДЗ Максаковский класс рабочая тетрадь по гдз от путина география контурные карты класс xohaliprofsoyuzohranaru окт г География класс рабочая тетрадь максаковский География география класс гдз контурные карты германия класс blentechru pic gdzkonturnyekarty сент г Альбом контурные карты по географии класс contour maps in geography class Тема Зарубежная Европа Рабочая тетрадь класс Максаковский Ответы и ГДЗ Задание Контурные карты по географии класс ФГОС subject k класс ГДЗ География класс Рабочая тетрадь по географии класс Максаковский Просвещение ГДЗ география класс geografiya Рабочая тетрадь по Географии класс Контурные карты Приваловский Обложка Рабочая тетрадь по География Рабочая тетрадь классы Максаковский ВП geog geo Рабочая тетрадь по курсу географии классы предназначена для Политическая карта мира Тема ГДЗ решебник по географии класс рабочая тетрадь класс География CENTER вы найдете ответы к рабочей тетради и контурные карты по географии класс Максаковский Вы В ответ на официальный запрос мы удалили некоторые результаты с этой страницы Вы можете ознакомиться с запросом на сайте LumenDatabaseorg В ответ на официальный запрос мы удалили некоторые результаты с этой страницы Вы можете ознакомиться с запросом на сайте LumenDatabaseorg Похожие запросы гдз контурные карты по географии класс дрофа гдз по географии класс контурные карты гдз по географии класс максаковский рабочая тетрадь гдз по географии учебник класс максаковский учебник гдз по географии класс максаковский ответы на задания гдз контурные карты по географии класс дрофа гдз контурные карты по географии класс дрофа рабочая тетрадь по географии класс максаковский онлайн Следующая Войти Настройки Конфиденциальность Условия

ГДЗ по другим предметам gt;gt;. Ответы к рабочей тетради по географии 10 класса. Максаковский. Ответы к рабочим тетрадям, контурным картам и вопросам учебника.
DonKniga.com.ua — интернет-магазин ДонКнига, купить книгу, продажа книг, доставка книг по Киеву и Киевской области, по всей территории Украины, литература.
Гдз По Комплексному Анализу Текста 6 Класс. Аннотация, отзывы читателей,. 62962887334893 гдз география 10 класс максаковский задания. На нашем блоге вы сможете найти контрольную работу под любой учебник.
Гдз, решебники, рабочие тетради.. Anatu гдз по истории спб 6 класс краеведение ермолаева 1 часть 0 в Детский мир им. anatu.
Гдз п русскому языку 6 клaсс. Гдз по мaтемaтике 3 клaсс рудницкaя юдaчевa. Гдз по физике 10 клaсс лaборaторные рaботы губaнов.
ГДЗ по английскому языку. ГДЗ по географии (2) 7 класс Вашему вниманию предлагается … Скачать бесплатно Контурные карты 6 класс. Максаковский В.П. Рабочая …
Зачет по теме Политическая карта мира 3. Аудио-, видеоматериалы 1 Комплект интерактивных карт по географии (CD диски) 62. 4 Тестовые материалы для подготовки к ГИА 9 класс.
Данные гдз книг и тетрадей помогут вам проверить выполненное домашние задание. ГДЗ от Путина ру — онлайн решебники (ГДЗ) к учебникам и рабочим тетрадям.

Решения ICSE

для географии класса 10 — Изучение карты: интерпретация и топографические карты

Решения ICSE для географии класса 10 — Изучение карты: интерпретация и топографические карты

Решения ICSE

Селина Решения ICSE

APlustopper.com предоставляет решения ICSE для изучения географии класса 10 Глава 1: интерпретация и топографические карты для экзаменов Совета ICSE. Мы предоставляем пошаговые решения для решений ICSE Geography Class 10 Pdf.Вы можете загрузить учебники ICSE по географии для 10 класса с возможностью бесплатной загрузки в формате PDF.

Загрузить справочник формул для классов 9 и 10 ICSE

Очень короткие вопросы

Вопрос 1: Что такое плоская земля?
Ответ: Земля высотой от 20 до 40 метров над уровнем моря называется равниной. Этот тип территории имеет достаточный уклон для естественного дренажа. Этот район вообще подходит для земледелия.

Вопрос 2: Что вы понимаете под термином Grid?
Ответ: Сетка — это набор линий, используемых для определения точного местоположения мест на карте.Большинство карт используют сетку, образованную широтой и долготой, чтобы найти места.

Вопрос 3: Что такое «National Grid Reference»?
Ответ: Линии сетки — это воображаемые линии, нарисованные на карте съемки красным цветом. Начало сетки — точка к юго-западу от карты. «Национальная сетка привязки» используется для указания правильного положения определенного объекта на карте.

Вопрос 4: Опишите важность цветов на топографическом листе?
Ответ: Использование различных цветов необходимо для отображения различных функций на верхнем листе. E.грамм. посевная площадь всегда отображается желтым цветом, а зеленый цвет на верхнем листе показывает лесные массивы и лесные массивы.

Вопрос 5: Что вы понимаете под мостовой?
Ответ: Это поднятая дорога из металла, проложенная над ручьем, рекой, железнодорожной линией или низменной болотистой местностью.

Вопрос 6: Что такое линия огня?
Ответ: Линия огня — это тропа, расчищенная вокруг леса для предотвращения распространения огня.

Вопрос 7: Что такое речная система
Ответ: Реки обычно поднимаются по склонам и соединяются с большим количеством других небольших рек или ручьев. Вся территория, осушаемая заклепкой и ее притоками, называется речной системой.

Вопрос 8: Как важность поселения отражена на карте?
Ответ: Важность населенного пункта отражается в его размере и количестве маршрутов, которые в нем пересекаются.

Короткие вопросы

Вопрос 1: Что такое «Топографическая карта»?
Ответ: Топографическая карта дает подробную информацию о физических и антропогенных особенностях небольшой территории.Физические особенности изображены контурами. Культурные или антропогенные особенности представлены символами, называемыми условными знаками.

Вопрос 2: Что такое шкала?
Ответ: Масштаб очень важен для карты, потому что он представляет большую территорию страны на небольшом листе бумаги. Масштаб — это отношение между любыми двумя точками на карте к соответствующему расстоянию между теми же двумя точками на реальной земле.

Вопрос 3: Какие три метода обозначения масштаба на картах?
Ответ: Три метода обозначения масштаба на картах:
(i) Утверждением (ii) Репрезентативной дробью
(iii) Графическим или линейным масштабом.

Вопрос 4: Преобразовать следующие шкалы в R.F.
Ответ: Если масштаб от 2 см до 1 км, рассчитайте R.F.

Вопрос 5: Преобразуйте следующие значения R.F. в шкалы.
Ответ: Если R.F., это 1: 2,500,000, какой масштаб (ответ в сантиметрах и километрах).

Вопрос 6: Что вы подразумеваете под «маржинальной информацией»?
Ответ: Поля Информацию можно получить на полях листа.Название штата, которому принадлежит территория, дано вверху листа. Также указывается год обследования местности. Номер листа указан в правом верхнем углу. Направление магнитного склонения и истинное положение на север также указано в правом верхнем углу.

Вопрос 7: Почему важны направления на карте?
Ответ: Поскольку картографирование — это в первую очередь наука о местонахождении, изучающий географию должен быть знаком с направлениями, а также с методами определения направлений в различных случаях и при различных обстоятельствах.

Вопрос 8: Укажите методы определения направлений.
Ответ: Некоторые методы определения направлений:
(i) Угловой пеленг (ii) Магнитный компас
(iii) Полярная звезда (iv) Восход и закат.

Вопрос 9: Как направление по компасу объекта, особенности или места задается из данной точки?
Ответ: (i) Направление: верхняя часть карты — север, ваша правая рука — восток, а левая — запад.Нижняя часть — Юг. Вы должны быть осторожны, из какого города или объекта указывать направление.

Из B — A в северо-восточном направлении
Из A — B в юго-западном направлении
Из Y — X в SE направлении
Из P — M в южном направлении
Из M — P в северном направлении
Из X — Y находится в северо-западном направлении
(ii) Пеленг: направление может быть указано с помощью системы углового пеленга. Пеленг — это горизонтальный угол между севером и линией, соединяющей положение наблюдателя и объекта по часовой стрелке.Пеленг от наблюдателя к объекту называется прямым пеленгом (FB). Пеленг от объекта к наблюдателю называется обратным пеленгом (BB).

Вопрос 10: Рассчитайте средний уклон вдоль двух станций A и B, если расстояние по горизонтали между ними составляет 1 200 метров, а перепад высот по вертикали — 240 метров.
Ответ:

Вопрос 11: Что такое водораздел?
Ответ: Водораздел или водораздел — это гребень, который отделяет притоки, стекающие в одну сторону, от притоков, впадающих в другую.Водораздел никогда не проходит на полпути вверх по склону, и с двух сторон от него уклон должен быть противоположным и направленным вниз.

Вопрос 12: Что такое «восток» и «север»?
Ответ: Положение определенного объекта на листах указано в восточном и южном направлениях. Линии сетки, направленные на восток, известны как северные полюса. Линии сетки, направленные на север, известны как восточные направления. Они были названы восточными и северными, потому что они указывают расстояния на восток и север соответственно от исходной точки.

Вопрос 13: Площадь измеряется методом квадратов сетки?
Ответ: Чтобы найти площадь методом квадратов сетки: Квадрат сетки на верхнем листе имеет размер 2 см × 2 см или 1 км, × 1 км, поскольку 2 см, равняется 1 км. Таким образом, площадь каждого квадрата составляет 1 кв. Км. Чтобы найти конкретную область на топографической карте:

Обратите внимание, что количество полных квадратов сетки внутри области и количество квадратов, лежащих внутри области, не являются полными.
(i) Квадратная закрытая половина принимается как 1/2 квадрата.
(ii) Площадь, покрытая более чем наполовину, принимается как 2/3 квадрата.
(iii) Квадрат, покрытый менее чем наполовину, считается 1/3 квадрата.
Сложите все квадраты таким образом, чтобы вычислить площадь, которую необходимо вычислить.

Вопрос 14: Какие цвета используются на топо-листах? В чем их значение?
Ответ: В геодезической карте используются шесть цветов, чтобы показать различные особенности. Это следующие:
Черный: все названия, берега рек, изрезанные участки, сухие потоки, обследованные деревья, высота и их нумерация, железнодорожные пути, телефонные и телеграфные линии.
Желтый: все посевные площади.
Зеленый: все лесные / засаженные деревьями области, отдельные деревья и кустарники.
Коричневый: Контурные линии, их нумерация, каменистые отходы, песчинки.
Синий: все водоемы, в которых есть вода.
Красный: Линии сетки и их нумерация, дороги, подъездные пути, поселения, хижины и другие постройки. Примечание: иногда мы находим белые пятна здесь и там, чтобы показать бесплодные земли.

Вопрос 15: Что такое узловой центр?
Ответ: Город, который находится на пересечении многих маршрутов, обычно очень важен и известен как узловой центр.Узловой центр обычно имеет плотное поселение. Большие города с плотной застройкой отражают наличие промышленности и торговли. Доступность полезных ископаемых также может вызвать рост больших городов.

Длинные вопросы

Вопрос 1: Каким образом масштаб отображается на карте?
Ответ: Представление масштаба карты: Масштаб карты представлен в трех формах:
(i) Утверждение: Это самый простой метод описания масштаба карты.На картах обычно пишется от 1 см до 10 км.
(ii) A Соотношение: в этом методе масштаб карты выражается в числовом соотношении. Числитель представляет расстояние на карте и всегда выражается в единицах измерения. Знаменатель представляет собой соответствующее расстояние на земле, то есть одна единица на карте эквивалентна количеству единиц на земле.
Он также известен как шкала соотношений или репрезентативная фракция (R.F.).

(iii) График: В этом методе масштаб представлен с помощью линейного (линейного) графика.Он состоит из прямой линии, разделенной на ряд равных частей (первичной и вторичной), которые отмечены, чтобы показать, что эти деления представляют на фактическом основании.

Вопрос 2: Расстояние измеряется на топографической карте. Существует два метода измерения расстояния на топошлитах: (i) прямой метод, (ii) косвенный метод. Измерение прямого расстояния — это прямой метод, а измерение по намотке — косвенный метод. Объясните оба метода.
Ответ: (i) Измерение прямого расстояния: Измерение кратчайшего расстояния между двумя точками на топографической карте вдоль прямой линии может быть легко выполнено с помощью пары разделителей.Держите два заостренных конца разделителя над линией или двумя точками, которые необходимо измерить, а затем тщательно держите на шкале. Запишите расстояние на карте в сантиметрах; затем либо с помощью графической шкалы, которая представлена ​​под топографическим листом, либо с помощью шкалы утверждений, расстояние может быть измерено в километрах и метрах.
Например, два места на карте находятся на расстоянии 5,6 км.
Масштаб указан 2 см. = 1 км.
1 см. = 1/2 км.
5,6 см. = 1/2 × 5,6
= 2,8 км.
(ii) Для измерения курса по изгибу: измерить маршрут по извилистому маршруту сложно. Его нужно измерить с помощью полоски бумаги или нитки, например, чтобы измерить реку на топографическом листе, возьмите кусок нити и завяжите узел на одном конце. В конце нити (близко к узлу) отметьте ручкой точку, с которой вы начнете измерение (точка A). Проведите нить по линии, соприкасаясь, насколько это возможно для измерения, и продолжайте отмечать чернилами на поворотах до последней точки. Протяните нить на шкале, вы получите расстояние до земли в километрах и метрах, всегда важна единица измерения e.г., если рассчитанное расстояние между двумя точками составляет 3,3 км. Если спросить в километрах, ответ будет 3км., 300м. Если спросить в метрах, ответ будет 3300 метров.

Вопрос 3: Что такое вогнутый уклон и выпуклый уклон? Объяснять?
Ответ: Вогнутый склон: склон очень пологий у подножия, и к вершине он становится все круче и круче. Расстояние между последовательными контурами на нижнем уровне больше, чем на верхнем уровне.

Выпуклый склон: склон очень крутой у подножия и постепенно становится пологим к вершине.Таким образом, контурные линии расположены неравномерно, но есть последовательность, которая прямо противоположна диаграмме вогнутых уклонов. Расстояние между последовательными контурами на более низком уровне меньше, чем на
на более высоком уровне.

Вопрос 4: (i) На топографической карте две станции A и B расположены на высоте 500 метров и 100 метров соответственно. Если расстояние между ними по горизонтали 2 км, рассчитайте уклон.
(ii) На топографической карте даны два места A и B.Рассчитайте градиент между ними.
Ответ: (i) Горизонтальное расстояние = 2 км = 2000 м
Вертикальный перепад высот = 500 — 100 = 400 метров

(ii) Горизонтальное расстояние: Измерьте расстояние в сантиметрах. между двумя местами с вашей линейкой. Затем из Р.Ф. на топографическом листе рассчитайте фактическое расстояние между двумя точками в км, а затем преобразуйте его в метры.
Вертикальный интервал по высоте: Найдите разницу между значениями двух контуров, в которых лежат места, или, если горизонтальные линии не показаны, найдите разницу в высотах пятен для этих двух мест.

Вопрос 5: (i) В чем важность карты рельефа.
(ii) Как мы можем показать занятия по свидетельствам имен или по рельефным признакам?
Ответ: (i) Изучение рельефа требует осторожного обращения. Это вид на все с высоты птичьего полета. область, где четко показаны холмы, плато и низины. Отмечен промежуток между двумя холмами. Максимальные высоты могут быть определены и обозначены с привязкой к сетке.
(ii) В следующем списке профессии показаны с указанием названий или с учетом рельефных особенностей.

Доказательства или выводы о помощи Расширение профессий
(i) Пятно зеленого леса Лесное хозяйство
(ii) Желтый цвет и сады Сельское хозяйство
(iii) Пастбища и луга или кустарники Выпас крупного рогатого скота или разведение овец.
(iv) Карьеры и шахты Разработка карьеров и горнодобывающая промышленность
(v) Поселок возле основных дорог — Ежегодная ярмарка (письменная) Торговля
(vi) Шахты и фабрики Промышленное развитие
(vii) Парк, поля для гольфа, тиры Отдых и культурное развитие

Вопрос 6: Назовите важные особенности речной долины.
Ответ: Форма контуров позволяет выделить различные особенности речной долины. Важными особенностями речной долины являются следующие:
Направление реки: Его можно узнать, изучив общий уклон местности. Река всегда течет с более высокого уровня на более низкий уровень, поэтому в каком бы направлении ни была наклонная земля, река будет течь в этом направлении, все небольшие ручьи также присоединяются к реке в том же направлении.

Вопрос 7: Покажите различные схемы дренажа.
Ответ: Типы дренажа:

Вопрос 8: Как найти ссылку на четырех- и шестизначную сетку?
Ответ: Ссылка на четырехзначную сетку: Квадратная сетка или ссылка на четырехзначную сетку, например, ссылка на четырехзначную сетку для A на рисунке — 6218. Это означает, что A находится между восточными координатами 62–63 и северными точками 18–19.

Ссылка на шестизначную сетку: Ссылка на шестизначную сетку указывает точное местоположение объекта.Например, шестизначная ссылка на сетку A будет 623188. Первые три цифры указывают на восток, а следующие три цифры — на север. Чтобы получить третью цифру Востока, нужно мысленно разделить пространство между 62 и 63 на 10 равных частей.
Таким образом, третья цифра дойдет до 3. Аналогично, чтобы получить шестую цифру, мысленно разделите пространство между 18 и 19 на 10 равных частей. Таким образом, шестая цифра равна 8. Шестизначная ссылка на сетку A равна 623188.
Квадраты сетки на топошитах обычно составляют 1 кв. Км..
Иногда встречаются квадраты, в которых написано QC и QD. Они обозначают двухбуквенные номера площадью 100 кв. Км.

Вопрос 9: Назовите различные типы схем расчетов, которые можно найти на верхнем листе.
Ответ:

Вопрос 10: Укажите значения важных терминов, используемых в топошетах.
Ответ: Open Scrub: Это земля, где ничего не растет. Это знак пустыни. Он указывает на занятие как разведение овец или коз.
Обнажение скалы: обнаженная часть скалы. Выращивание невозможно.
Листовой камень: Это масса или плита, часть которой выступает над поверхностью. Показано белым, это тоже напечатано.
Линия огня: в лесу проводится расчистка для проверки распространения огня.
Causeway: Causeway — это приподнятая дорога или платформа через небольшой ручей (не путать с мостом). Его наличие указывает на районы со скудным или сезонным выпадением осадков. Ручей, имеющий дамбу, доступен для движения в сухой сезон.
Изломанная земля: это выветренная часть земли, образовавшаяся в результате эрозии во время сильного дождя во время наводнения. Он встречается в основном в засушливых регионах вокруг ручьев и рек. На верхнем листе это показано крошечной изогнутой черной линией.

Акведук: это канал, по которому течет вода.

Карьер: это карьер, из которого добывается известняк, мрамор, камень, песок или глина для строительства или других целей.
Печь для обжига извести: это завод по производству извести. Известь используется для изготовления строительного раствора, который используется для скрепления кирпичей.
Сухой колодец: слово «сухой», написанное в скобках возле колодца, обозначает колодец, который остается сухим в сухой сезон.
Солоноватая вода: слово «солоноватая» в скобках возле колодца означает, что вода в колодце имеет высокое содержание соли и не пригодна для питья. Можно использовать для полива.

Вопрос 11: Что такое контрольные показатели?
Ответ: Контрольные отметки: это высоты, которые обозначены буквами BM, например, BM 942 м., На камнях, вделанных в землю, на видных зданиях или на скалах для постоянной ссылки для целей исследования.Они показывают точные высоты, определенные после надлежащей съемки. Они представлены как:
(i) Триангулированная станция: это высоты, показанные с помощью треугольника с высотой, указанной рядом с ним, например. ∆ 549. Высота не очень точна, так как расстояние до земли измеряется с помощью угловых измерений и рассчитывается с помощью тригонометрии.
(ii) Высота пятна: маленькая точка с высотой 549, напечатанная рядом с ней, видна по всей карте. Они представляют собой положение на земле, высота которого была точно определена.Это обследованная высота, она существует только на карте.
(iii) Относительная высота: Относительная высота — это разница между самой высокой и самой низкой отметкой на местности. Относительная высота, например 25р против колодца синего цвета, укажите его глубину в метрах. Относительная высота, например 10r напротив насыпи черным цветом показывает ее относительную высоту в метрах. Число — высота в метрах.

Вопрос 12: Кратко опишите размеры и масштаб топографических карт.
Ответ:

Топография

Вопрос 1: Изучите выдержку из Карты Обзора Индии №45D / 10 и ответьте на следующие вопросы:
(a) (i) Дайте четырехзначную сетку координат поселения Хамирпура.
(ii) Дайте ссылку на шестизначную сетку храма в поселении Джолпур.
Ответ: (i) 0123 (ii) 079187
(b) (i) Что обозначает синий кружок в квадрате сетки 0619?
(ii) Какое направление по компасу на Дантрай от Джолпура?
Ответ: (i) Облицованный многолетний колодец (На карте кружок не показан)
(ii) Северо-запад
(c) В чем разница между:
(i) Схема поселений в 0725 г. а поселок Идарла?
(ii) Схема дренажа ручьев в 0624 и 0824?
Ответ: (i) Поселение 0725 г. — рассредоточено.
Поселение в Идарле — Nucleated.
(ii) Дренаж 0624 — дендритный.
Дренаж 0824 — дендритный
(d) (i) Какое значение имеет контурная линия в квадрате 0226?
(ii) Каков интервал изолиний на карте?
Ответ: (i) 300 метров.
(ii) 20 метров.
(e) Назовите любые два фактора, свидетельствующие о том, что регион на отрывке карты является сельским.
Ответ: (i) Нет крупного населенного пункта
(ii) Нет дороги с металлизированным покрытием
(f) (i) Каким образом объект, обозначенный черными кривыми в 0721, показывает, что осадки в этом регионе являются сезонными?
(ii) Упомяните на карте один объект, созданный руками человека, который также свидетельствует о том, что количество осадков носит сезонный характер.
Ответ: (i) Черная изогнутая линия в 0721 представляет собой изломанную землю. Измельченный грунт образуется из-за чередования засушливых и влажных периодов вдоль берегов сезонной реки, где почва мягкая.
(ii) Большое количество высаженных многолетних колодцев.
(g) (i) Назовите два природных объекта в 0527.
(ii) Назовите два искусственных объекта в 0325.
Ответ: (i) В 0527 естественными объектами являются сезонные деревья ручьев и бесплодные земли.
(ii) Автомобильная дорога в засушливый сезон и поселок.
(h) Назовите две особенности, которые делают Дантрай более важным поселением, чем другие поселения в отрывке карты.
Ответ: В поселке Дантрай есть полицейский участок и почта. В других населенных пунктах их нет.
(i) Вычислите площадь региона, лежащего к югу от северной широты 21, в квадратных километрах.
Ответ: Длина — 19,5 см → 9,75 км
Ширина — 6 см → 3 км (по шкале от 2 см до 1 км)
Площадь 9.75 × 3 = 29,25 кв. Км.
(j) Что это следующие?
(i) Черная вертикальная линия между восточными координатами 09 и 10.
(ii) 302 в квадрате сетки 0425.
Ответ: (i) Долгота
(ii) 302 в квадрате сетки 0425 — это высота пятна.

Вопрос 2: Изучите выдержку из карты Обзора Индии № 45D / 10 и ответьте на следующие вопросы:
(a) Назовите и дайте четырехзначную ссылку на сетку населенного пункта, где проживают жители региона. встречаться не реже одного раза в год.
Ответ: Malgaon 1520.
(b) Что означает условный символ в сетке 145132?
Ответ: Высота пятна 270.
(c) Если бы человек пошел из Гулабганджа (1820 г.) в Харматию (1916 г.):
(i) В каком направлении он бы пошел?
(ii) Какие два типа дорог он будет использовать?
Ответ: (i) Юго-восток (ii) Металлическая дорога и колея для тележек
(d) Что означают следующие числа в квадрате сетки 1718 и 1818?
(i) 280 (ii) 281
Ответ: (i) Высота контура 280 м (ii) Высота пятна 281 м
(e) В чем два различия между поселением Бамба в 1914 году и поселением в 1813 году?
Ответ: В Бамбе поселения являются зарождающимися и временными, а в квадрате 1813 поселения рассредоточены и постоянны.
(f) Назовите четыре объекта, которые есть в Анадре, что делает ее важным поселением.
Ответ: Объекты в Анадре ПТО (ОТДЕЛЕНИЕ ПОЧТЫ И ТЕЛЕГРАФИИ), полицейский чауки, ДБ (бунгало Дак), амбулатория.
(g) Какое самое быстрое средство связи для жителей Дабани (1313 г.)?
Ответ: Телефон.
(h) Что означает следующее:
(i) Солоноватое в 1915 году (ii) Козуэй в 1715 году (iii) 6r в 1218 году?
Ответ: (i) Солоноватая: вода с высоким содержанием соли и не пригодна для питья, но может использоваться для орошения.
(ii) Дорога: Дорога над ручьем.
(iii) 6р. Относительная высота берега реки 6м.
(i) Укажите два фактора, которые подтверждают следующее:
(i) Река Сипу находится в среднем течении
(ii) Количество осадков в регионе, показанном на фрагменте карты, носит сезонный характер.
Ответ: (i) Сипу Нади находится в среднем течении, потому что там есть меандры, а земля ровная, что показывает, что река медленная.
(ii) Количество осадков носит сезонный характер, что видно по заштрихованной реке и наличию пересеченной местности.
(j) Каково расстояние в километрах между дистанционным камнем 20 в 1818 году и дамбой в 1715 году вдоль дороги с металлическим покрытием?
Ответ: Расстояние 2,8 км.

Вопрос 3: Изучите выдержку из карты Обзора Индии № 45D / 10 и ответьте на следующие вопросы:
(a) Дайте ссылку на шестизначную сетку:
(i) печь для обжига кирпича
( ii) храм возле Асава.
Ответ: (i) 089132
(ii) 059128
(b) В чем разница в схеме дренажа в квадрате сетки 0916 и 0712?
Ответ: 0916 решетка 0712 радиальная
(c) Дайте четырехзначную сетку для каждого из следующих элементов:
(i) Каменные отходы
(ii) Открытый скраб.
Ответ: (i) 1014
(ii) 0816, 0916
(d) Рассчитайте расстояние в километрах по металлизированной дороге между насыпями в квадрате сетки 0512 и 0808.
Ответ: 5,5 км
(e) (i) Что обозначают крошечные изогнутые черные линии в квадрате сетки 0315?
(ii) Какова основная причина этой особенности?
Ответ: (i) пересеченный грунт
(ii) возник в результате эрозии во время сильного дождя во время наводнения.
(f) (i) Какое географическое название вы бы дали общей структуре поселений в регионе, показанном на карте?
(ii) Объясните причину своего ответа.
Ответ: (i) ядро ​​
(ii) люди строят свои дома в районах, где есть колодцы и они могут заниматься сельским хозяйством. На карте это показано желтым цветом.
(g) Каково общее направление течения Сипу Нади, указанное на фрагменте карты? Укажите причину в поддержку своего ответа.
Ответ: с северо-востока на юго-запад. На это указывает стрелка в русле реки.
(h) Назовите две вероятные профессии жителей поселка Ревдар в клетке 0313 и 0413.
Ответ: Земледелие и выпас скота.
(i) Какие дороги соединяют (i) Марол с Митаном и (ii) Ревдар с Караунти соответственно?
Ответ: (i) Путь от Марола до Митана.
(ii) Металлическая дорога от Ревдара до Караунти.
(j) Укажите две причины, чтобы показать, что в районе, изображенном на карте, выпадают сезонные осадки.
Ответ: пересеченный грунт и насыпи в реке.

Вопрос 4: Изучите выдержку из Карты Обзора Индии №45D / 10 и ответьте на следующие вопросы:
(a) Дайте четырехзначную сетку координат населенного пункта с
(i) Высота пятна 261 (ii) Почтовое отделение.
Ответ: (i) 1107 (ii) 1003
(b) В чем разница в схеме дренажа в квадрате сетки 1606 и в квадрате 1007?
Ответ: 1606 — Радиальный 1007 — Решетка.
(c) Назовите два природных объекта, видимых в квадрате сетки 0910.
Ответ: Два природных объекта, видимых в сетке 0910:
(i) Река (ii) Изломанная земля.
(d) Рассчитайте расстояние в километрах по металлизированной дороге от дамбы к востоку от Вас до камня 10. 7.3 км.
(e) (i) Какая шкала является общепринятой?
(ii) Укажите длину данной карты в километрах.
Ответ: (i) Общепринятой шкалой является репрезентативная фракция (RF).
(ii) Длина данной карты 10.5 км (масштаб 2 см = 1 км).
(f) Дайте ссылку на шестизначную сетку:
(i) Храм к югу от поселения Дхавли
(ii) 277.
Ответ: (i) Ссылка на шестизначную сетку храма в поселении Дхавли: 111073.
(ii) Ссылка на шестизначную сетку для 277 равна –177057.
(g) Какие два разных типа дорог показаны на фрагменте карты?
Ответ: (i) Металлическая дорога (ii) Путь для тележек
(h) Назовите два основных занятия, показанных на отрывке карты.
Ответ: Две профессии:
(i) земледелие, (ii) лесное хозяйство.
(i) Как пологие и крутые склоны показаны на карте?
Ответ: Расстояние между контурными линиями указывает на пологие или крутые склоны: (i) если линии расположены близко друг к другу, это показывает крутой уклон, (ii) если контурные линии расположены широко друг от друга, это пологий уклон. .
(j) Что означает следующее?
(i) 3r in 1103 (ii) Open Scrub in 1502
Ответ: (i) 3r in 1103 показывает относительную высоту насыпи, построенной на сухом резервуаре.
(ii) Открытый кустарник не является пахотной землей, но используется для выращивания овец или коз.
(k) Куда направляются Даттани и Дхавли от Чанделы?
Ответ: Направление Даттани и Дхавли от Чанделы:
(i) Даттани находится к западу от Чанделы.
(ii) Дхавли находится на северо-западе от Чанделы.
(l) Назовите два искусственных объекта в квадрате сетки 1210.
Ответ: Искусственные объекты в квадрате сетки 1210:
(i) поселение, (ii) путь тележки.
(м) Что означают слова «Мотоцикл в сухой сезон» в квадрате сетки 1701?
Что они указывают на количество осадков, выпавших в регионе, показанном на отрывке карты?
Ответ: В районе 1701 есть дорога для повозок (дорога качха), по которой нельзя пересекать автомобили или моторы в сезон дождей, так как земля становится влажной и илистой, но в сухой сезон, когда земля твердая и сухая, моторы могут пересечь эту область.
Из карты видно, что сейчас засушливый сезон, дождей не было.

Вопрос 5: Изучите выдержку из Карты Обзора Индии № 45D / 10 и ответьте на следующие вопросы:
(a) Какое направление по компасу на точку Сансет от поселения Анадра?
Ответ: Направление по компасу точки захода солнца от поселка Анадра — юго-восток.
(b) Какова схема дренажа в квадрате 2315 сетки?
Ответ: Рисунок дренажа в квадрате сетки 2315 является радиальным.
(c) Назовите любые две детали на отрывке карты, которые указывают на то, что в регионе есть сезонные осадки.
Ответ: Существует сухая Нади, текущая из Лакхава Ка Кхера, а также несколько сухих резервуаров в этой области, которые показывают, что в этом регионе есть сезонные осадки, например, когда идет дождь, эти особенности Нади и резервуары полны воды .
(d) Вычислите расстояние в километрах вдоль дороги с металлическим покрытием от дамбы в квадрате сетки 1715 до камня расстояния, отмеченного 20 в квадрате сетки 1818.
Ответ: 9,6 см = 2,8 км.
(e) Какое преимущество имеет репрезентативная фракция над вербальной шкалой?
Ответ: Словесная шкала не является подходящим методом представления масштаба карты, поскольку в этом методе мы делаем такие утверждения, как 1 см = 1 км или 1 дюйм = 5 миль, но этот тип утверждений не может использоваться в в разных частях света, поскольку им необходимо знать единицы измерения. Но в репрезентативной фракции этой проблемы нет, поскольку единицы измерения в числителе и знаменателе совпадают.

Это означает, что 1 единица на карте соответствует 1,00 000 единиц на земле, если единица измерения — см, то это будет 1 см на карте = 1 000 см на земле. Единицу измерения можно изменить или преобразовать в любую единицу. Таким образом, он имеет международное значение и называется «Международный масштаб».
(f) Дайте ссылку на шестизначную сетку:
(i) Анджини Деви ка Мандир (ii) D 1327.
Ответ: (i) Анджини Деви ка Мандир находится в шестизначной сетке 229160.
(ii) Шестизначная сетка D 1327 = 217105.
(g) Назовите три различных типа дорог в квадрате сетки 2411 и один в квадрате сетки 2515.
Ответ: Три разных типа дорог в квадрате сетки 2411:
(i) Металлическая дорога (ii) ) Неметаллическая дорога (iii) Путь
Один из видов дороги в квадрате сетки 2515 — «Пешеходная дорожка».
(h) Укажите в отрывке карты два занятия жителей северной части региона. Приведите причины, подтверждающие ваш ответ.
Ответ: Люди, живущие в северном регионе отрывка карты, занимаются двумя основными занятиями:
(i) Земледелие: В северо-западном регионе фрагмента карты люди принимают земледелие как свое занятие, потому что здесь «желтая стирка» регион показывает, что земля.культивируется, во-вторых, в нем много многолетних колодцев, которые поддерживают выращивание в этом регионе.
(ii) Лесное хозяйство: в северо-восточном регионе есть свидетельства наличия лесов, поскольку он окрашен в зеленый цвет, а в двух местах написано «Довольно густые смешанные джунгли» и «Открытые смешанные джунгли», поэтому лесное хозяйство является основным занятием. людей в этом регионе.
(i) В чем разница между наклоном в квадрате сетки 2115 и квадратом 1811 года. Обоснуйте свой ответ.
Ответ: В квадрате сетки 2115 наклон постепенный и пологий, поскольку контурные линии, проведенные в этой области, находятся далеко друг от друга, тогда как, как и в 1811 году, контурные линии нарисованы очень близко друг к другу, что означает, что он имеет крутые уклоны. .
(j) Для чего:
(i) линия огня в квадрате сетки 2316.
(ii) трубопровод в квадрате сетки 2209?
Ответ: (i) Линия пожара — это вырубка, сделанная в лесу для предотвращения распространения лесного пожара, как показано в квадрате сетки 2316.
(ii) Трубопровод в квадрате сетки 2209, забирает воду из многолетнего резервуара Кодра, показанного в южной части по сетке 220090 в главный город «Абу».
(k) Что означает следующее?
(i) Красный квадрат в квадрате сетки 2514.
(ii) 4r в квадрате сетки 1612,
Ответ: (i) Красный квадрат в квадрате сетки 2514 показывает «временную хижину».
(ii) 4r в квадрате сетки. 1612 показывает, что относительная высота насыпи на сухом или сезонном резервуаре составляет 4 метра.
(l) Абу — популярный курорт. Назовите любые три особенности, которые можно увидеть в отрывке из карты, которые привлекают отдыхающих в Абу.
Ответ: (i) Гора Абу расположена на большой высоте, что обеспечивает приятный климат, некоторые из вершин имеют высоту 1409 и 1327 метров.
(ii) Есть много храмов, которые стоит увидеть, например, Анджини Деви Ка Мандир, Адхор Деви Ка Мандир, Храм Дилвара и т.д. для альпинизма, точки заката, жилища в пещерах и т. д.

Вопрос 6: Изучите выдержку из Карты Обзора Индии № 45D / 10 и ответьте на следующие вопросы:
(a) Дайте ссылку на шестизначную сетку:
(i) Храм в деревне Дхавли (ii) ∆ 480
Ответ: (i) 111073 (ii) 138045.
(b) Что означает следующее?
(i) 6r в квадрате сетки 1903
(ii) Слово Brackish в квадрате сетки 1403.
Ответ: (i) Относительная глубина насыпи составляет 6 метров.
(ii) Это соленая вода.
(c) (i) В квадрате сетки 1909 несколько контуров сливаются в одной точке. Что это собой представляет?
(ii) Укажите значение красных пунктирных линий на фрагменте карты.
Ответ: (i) Утес (ii) Тропинка.
(d) Назовите тип дождя, выпавший в регионе, показанном на фрагменте карты. Укажите причину в поддержку своего ответа.
Ответ: Осадки сезонные, потому что большинство ручьев сухие и не имеют воды.
(e) Чем рисунок дренажа в квадрате сетки 1606 отличается от рисунка дренажа в квадрате сетки 1708?
Ответ: 1606-радиальный
1708-решетчатый.
(f) (i) Какова схема заселения в квадрате 1904 года?
(ii) Назовите населенный пункт, в котором есть почтовое отделение.
Ответ: (i) Рассеянный (ii) Даттани
(g) Назовите две формы рельефа, представленные узором контуров в квадрате сетки 1608.
Ответ: Пасс и шпор.
(h) Рассчитайте расстояние до земли в километрах вдоль дороги с металлическим покрытием между дамбой в 1502 и дистанционным камнем, обозначенным цифрой «14» в квадрате сетки 1203.
Ответ: 3,5 км
(i) Укажите две причины об отсутствии человеческого жилья в северо-восточном районе извлечения карты.
Ответ: Район гористый и не имеет транспортной инфраструктуры.
(j) (i) Укажите разницу в высоте между самой высокой высотой пятна в отрывке карты и высотой контура в квадрате сетки 1006.
(ii) Какое направление по компасу у Патлавы ка Голиа (590.) от Качоли Дунга (443.)?
Ответ: (i) 1023 — 260 = 763 м (ii) северо-запад.

Вопрос 7: Изучите выдержку из карты Обзора Индии № 45D / 10 и ответьте на следующие вопросы:
(a) Дайте ссылку на шестизначную сетку:
(i) Триангулированная высота 217
( б) Облицованный колодец у Чехлы.
Ответ: Шесть рис. Ссылка на сетку:
(i) D 217 — 2 (ii) обсаженная скважина —
4.
(b) (i) Назовите левый приток главной реки.
(ii) Укажите направление, в котором течет этот левый приток.
Ответ: (i) Левобережный приток — Баларам-нади.
(ii) Он течет с юго-востока на северо-запад.
(c) (i) Упомяните особенность, связанную с потоками в квадрате сетки 9879.
(ii) Назовите типы дренажной схемы, обнаруженные в квадрате сетки 9382.
Ответ: (i) Характеристика, связанная с ручьем сломана земля.
(ii) Дендритный узор.
(d) Дайте четыре ссылки на сетку для каждого из следующего:
(i) Открытый скраб
(ii) Bantawada.
Ответ: Четыре рис. ссылка на сетку
(i) Открытый кустарник — 9573
(ii) Бантавада — 9978
(e) Назовите два рельефа, которые можно увидеть в квадрате сетки 9782 и 9574.
Ответ: Два рельефных элемента:
9782 — Откос
9574-Хребет / водораздел.
(f) Почему вы обнаруживаете ограниченное выращивание в отрывке карты?
Ответ: Назовите две причины для вашего ответа.
На карте ограничено возделывание, потому что регион
(i) — это Сэнди с Сандунесом.
(ii) Наличие пересеченного грунта, который является пустошью.
(g) Какое направление по компасу на Антроли (9576) и Чехлу (9281) от Сангла?
Ответ: Сангла в Антроли — Северо-восток.
Сангла в Чехлу — Север.
(h) Какой тип осадков выпадает в регионе, показанном на фрагменте карты? Обоснуйте свой ответ одной причиной.
Ответ: В этом регионе мало сезонных осадков.
Reason-Реки и ручьи сезонные.
(i) Вычислите расстояние в километрах по дороге телеги между Читрасани (999747) и Пироджпурой (978753).
Ответ: Расстояние в см = 5,2
Расстояние в км = 5,2 / 2 (масштаб от 2 см до 1 км)
Расстояние в км = 2,6 км.
(j) (i) Какое географическое название вы бы дали общей структуре поселений в регионе, показанном на карте?
(ii) Назовите один регион, показанный на карте, который непригоден для выращивания сельскохозяйственных культур.
Ответ: (i) Общая схема расселения — ядерная.
(ii) Один из регионов, непригодных для выращивания сельскохозяйственных культур, — это бесплодная земля / область белого цвета.

Вопрос 8: Изучите выдержку из Карты Обзора Индии № 45D / 10 и ответьте на следующие вопросы:
(a) Дайте ссылку на шестизначную сетку:
(i) Триангулированная высота 307.
(ii) Высота пятна 196.
Ответ: (i) 859843 (ii) 860788.
(b) Дайте ссылку на четырехзначную сетку для каждого из следующего:
(i) Место слияния реки Сипу и Махадевийо Нала.
(ii) Листовой камень.
Ответ: (i) 8189 (ii) 8188.
(c) Измерьте кратчайшее расстояние в километрах между храмом в квадрате сетки 8192 и многолетним колодцем в Бхакодаре 8188.
Ответ: 8 см = 4 км.
(d) Что означает следующее?
(i) Черные изогнутые линии в 7788.
(ii) Синяя линия в русле реки Сипу.
Ответ: (i) пересеченный грунт (ii) многолетний ручей.
(e) (i) Какова общая структура населенных пунктов в регионе, показанном на карте?
(ii) Объясните причину своего ответа.
Ответ: (i) Nucleated
(ii) Есть несколько путей, соединяющих поселения. Люди могут заняться сельским хозяйством.
(f) Какая основная форма орошения показана на отрывке карты? Зачем это нужно?
Ответ: Многолетние колодцы, потому что реки сезонные.
(g) (i) Какой вид транспорта является основным в этом регионе?
(ii) Приведите карту, подтверждающую ваш ответ.
Ответ: (i) Тележки.
(ii) одиночные красные линии, показанные на карте.
(h) (i) Какое направление по компасу на Дантивада, 8582, от Бхадли Кота, 7886?
(ii) Каково общее направление течения реки Арадо. N?
Ответ: (i) юго-восток (ii) с севера на юг.
(i) (i) Назовите тип схемы дренажа в квадрате сетки 8584.
(ii) Что вы подразумеваете под 25r в квадрате сетки 8286?
Ответ: (i) дендритный
(ii) относительная глубина многолетнего колодца 25 метров.
(j) (i) Что означает «Р.Ф.»?
(ii) Что такое R.F. показано на этой карте отрывок?
Ответ: (i) Репрезентативная дробь: это отношение горизонтального расстояния между двумя точками на карте к расстоянию до соответствующих точек на земле.
(ii) 1: 50000.

Вопрос 9: Ответьте на следующие вопросы в опросном листе № 45 D / 10.Восток с 00 по 10; Северное положение с 17 по 28. (Карта G)
(a) Дайте ссылки на четырехзначную сетку следующих элементов:
(i) Дхад Талао (ii) Листовая порода на северо-востоке
(iii) Обнажение скальной породы на севере (iv) ) Каменные отходы на юго-востоке
(v) Открытый кустарник к югу от Дхад Талао
Ответ: (i) 0722 (ii) 0826 (iii) 0526
(iv) 0918 (v) 0721
(b) Приведите следующие номера шестизначной сетки:
(i) Полицейский Чоуки в деревне Дантрал
(ii) Храм в деревне Бхамра
(iii) Колодец с водой в поселении Девка
(iv) Высота пятна 310 возле деревни Джолпур
( v) Резервуар возле поселения Индария
Ответ: (i) 032238 (ii) 050210 (iii) 019187
(iv) 075188 (v) 085259
(c) Дайте направление следующего:
(i) Деревня Дантрал (0324) из Дханна (0723)
(ii) Деревня Бхамра (0521) из Данна (0723)
(iii) Деревня Джолпур (0718) из Малавы (0519)
(iv) Деревня Варка (0716) из Биканваса ( 0316)
(v) Деревня Рампура (0617) из Дханна (0723)
Ответ: (i) Северо-запад (ii) Юго-запад (iii) Восток
(iv) Восток (v) Юг.
(d) Дайте следующие значения:
(i) 15r синим цветом в квадрате сетки 0224.
(ii) 300 коричневым цветом в квадрате сетки 0426.
(iii) Черная линия в самой северной части карты .
(iv) Красная линия в квадрате сетки 0819.
(v) Черные точки в квадрате сетки 0617.
Ответ: (i) 15r означает, что относительная глубина скважины с многолетней облицовкой составляет 15 м.
(ii) Контурная линия 300 м.
(iii) 24 ° 45 ′ — самая северная широта, показанная на фрагменте карты.
(iv) Тележка для тележек.
(v) Естественный сезонный резервуар с насыпью.
(e) Приведите следующие модели поселений:
(i) Деревня Дханн (ii) В квадрате сетки 0220
Ответ: (i) Общая структура деревни Дханн представляет собой кластерный тип поселения.
(ii) Общая картина — рассредоточенные или рассредоточенные поселения.
(f) Измерьте следующие расстояния:
(i) Прямое расстояние в км. от Индарии (0826) до Джолпура (0718)
(ii) Косвенное расстояние в км.вдоль пути телеги от Дантрала (0324) до Дханна (0723)
Ответ: (i) Прямое расстояние между Индарией и Джолпуром составляет 14 см.
1 см., На карте равно 500 м. на земле
14 см., равно 500 × 14 = 7000 м. или 7 км.
(ii) Расстояние вдоль тележек между Дантралом и Дханном составляет 8 см.
1 см. на карте равно 500 м.
8 см. равно 500 × 8 = 4000 м. или 4 км.
(g) Приведите следующие схемы дренажа:
(i) 0827 (ii) В квадрате сетки 0226
Ответ: (i) Схема дренажа — решетчатая.
(ii) Рисунок дренажа дендритного типа напоминает дерево.
(h) Назовите три искусственных объекта, обнаруженных в квадрате сетки 0419, и нарисуйте их символы.
Ответ: Храм, телеги, многолетний колодец.
(i) Приведите два доказательства того, что в регионе выпадают сезонные осадки.
Ответ: Наличие пересеченного грунта и сезонных рек.
(j) Объясните природу реки.
Ответ: Основная река Сукли Нади, текущая в южном направлении, является сезонной рекой, водотоки многочисленны и хорошо очерчены, поскольку могут прорезать русла в мягкой почве.Они являются периодическими и зависят от муссонных дождей для водоснабжения.
(к) Разъяснить рельеф местности.
Ответ: Район преимущественно равнинный. Земля выше на северо-востоке и полого спускается к юго-западу. Есть несколько обнажений скал, поднимающихся на высоту около 300 м. Большая часть равнины песчаная, и наличие песчинок указывает на пустынные условия.
(l) Каков общий уклон участка? Обоснуйте свой ответ.
Ответ: Земля имеет уклон с северо-востока на юг.Наивысшая точка высотой 311 видна на северо-востоке, часть карты и юго-западная часть карты показывает высоту точки 263. Течение рек с северо-востока. к S.W. также является показателем уклона земли.
(м) Какое основное занятие у людей? Обоснуйте свой ответ.
Ответ: Основное занятие народа — сельское хозяйство. Это обозначено на карте желтой размывкой, которая покрывает почти весь фрагмент карты. Наличие большого количества скважин с многолетней футеровкой также подтверждает этот вывод.
(n) Какой режим полива? Обоснуйте свой ответ.
Ответ: Колодезное орошение — самый распространенный способ орошения. Это связано с наличием большого количества многолетних колодцев, которые снабжены водой круглый год.
(o) Какой вид транспорта? Обоснуйте свой ответ.
Ответ: Вид транспорта — тележный. В основном это связано с тем, что это сельская местность, и большинство этих тележек соединяют деревни в этом регионе.

Вопрос 10: Изучите выдержку из Карты Обзора Индии № 45D / 10 и ответьте на следующие вопросы: (Карта G)
(a) Дайте ссылку на четырехзначную сетку:
(i) Дадарла (ii) Бхамра
Ответ: Ссылка на четырехзначную сетку: (i) Дадарла -1020 (ii) Бхамра — 0420
(b) Какое самое крупное поселение показано на фрагменте карты? Какое средство связи доступно только этому поселению?
Ответ: Дантрай — крупнейшее поселение, показанное на фрагменте карты.Почтовое отделение (ПО) — единственное средство связи, доступное исключительно для этого населенного пункта.
(c) Что означает РФ? Укажите РФ предоставленной вам карты.
Ответ: RF — представительная фракция.

RF карты составляет 1: 50 000, т.е. 1 юнит на карте равен 50000 юнитам на земле.
(d) Какова общая площадь фактической земли, представленной на фрагменте карты? (Покажите с помощью простого расчета)
Ответ:

(e) Что подразумевается под «интервалом изолиний»? Укажите интервал между контурами предоставленного вам топографического листа.
Ответ: Интервал изолиний означает разницу между двумя последовательными горизонтальными линиями. Межосевой интервал данного топографа составляет 20 метров.
(f) Укажите наибольшую и наименьшую высоту точек, найденных на отрывке карты. Также укажите, в метрах они или в футах.
Ответ: Наивысшая высота точки 339 метров, а самая низкая — 263 метра.
(g) Укажите в шестизначной сетке:
(i) Высота пятна 339 (ii) Постоянная хижина, расположенная недалеко от Дхад Талао.
Ответ: (i) Высота пятна 339 — 042276
(ii) Постоянная хижина, расположенная недалеко от Дхад Талао — 077225
(h) Что, по вашему мнению, является основным (i) занятием и (ii) религией человека? люди вышеупомянутой области?
Ответ: (i) Род занятий: Основным занятием жителей этой местности является сельское хозяйство. Желтым цветом на карте обозначены земли сельскохозяйственного назначения. Белыми пятнами на карте показаны невозделанные земли.
(ii) Религия: Символ храма на всей карте доказывает, что в основном люди принадлежат к религии индуистов.
(i) Назовите один «естественный» и один «искусственный» источник орошения, используемые в этом регионе.
Ответ: Естественный источник орошения — Сукли Нади. Сплошной синий круг и полый синий кружок указывают на наличие многолетних колодцев с облицовкой и без нее, которые являются искусственным источником орошения.
(j) Назовите общий вид транспорта, используемый в этом регионе. Какая особенность этого режима упоминается в отрывке карты?
Ответ: Общий вид транспорта — бык или телега.Одна красная линия, показанная на карте, указывает путь для тележек. Это путь шире, чем вьючная тропа, сделанная волом или телегой.

Вопрос 11: Ответьте на следующие вопросы в опросном листе № 45 D / 7.
(a) Дайте шестизначные ссылки на сетку следующих значений:
(i) ∆ 364 (ii) Храм в Рампуре
Ответ: (i) Шестизначная ссылка на сетку ∆ 364 — 957744.
( ii) Шестизначная сетка храма в Рампуре — 954806.
(b) Назовите любые две детали на отрывке карты, которые показывают, что в этом регионе есть сезонные осадки.
Ответ: Две особенности, которые показывают, что в регионе есть сезонные осадки:
(i) Сухая река с водным руслом, показанная в северной части фрагмента карты.
(ii) Сухие резервуары показаны в южной части фрагмента карты.
Две приведенные выше детали показывают, что в сезон дождей в них есть вода.
(c) Рассчитайте расстояние в километрах по рельсовому пути, соединяющему Джуволь (

    6) и Амиваду (
    7).
    Ответ: Расстояние между Джуволем и Амивадой = 5 см = 2 1/2 км (или 2 км и 500 метров) = 2 км 500 метров (согласно масштабу карты).
    (d) Какая основная форма орошения показана на отрывке карты? Приведите доказательства, подтверждающие ваш ответ.
    Ответ: (i) Основной формой орошения, показанной в отрывке карты, являются колодцы с многолетней облицовкой.
    (ii) Имеется четкое свидетельство этого типа орошения, показанное в северо-западном углу карты, извлеченной вокруг деревень Джувол и Чехла, которые расположены на пахотных землях, а найденные в этой области многолетние колодцы являются единственный источник орошения.
    (e) Назовите одну причину, чтобы объяснить, почему потоки в квадрате сетки 9478 не соединяются с рекой. Найдите другой квадрат сетки в отрывке карты, который имеет похожие потоки.
    Ответ: Ручьи в квадрате сетки 9478 не впадают в главную реку, так как это «исчезающие» потоки, им не хватает воды, чтобы достичь главной реки, и они высыхают в песке. Подобные типы потоков встречаются и в квадрате сетки 9575.
    (f) Что означает следующее:
    (i) Черные пунктирные линии в 9575 (ii) Черные изогнутые линии в 9879.
    Ответ: (i) Это исчезающие потоки, которые обычно показаны черными пунктирными линиями.
    (ii) Черные изогнутые линии показывают пересеченную местность.
    (g) Идентифицируйте две формы рельефа, показанные контурами в квадрате сетки 9876.
    Ответ: Формами рельефа, показанными контурами в квадрате сетки 9876, являются:
    (i) Конический холм с высотой пятна 381.
    (ii ) Южная часть этого холма — «отрог».
    (h) (i) Каково общее направление Балрам Нади?
    (ii) С каким берегом главной реки впадает Балрам Нади?
    Ответ: (i) Балрам Нади течет с юго-востока на северо-запад.
    (ii) Балрам Нади впадает в главную реку на ее левом берегу.
    (i) Какое основное занятие люди, проживающие в районе, показанном на фрагменте карты? Назовите одну причину в поддержку своего ответа.
    Ответ: (i) Основным занятием людей, живущих в районе, показанном на фрагменте карты, является сельское хозяйство.
    (ii) «Большая часть территории имеет« желтую полосу », которая указывает на возделываемые земли.
    (j) Что подразумевается под масштабом карты? Каков масштаб предоставленного вам фрагмента карты?
    Ответ: (i) Масштаб определяется как соотношение между расстоянием между любыми двумя точками на карте и фактическим расстоянием до тех же точек на земле.
    (ii) Масштаб предоставленной выдержки карты составляет 2 см = 1 км. Это означает, что 2 см на карте равны 1 км по земле.

    Вопрос 12: Изучите выдержку из Картографического листа № 45D / 7 Обзора Индии и ответьте на следующие вопросы.
    (а) Что вы понимаете под Востоком и Севером?
    Ответ: Это линии сетки, отмеченные красным на топокартах. Вертикальные линии называются восточными, а горизонтальные — северными.
    (b) Укажите значение каждого из следующих условных символов, расположенных в квадрате сетки 1622.
    (i) 20r (ii) PO
    Ответ: (i) 20r означает, что относительная глубина уровня воды в колодец 20м.
    (ii) PO — это условный знак почтового отделения.
    (c) Каково общее направление течения Сипу Нади, указанное на фрагменте карты? Назовите одну причину в поддержку своего ответа.
    Ответ: с востока на юго-запад. Это видно по уменьшению значения высот пятен.
    (d) Дайте ссылку на шестизначную сетку:
    (i) Точка, где канал выходит из земляных работ, рынок плотин 10r.
    (ii) Высота пятна 1425 в юго-восточном углу отрывка карты.
    Ответ: (i) 223214 (ii) 236142
    (e) Андхра является более развитым поселением, чем Гулабгандж. Назовите две причины.
    Ответ: (i) Андхра соединен телефонной линией, а Глабгандж — нет.
    (ii) В Андхре есть почта и телеграф и бунгало «Дак».
    (f) Учет необходимости «линии огня» в районе джунглей данного фрагмента карты.
    Ответ: Fireline — это расчистка 5 м в лесу для предотвращения распространения лесных пожаров. В жаркое засушливое время года эти леса часто страдают от серьезных лесных пожаров.
    (g) (i) Какую основную цель решает большинство скважин в квадрате сетки 1621?
    (ii) Почему вода в некоторых из этих колодцев непригодна для питья?
    Ответ: (i) В солоноватом колодце 1621 года была соленая вода, которая использовалась для промывки извести.
    (ii) Из-за большого количества растворенных солей вода непригодна для питья.
    (з) Назовите два возможных занятия жителей поселка Токра в квадратах 2220 и 2221.
    Ответ: Возделывание и управление водохранилищем.
    (i) Какова общая длина канала в метрах?
    Ответ: 12,5 см / 2 = 6,25 км.
    (j) Укажите угловое расстояние храма в Питхапуре в 2022 году от вершины холма Бхуми Марги.
    Ответ: N14 0 W.

    Вопрос 13: Изучите выдержку из Карты Обзора Индии № 45 D / 7 (A — B) и ответьте на следующие вопросы:
    (a) (i) Что означает аббревиатура RF, обозначающая ?
    (ii) Что означает 20r (квадрат сетки 8295).
    (iii) Что означает V возле деревни Шергад в юго-восточном углу.
    Ответ: (i) Это обозначает репрезентативную фракцию.
    (ii) Относительная высота песчаного холма от уровня земли до вершины составляет 20 метров.
    (iii) Заброшенный форт.
    (b) (i) Какой тип рельефа вы видите на северо-западе карты (Западная Паншвала)?
    (ii) Почему земля, показанная белым цветом, не возделывается?
    Ответ: (i) Земля покрыта ровным песком, а также песчаными дюнами, образовавшимися в результате действия ветра, препятствующего на свободе.
    (ii) Это потому, что эта земля либо каменистая, либо сильно размыта по краям.
    (c) (i) Важность села Жегол в сетке кв.8590 на юге?
    (ii) Определить длину совместного течения рек Сипу, Варка и Ханва.
    Ответ: (i) Это важный пункт пересечения границы, потому что здесь с разных сторон сходятся до 5 (пяти) тележек.
    (ii) Это примерно 1,5 км. Нить используется для измерения длины по шкале.
    (d) Какой тип рельефа находится недалеко от Варканалы и Ханваналы возле Гангудры в центре местности?
    Ответ: Это щебень в виде оврагов.Причиной является речная эрозия, пронизывающая более мягкие песчаные отложения в сезон дождей.
    (e) Дайте четырехзначную сетку координат хорошо обозначенного меандра Варканалы и реки Сипу.
    Ответ: 8593 и 8697.
    (f) (i) Что обозначает черная линия, отмеченная 24 ° 30 ′ на севере.
    (ii) Что означает черная линия 72 ° 20 ′ с севера на юг в западной части карты?
    (iii) Что обозначают цифры 280, 300, 500 и т. Д., Написанные коричневым цветом на восточном поле карты.
    Ответ: (i) Черная линия восток-запад на севере обозначает широту в градусах и минутах.
    (ii) Черная линия с севера на юг указывает градусы и минуты долготы.
    (iii) Это окончания контурных линий в метрах, указывающих на возвышенность вдали.
    (g) (i) Укажите на карте две искусственно созданные объекты.
    (ii) Каково направление линий сетки реки Сипу 83 C 86?
    (iii) Какова привязка к сетке (четыре цифры) печи для обжига извести, расположенной недалеко от деревни Рампура на северо-западе?
    Ответ: (i) Деревенские хижины и колодцы.(ii) с северо-северо-востока на юго-запад. (iii) 8397.

    Вопрос 14: Изучите выдержку из Карты Карты Обзора Индии № 45 D / 7 (A и B) и ответьте на следующие вопросы.
    (a) Дайте ссылку на шестизначную сетку
    (i) ∆ 277 (→) (ii) Известковая печь возле Рампуры (→) (iii) Каменные отходы (→).
    Ответ: (i) ∆ 277 → 896988 (ii) Известковая печь возле Рампуры → 834973
    (iii) Каменные отходы 857962
    (b) (i) На что указывает синяя линия в реке Сипу?
    (ii) В какой деревне вы видите узловую функцию 7 Что вы имеете в виду?
    Ответ: (i) Синяя линия в реке Сипу указывает на постоянный водный канал.
    (ii) Jegol. Когда деревня соединяется со многими другими деревнями дорогой, это узловая функция дороги.
    (c) (i) Что обозначают черные линии вдоль потоков в квадрате сетки 9199?
    (ii) Как формируются эти черты?
    (iii) Какой тип области показан в квадрате сетки 8696?
    Ответ: (i) Эти черные линии указывают на излом.
    (ii) Они образуются из-за эрозии почвы во время наводнений.
    (iii) Песчаный район с песчаными дюнами.
    (d) Дайте ссылку на шестизначную сетку следующего:
    (i) Печь для обжига извести около деревни Паншвала.
    (ii) Колодец с водой у села Гонодара.
    (iii) Сухой резервуар возле деревни Мохуди Моти.
    Ответ: (i) 834974 (ii) 829921 (iii) 796919
    (e) (i) Какое направление течения реки Сипу?
    (ii) Укажите, почему он течет именно в этом направлении?
    Ответ: (i) Направление реки Сипу — с северо-востока на юго-запад.
    (ii) Он течет в этом направлении, потому что высота уменьшается с севера на юг.
    (f) Укажите три природных объекта в квадрате сетки 8998.
    Ответ: (i) Ручьи (ii) Джунгли (iii) Рассеченный холм
    (g) (i) Что означает термин открытый скраб, напечатанный на карте.
    (ii) Как вы можете получить представление об общем характере склона в этом районе?
    Ответ: (i) Грубая трава покрывает поверхность земли не очень близко.
    (ii) Течение рек указывает на северо-восточное и юго-западное направление склона на большей части территории.

    Вопрос 15: Изучите выдержку из Карты Обзора Индии № 45D / 7 и ответьте на следующие вопросы:
    (a) Дайте ссылку на шестизначную сетку:
    (i) ∆ 225 (ii ) Колодец возле Гангувады.
    Ответ: (i) 876950 (ii) 7
    (b) Назовите наиболее важный многолетний источник орошения, показанный на отрывке карты.
    Ответ: Хорошо с облицовкой.
    (c) (i) Что означает 3r в квадрате сетки 8895?
    (ii) Что вы подразумеваете под 20r в квадрате сетки 8994?
    Ответ: (i) Относительная высота берега ручья (ii) Относительная глубина колодца 20 м.
    (d) Укажите две точки, свидетельствующие о том, что в регионе на отрывке карты есть сезонные осадки.
    Ответ: Существует большое количество сухих потоков, открытых кустов, разбитой земли, сухих резервуаров и т. Д.
    (e) (i) Что обозначает синяя линия в Варканале?
    (ii) Каково общее направление Варканалы?
    Ответ: (i) Река с небольшим ручьем. (ii) с юго-востока на запад.
    (f) (i) Что означает 1: 50000, напечатанное под отрывком карты?
    (ii) Рассчитайте расстояние в км. Вдоль пути телеги между Джегалом в квадрате сетки 8590 и Одхавой в квадрате сетки 8892.
    Ответ: (i) 1 единица на карте равна 50000 единицам земля.
    (ii) Расстояние 3,6 км.
    (g) Какая часть карты более развита в сельском хозяйстве? Обоснуйте свой ответ.
    Ответ: Юго-западная часть. Потому что меньше скрабов и большое количество колодцев.
    (h) Дайте четырехзначную сетку привязки слияния рек Сипу и Варканалы.
    Ответ: 8392.
    (i) Назовите два типа растительности в данном отрывке карты.
    Ответ: Открытые смешанные джунгли, открытый кустарник и открытые джунгли.
    (j) Какова схема дренажа в квадрате сетки 8599?
    Ответ: Дендритный.
    (k) (i) Сколько типов треков показано в отрывке карты? Назови их.
    (ii) Какой вид транспорта обычно используется в южной части этой карты?
    Ответ: (i) Гусеница тележки и гусеница. (ii) Тележки.
    (л) Какова общая схема расселения? Почему?
    Ответ: Кластерные или ячеистые поселения из-за больших обрабатываемых земель и наличия воды.

    Дополнительные ресурсы

    ICSE География Класс 10 Банк вопросов Глава 2 Интерпретация топографических карт 2 — Решения ICSE

    ICSE География Класс 10 Банк вопросов Глава 2 Интерпретация топографических карт 2

    Вопросы, основанные на контурах

    I. Приведите термины, используемые в контурах для каждого из следующего:

    Вопрос 1.
    Число, которому предшествует точка.
    Ответ:
    Высота пятна

    Вопрос 2.
    Числовая разница между двумя горизонтальными линиями.
    Ответ:
    Вертикальный интервал или В.И.

    Вопрос 3.
    Число, которому предшествует треугольник.
    Ответ:
    Точка триангуляции.

    Вопрос 4.
    Разница между максимальной и минимальной высотой на участках с крутым уклоном.
    Ответ:
    RelativeHeight (‘r’)

    Вопрос 5.
    Точная высота некоторой точки, отмеченной над землей.
    Ответ:
    Высота пятна

    II. Опишите следующие термины:

    Вопросы 1.
    Интервал контура
    Ответ:
    Интервал контура —
    Интервал контура означает разницу высот между двумя последовательными контурами. На рис. один толстый контур показывает высоту 300 метров, а следующий тонкий контур показывает высоту 320 метров. Шаг изолиний — 20 метров.

    Вопрос 2.
    Интервал по вертикали
    Ответ:
    Интервал по вертикали (V.I.):
    Расстояние по вертикали между любыми двумя горизонтальными линиями называется интервалом по вертикали (V.I.).

    Вопрос 3.
    Эквивалент по горизонтали
    Ответ:
    Эквивалент по горизонтали (H.E.):
    Фактическое расстояние между двумя точками на двух горизонтальных линиях называется эквивалентом по горизонтали (H.E.).

    Вопрос 4.
    Градиент
    Ответ:
    Градиент:
    Отношение между вертикальной высотой и горизонтальным расстоянием склона земли, измеренного вдоль земли, называется Градиентом (G).

    III. Объясните, как бы вы различили следующие элементы на основе контуров:

    Вопрос 1.
    пологий и крутой склон
    Ответ:
    пологий и крутой склон:

    пологий склон

    крутой склон

    На таком склоне изменение высоты происходит постепенно. Обычно градиент 1:25 называется пологим уклоном. Контуры в таком склоне расставлены широко. Когда земля внезапно скатывается вверх или вниз, это называется крутым уклоном.
    На крутом склоне контуры нарисованы близко друг к другу.

    Вопрос 2.
    Scarp and Dip
    Ответ:
    Scarp and Dip:

    Dip

    Scarp

    (i) с пологим уклоном . У уступа узкая вершина с крутым уклоном.
    (ii) Контурные линии далеко разнесены. Контурные линии становятся ближе.
    (iii) Он имеет градиент примерно 1:12. Он имеет уклон 1: 3

    Вопрос 3.
    Хребет и водораздел
    Ответ:
    Хребет и водораздел:
    Хребет:
    Это длинная и узкая возвышенность, круто спускающаяся по бокам. Имеет ряд вершин. Это хребет, который отделяет притоки, впадающие в одну сторону, от притоков, впадающих в другую.
    Контуры, обозначающие гребень, имеют удлиненную форму и расположены близко друг к другу.
    Что-то там встречаются вершины и переходы через хребет.

    Водораздел:
    Линия, разделяющая два соседних водосборных бассейна, известна как водораздел. Он напоминает по форме хребет, по обеим сторонам которого текут потоки. По обе стороны водораздела или водораздела склоны должны быть противоположными и нисходящими. На той стороне, на которой гребень более крутой, контуры нарисованы близко друг к другу, тогда как на стороне, на которой гребень более пологий, контуры нарисованы далеко друг от друга.

    IV. Вопросы с краткими ответами

    Вопрос 1.
    Что такое «Контур»?
    Ответ:
    Воображаемые линии, проведенные на карте для соединения мест, имеющих одинаковую высоту над уровнем моря.

    Вопрос 2.
    Укажите разницу в двух точках между реперной отметкой и высотой точки.
    Ответ:
    Реперы:
    Отметки, начертанные на камне или показанные на здании, чтобы подтвердить точную высоту, определяемую в ходе обследований. Где as:
    Высота пятна отображается только маленькой точкой и высотой в метрах e.грамм. 150 м.

    Вопрос 3.
    Что вы понимаете под В.И. и он.?
    Ответ:
    Вертикальный интервал (V.I.):
    Вертикальное расстояние между любыми двумя горизонтальными линиями называется вертикальным интервалом (V.I.).
    Горизонтальный эквивалент (H.E.):
    Фактическое расстояние между двумя точками на двух горизонтальных линиях называется горизонтальным эквивалентом (H.E.).

    Вопрос 4.
    Какие преимущества у контуров перед затенением холмов и штриховкой?
    Ответ:
    Затенение и штриховка холмов только создают впечатление склона, но не указывают высоту земли над уровнем моря, тогда как контуры показывают как впечатление склона, так и высоту земли над уровнем моря.

    Вопрос 5.
    Почему два контура никогда не пересекаются?
    Ответ:
    Эти два контура никогда не пересекаются друг с другом из-за высоты местности, а также из-за ее рельефа. Хотя это может случиться только в случае очень крутого обрыва, что является редкостью.

    V. Структурированные вопросы

    Вопрос 1.
    Что подразумевается под облегчением? Назовите три метода, которые используются для отображения рельефа Земли.
    Ответ:
    Рельеф означает фактическую конфигурацию местности, которая включает высоту и уклон.Рельефные элементы земной поверхности включают горы, долины, холмы, равнины и плато. Все эти характеристики имеют три измерения, а именно длину, ширину и высоту. Но карта, на которой они показаны, представляет только два измерения, то есть длину и ширину.
    Чтобы показать особенности рельефа земли. Основные используемые методы — штриховка, штриховка холмов, линии формы и контуры.

    Hachures —
    Это короткие отдельные линии, которые следуют по направлению склона.Фактически, штриховки изображают направление, в котором вода стекала бы по склону по поверхности, если бы она текла свободно.

    Затенение холмов:
    При использовании метода затенения холмов рельеф местности отображается с помощью эффекта света и тени ’i. е., он основан на предположении, что когда источник света, такой как большая лампа, используется для освещения модели рельефа рассматриваемой области, получаются тени форм рельефа.

    Форм-линии:
    Форм-линии — это прерывистые контурные линии, соединяющие места с одинаковой высотой над уровнем моря.Они представляют собой лишь общий рисунок местности и не дают представления об определенных высотах. К этому методу прибегают, когда рельеф местности сложный и точные данные недоступны.

    Вопрос 2.
    Что обозначают контурные линии? Как изолинии показывают разные уклоны земли?
    Ответ:
    Контурные линии — это воображаемые линии, соединяющие места, имеющие одинаковую высоту над средним уровнем моря.
    Контурные линии показаны коричневым цветом двумя способами:
    (i) толстые коричневые линии; и
    (ii) тонкие коричневые линии.

    Контурные линии показывают:
    (a) Высота участка:
    Высота контура обозначается цифрами, вставленными в разрыв линии.

    (b) Крутизна его склона:
    Когда контуры очень близки, они представляют крутые склоны. Когда они удалены друг от друга, они представляют собой постепенно увеличивающиеся склоны. Отсутствие контурных линий указывает на то, что местность плоская, т. Е. Невысокая.

    (c) Форма земли на разной высоте:
    Контурные линии указывают форму земли.Например, почти круглые контуры, значение которых уменьшается внутри, представляют собой озеро.

    Вопросы, основанные на масштабах и направлениях

    I. Вопросы с кратким ответом:

    Вопрос 1.
    Как направление показано на карте?
    Ответ:
    Направление показано на карте системой углового пеленга, которая представляет собой горизонтальный угол между линией север-юг и линией, соединяющей положение наблюдателя и объекта по часовой стрелке.

    Вопрос 2.
    Что вы имеете в виду под Р.Ф.?
    Ответ:
    Отношение между единичным расстоянием на карте и соответствующим единичным расстоянием на земле называется репрезентативной долей (R.F.).

    Вопрос 3.
    Каковы преимущества Р.Ф.?
    Ответ:
    Преимущество репрезентативной дроби в том, что может быть принята любая единица расстояния, и она может применяться повсеместно.

    Вопрос 4.
    Объясните Магнитное склонение с помощью диаграммы.
    Ответ:
    Угловое расстояние между истинным севером и магнитным севером называется магнитным склонением.На большинстве крупномасштабных карт он представлен двумя стрелками, отходящими от общей точки: одна указывает на истинный север, а другая — на магнитный север, а угол между ними — это магнитное склонение.

    Вопрос 5.
    Чем отличаются «истинный север» и «север по сетке»?
    Ответ:
    Полукруглые линии долготы считаются прямыми, когда они представлены на картах. Таким образом представленный Север — это Север Сетки. Он отличается от Истинного Севера сферической формой земли.

    Вопрос 6.
    Преобразовать следующую числовую шкалу (RF) в шкалу отчетов:
    (i) 1: 1,000
    (ii) 1: 50,000
    (iii) 1: 5,00,000
    Ответ:
    (i) 1: 1,000
    Числовая шкала (RF) = Расстояние на карте
    Расстояние на земле = 1/1000 = от 1 см до 1000 см
    Так как, 1 км = 100 000 см Или 1 × 100 см = 1000 × 100 см 100 см до 1 км
    Таким образом, масштаб выписки составляет от 100 см до 1 км.

    (ii) 1: 50 000
    Числовая шкала (RF) = расстояние на карте
    Расстояние на земле = 1/50 000 = от 1 см до 50 000 см
    Так как, 1 км = 100 000 см
    Или
    1 × 2 см = 50,000 × 2 см
    2 см = 1,00,000 см
    От 2 см до 1 км
    Таким образом, масштаб утверждения составляет от 2 см до 1 км.

    (iii) 1: 5,00,000
    Числовая шкала (RF) = расстояние на карте
    Расстояние на земле = 1 / 5,00,000
    = от 1 см до 5,00,000 см
    Поскольку, 1 км = 1,00,000 см
    От 1 см до 5 км
    Таким образом, масштаб выписки составляет от 1 см до 5 км.

    Вопрос 7.
    Преобразовать следующее в числовые шкалы (R.F.)?
    (a) 10 см до километра
    (b) 20 километров до сантиметра.
    (c) от 2 см до 5 км
    (d) от 6 дюймов до 1 мили
    (e) от 1 дюйма до 2 1/2 миль
    (f) от 7 см до 6.3 км
    Ответ:
    (a) от 10 см до 1 км
    As, 1 км = 1,00 000 см
    Поскольку Р.Ф. = Расстояние на карте (в см)
    Расстояние на земле (в см)
    R.F. = 10 / 1,00,000 = 1 / 10,000
    Таким образом, числовая шкала (RF) составляет 1: 10,000

    (b) (от 1 см до 20 километров)
    As, 1 км = 1,00,000 см
    20 км = 1 , 00000 × 20 см
    = 20000000 см
    Т.к., РФ = Расстояние на карте (в см)
    Расстояние на земле (в см)
    R.F. = 1 / 20,00,000
    Таким образом, числовая шкала (R.F.) составляет 1: 20,00,000.

    (c) от 2 см до 5 км
    As, 1 км = 1,00,000 см
    5 км = 1,00,000 × 5 см
    = 5,00,000 см
    Поскольку R.F. = Расстояние на карте (в см)
    Расстояние на земле (в см)
    R.F. = 2 / 5,00,000 = 1 / 2,50,000
    Таким образом, числовая шкала (RF) составляет 1: 2,50,000

    (d) от 6 дюймов до 1 мили
    Мы знаем, что 1 миля = 63 360 дюймов
    Поскольку, РФ = Расстояние на карте (в дюймах)
    Расстояние до земли (в дюймах)
    R.F. = 6 / 63,360 = 1 / 10,560
    Таким образом, числовая шкала (R.F.) составляет 1: 10 560

    (e) от 1 дюйма до 2 1/2 мили
    As, 1 миля = 63 360 дюймов
    Мы знаем, что 2 1/2 мили = 63 360 × 2,5 дюйма = 1,58 400 дюймов
    Поскольку, РФ = Расстояние на карте (в дюймах)
    Расстояние до земли (в дюймах)
    R.F. = 1 / 1,58,400
    Таким образом, числовая шкала (RF) составляет 1: 1,58,400

    (f) от 7 см до 6,3 км
    As, 1 км = 1,00,000 см
    6,3 км = 1,00,000 × 6,3 см
    = 6,30 000 см Т.к.,
    РФ = Расстояние на карте (в см)
    Расстояние до земли (в см)
    R.F. = 7 / 6,30,000 = 1 / 90,000
    Таким образом, числовая шкала (R.F.) равна 1: 90,000

    II. Различают:

    Вопрос (a)
    Шкала утверждения и Графическая шкала.
    Ответ:
    (i) Масштаб утверждения:
    в этом методе масштаб указывается словами или мы делаем заявление о нем: например, 1 см. до 1 км от 1 дюйма до 5 миль.
    Значение в левой части обычно представляет расстояние на карте. Этот метод не подходит, так как нам необходимо знать единицы измерения в разных странах.

    (ii) Линейная или графическая шкала:
    В этом методе шкала представлена ​​прямой линией, разделенной на равные части (первичную и вторичную), чтобы показать, что эти отметки представляют на реальной поверхности. Первичные деления — это основные деления на шкале, градуированной слева направо. Второстепенные подразделения — это более мелкие подразделения, которые выпускаются справа налево в одном основном подразделении.

    Вопрос (b)
    Истинный север и магнитный север.
    Ответ:
    Истинный Север — это направление, указанное Полярной звездой, а Магнитный Север — это направление, указанное стрелкой Магнитного Компаса.

    III. Вопросы с развернутым ответом:

    Вопрос 1.
    Что касается масштаба, ответьте на следующие вопросы:
    (i) Назовите основные методы представления масштаба карты.
    (ii) Преобразуйте следующие утверждения в R.F.
    (а) 25 см по карте = 5 км по местности.
    (b) 2 1/2 дюйма на карте = 5 миль по земле
    (c) 7 см на карте = 63000 метров по земле
    (d) 5 см = 500 метров
    (e) 15 см = 6 км. (е) 10 см = 1 км.
    (г) 12 см = 72000 метров
    Ответ:
    Существует три основных метода представления масштаба.
    (i) Заявленный масштаб:
    в этом методе масштаб указывается словами или мы делаем заявление о нем: например,
    1 см. до 1 км
    1 дюйм до 5 миль.
    Значение в левой части обычно представляет расстояние на карте. Этот метод не подходит, так как нам необходимо знать единицы измерения в разных странах.

    (ii) Линейная или графическая шкала:
    В этом методе шкала представлена ​​прямой линией, разделенной на равные части (первичную и вторичную), чтобы показать, что эти отметки представляют на реальной поверхности.Первичные деления — это основные деления на шкале, градуированные слева направо. Второстепенные подразделения — это более мелкие подразделения, которые выпускаются справа налево в одном основном подразделении.

    (iii) Репрезентативная фракция:
    В этом методе масштаб представлен как отношение длины линии на карте к соответствующему действительному расстоянию на поверхности земли. Числитель всегда выражается как единица, т. Е. Одна единица расстояния на карте эквивалентна количеству единиц расстояния до земли.Эта фракция называется репрезентативной фракцией.

    (ii) (a) 25 см на карте = 5 км по земле
    As, 1 км = 1,00,000 см
    5 км = 1,00,000 x 5 см
    = 5,00,000 см
    Поскольку Р.Ф. = Расстояние на карте (в см)
    Расстояние на земле (в см)
    R.F. = 25 / 5,00,000 = 1 / 20,000
    Таким образом, R.F. составляет 1: 20,000

    (b) 2 1/2 дюйма на карте = 5 миль по земле
    As, 1 миля = 63 360 дюймов
    5 миль = 63 360 × 5 дюймов
    = 3,16 800 дюймов
    Так как R.F. = Расстояние на карте (в дюймах)
    Расстояние на земле (в дюймах)
    R.F. = 2,5 / 3,16,800 = 1 / 1,26,720
    Таким образом, R.F. составляет 1: 1,26,720

    (c) 7 см на карте = 63000 метров на земле
    As, 1 м = 100 см
    63000 м = 100 × 63000 см
    = 63,00000 см
    Так как Р.Ф. = Расстояние на карте (в дюймах)
    Расстояние до поверхности шины (в дюймах)
    R.F. = 7 / 63,00,000 = 1 / 9,00,000
    Таким образом, R.F. составляет 1: 9,00,000

    (г) 5 см = 500 метров
    As, 1 м = 100 см
    500 м = 100 × 500 см
    = 50,000 см
    Поскольку, R.F. = Расстояние на карте (в см)
    Расстояние на земле (в см)
    R.F. = 5 / 50,000 = 1 / 10,000
    Таким образом, R.F. составляет 1: 10,000

    (e) 15 см = 6 км
    As, 1 км = 1,00,000 см
    6 км = 1,00,000 × 6 см
    = 6,00,000 см
    Так как R.F. = Расстояние на карте (в см)
    Расстояние на земле (в см)
    R.F. = 15 / 6,00,00 = 1 / 40,000
    Таким образом, R.F. составляет 1: 40,000

    (f) 10 см = 1 км
    As, 1 км = 1,00,000 см
    Так как R.F. = Расстояние на карте (в см)
    Расстояние до земли (в см)
    R.F. = 10 / 1,00,000 = 1 / 10,000
    Таким образом, R.F. составляет 1: 10,000

    (г) 12 см = 72,000 метров
    As, lm = 100 см
    72,000 м = 100 × 72,000 см
    = 72,00,000 см
    Поскольку, R.F. = Расстояние на карте (в см)
    Расстояние на земле (в см)
    R.F. = 12 / 72,00,000 = 1 / 6,00,000
    Таким образом, R.F. составляет 1: 6,00,000

    Вопрос 2.
    Преобразуйте следующие репрезентативные дроби в утверждения.
    (i) 1: 63360 (для отображения миль).
    (ii) 1: 1000000 (чтобы показать километры).
    (iii) 1: 100 (для отображения метров).
    (iv) 1:10 (чтобы показать метры).
    (v) 1: 200 000 (показать километры).
    (vi) 1:50 (чтобы показать метры и сантиметры).
    Ответ:
    (i) 1: 63360 (показать мили)
    R.F. 1: 63360 означает, что один дюйм на карте соответствует 63360 дюймам на земле;
    , поскольку 63360 дюймов равны одной миле,
    Итак, 1 дюйм = 1 миля
    Утверждение: 1 дюйм = 1 миля.

    (ii) 1: 1,00,000 (для отображения километров)
    Числовая шкала (RF) = расстояние на карте
    Расстояние на земле
    = 1 / 10,00,000
    = от 1 см до 10,00,000 см
    Поскольку, 1 км = 100 000 см
    Или 10 км = 10 000 000 см
    От 1 см до 10 км
    Таким образом, масштаб утверждения составляет от 1 см до 10 км.

    (iii) 1: 100 (для отображения метров)
    Числовая шкала (RF) = расстояние на карте
    Расстояние на земле
    = 1/100
    = от 1 см до 100 см
    Поскольку, 1 м = 100 см
    От 1 см до 1 м
    Таким образом, масштаб выписки составляет от 1 см до 1 м.

    (iv) 1:10 (для отображения метров)
    Числовая шкала (RF) = Расстояние на карте
    Расстояние на земле
    = 1/10
    = от 1 см до 100 см
    Поскольку, 1 м = 1,0000 см
    Или 10 м = 1000 м
    От 1 см до 10 м
    Таким образом, масштаб выписки составляет от 1 см до 10 м.

    (v) 1: 2,00,000 (для отображения километров)
    Числовая шкала (RF) = Расстояние на карте
    Расстояние на земле
    = 1 / 2,00,000
    = от 1 см до 2,00,000 см
    Поскольку, 1 км = 1,00 000 см
    или
    2 км = 2,00 000 см
    1 см до 2 км
    Таким образом, масштаб утверждения составляет от 1 см до 2 км.

    (vi) 1:50 (для отображения метров и сантиметров)
    Репрезентативная фракция (RF) = расстояние на карте
    Расстояние на земле
    = 1/50
    = от 1 см до 50 см
    = от 2 см до 100 см
    Поскольку, 1 м = 100 см
    2 см до 1 м
    Или
    1 см = 50 см
    Таким образом, масштаб отчета составляет от 2 см до 1 м
    Или
    от 1 см до 50 см.

    Вопрос 3.
    Нарисуйте линейные шкалы и отметьте на них следующие расстояния.
    (i) 1 см = 1 км (9 км)
    (ii) 1 см = 100 км (1100) км
    (iii) 1: 5,00 000 (13 км)
    (iv) 1: 20 000 (250 метров)
    Ответ:
    (i) 1 см = 1 км (показано 9 км)

    (ii) 1 см = 100 км (показано 1100) км

    (iii) 1: 5,00 000 (показано 13 км)

    RF = 1 / 5,00,000
    Или масштаб 1 см = 5 км (1 км = 1,0,0000 см)

    (iv) 1:20 000 (показано 250 метров)

    R.F. = 1 / 20,000
    Или Масштаб 1 см — 200 км

    Вопрос 4.
    Расстояние между вокзалом Нью-Дели и автобусной остановкой Safdarjang Enclave составляет 20 км. На карте Дели это показано линией 3,5 см. Нарисуйте линейный масштаб карты и рассчитайте R.F.
    Ответ:
    Масштаб 3,5 см = 20 км
    = 20 × 100000 см
    i. е. 35 см = 20000000
    = 2000000 см
    1 см = 20000000/35 = 57142,8
    1 см = 571423 см почти
    i. е. 1 см = 5,7 км
    Р.Ф. = 1: 571423

    Вопрос 5.
    На карте Индии картограф забыл нарисовать масштаб карты. Студента, который знает расстояние между Меррут и Дели (70 км), попросили заполнить шкалу. Как ученик начертит масштаб, если он измерит на карте расстояние между Дели и Меерутом, равным 5 см? Дайте процедуре начертить шкалу и узнать Р.Ф.
    Ответ:
    Расстояние на карте 5 см равно расстоянию 70 км по земле
    Итак, 5 см = 70 км 70
    1 см = \ (\ frac {70} {5} \) = 14 km
    i.е. 1 см 1400000 см
    Масштаб 1 см = 14 см
    Р.Ф. = 1: 1400000

    Вопрос 7.
    На приведенной выше карте изображены дома четырех друзей A, B, C и D. Найдите расстояния, используя главную дорогу и дорогу кучи, чтобы ответить на следующие вопросы.
    Ответ:
    Масштаб 1 см = 10 000 см
    1 см = \ (\ frac {10000} {100000} \) км \ (\ frac {1} {10} \) км = 0,1 км
    или 1 см = 100 метров

    (a) Какой ребенок пойдет в школу первым, если все A, B, C покинут свой дом одновременно?
    Ответ:
    C доберется до школы первым.

    (b) Как далеко D должен пройти, чтобы добраться до деревенского колодца?
    Ответ:
    Измерив ниткой, расстояние между D и деревенским колодцем составляет 12 см. Таким образом, если 1 см равен 100 метрам, то 12 см будут равны 1200 метрам. Таким образом, D должен пройти 1200 метров, чтобы добраться до колодца села.

    (c) Как далеко находится дом B от автобусной станции?
    Ответ:
    При помощи нитки расстояние между Автовокзалом и домом Б составляет 12 см.
    Таким образом, он находится в 1200 метрах от дома Б.

    (d) Какое расстояние придется пройти C, чтобы добраться до железнодорожного вокзала?
    Ответ:
    По нитке расстояние 9 см.
    Итак, расстояние между C и вокзалом 9 × 100 = 900 метров.

    (e) В каком направлении находится дом C от D.
    Ответ:
    Дом C находится в северо-западном направлении от D.

    Вопросы, основанные на чтении и интерпретации карты

    Краткие ответы на вопросы

    Вопрос 1.
    Укажите, что обозначают следующие символы относительно землепользования.
    (a) Резервуары и каналы
    (b) Открытый кустарник
    (c) Каменные отходы
    Ответ:
    (a) На нем изображены развитые сельскохозяйственные угодья с ирригационными сооружениями из-за наличия резервуаров и каналов.
    (b) На нем показаны пастбища или пастбища.
    (c) На нем показаны невозделываемые земли из-за непригодных каменистых земель.

    Вопрос 2.
    Где в селе расположены густонаселенные районы.
    Ответ:
    Густонаселенные районы расположены вблизи речных долин и у обочин дорог из-за наличия водных и транспортных средств.

    Вопрос 3.
    На что указывает структура дендритного дренажа о земле?
    Ответ:
    Дендритный дренаж, подобный ветвям дерева, обозначает мягкую почву, которая легко прорезана рекой и ее притоками.

    Вопрос 4.
    Как можно определить климат места по топографическому листу?
    Ответ:
    Топо-листы не предоставляют никакой прямой информации о климате места, например о количестве осадков, температуре и влажности.Но об этом можно судить по широте области, покрытой топографическим листом, и по типу растительности, которая там встречается. Например, климат места на 10 ° северной широты будет теплее, чем в месте на 3 5 ° северной широты.

    Вопрос 5.
    Какие факторы влияют на поселения в районе?
    Ответ:
    (a) Рельеф и климат местности:
    Поселения не встречаются на крутых склонах холмов, болотистых землях или заболоченных территориях. Люди стремятся селиться там, где земля плоская и пахотная.

    (b) Безопасность:
    Для обеспечения безопасности людей и собственности поселения обычно строятся на возвышенностях, чтобы избежать наводнений. Поселок может расти в месте, которое имеет стратегическое положение как центр обороны. Поселение могло быть результатом защиты людей и имущества от врагов и контроля доступа к важным проходам, гаваням и фабрикам.

    (c) Наличие транспорта и связи:
    Города и деревни развиваются там, где дороги, реки, железные дороги и т. Д., помощь в транспорте и торговле. Порты развиваются в устьях рек, которые также связаны автомобильными и железными дорогами.

    (d) Поселок, расположенный в центре:
    Поселок может развиваться, если он расположен в центре по отношению к другим поселениям (деревням). Он становится административным, а также торговым центром для других населенных пунктов вокруг него. С этой целью во многих небольших городах и деревнях Индии проводятся рынки и ярмарки (мелас).

    (e) Наличие воды и земли для возделывания сельскохозяйственных культур и других целей.

    (f) Потенциал горнодобывающей промышленности и промышленности.

    II Сопоставьте следующее:

    Доказательства

    Занятия

    1. Поселок возле главной дороги (a) Сельское хозяйство
    2. Лесные парки
    3. Побережье, озера (c) Выпас скота
    4. Желтое пятно (d) Рыболовство
    5.Медоуз (e) Развлечения
    6. Зеленое пятно (f) Торговля

    Ответ:

    Доказательства

    Оккупация

    главная дорога (f) Торговля
    2. Парки (e) Развлечения
    3. Побережье, озера (d) Рыболовство
    4. Желтое пятно (a) Сельское хозяйство
    5.Луга (c) Выпас скота
    6. Зеленая поляна (b) Лесное хозяйство

    III. На что указывают следующие диаграммы? Напишите короткие заметки о функциях, как показано на каждой диаграмме.

    Ответ:
    (i) Первая диаграмма показывает рассредоточенные или рассредоточенные поселения. Эти поселения находятся в районах с малонаселенностью, защищенными долинами или у подножия холмов. Здесь здания находятся далеко друг от друга. Они почти изолированы.Есть большие фермерские дома.

    (ii) Вторая диаграмма отображает заросшие поселения, плотные поселения по обеим сторонам дорог Кутча, в которых фермы и другие здания сгруппированы вместе, часто вокруг какой-либо центральной особенности, такой как церковь или храм, или полицейские чауки, почтовое отделение и т. Д. К факторам, способствующим образованию зарослей или компактных участков, относятся необходимость защиты, наличие водоснабжения в определенных местах и ​​развитие современных транспортных средств.

    Практика чтения карт на основе вопросов

    Вопрос 1.
    Изучите отрывок из Карты Обзора Индии № 45D / 7 и ответьте на следующие вопросы:

    (a) Дайте ссылку на шестизначную сетку:
    (i) Триангулированная высота 225;
    Ответ:
    876950

    (ii) Колодец с облицовкой недалеко от Гангувады.
    Ответ:
    7

    (б) Назовите наиболее важный многолетний источник орошения, показанный на отрывке карты.
    Ответ:
    Хорошо.

    (c) (i) Что означает 6r в квадрате сетки 8495?
    Ответ:
    Относительная высота дюны 6м.

    (ii) Что вы подразумеваете под 20r в квадрате сетки 8994?
    Ответ:
    Относительная глубина скважины 20 м.

    (d) Укажите две точки, свидетельствующие о том, что в регионе на отрывке карты есть сезонные осадки.
    Ответ:

    • Существует большое количество сухих водотоков, открытых кустарников, пересеченной земли, сухих резервуаров и т. Д.
    • Открытый кустарник указывает на разбросанные небольшие или низкорослые деревья, а его присутствие указывает на умеренное или малое количество осадков в сезон.

    (e) (i) Что обозначает синяя линия на Варке N?
    Ответ:
    Река с небольшим ручьем.

    (ii) Каково общее направление Северного Варки?
    Ответ:
    С юго-востока на запад.

    (f) (i) Что означает 1: 50 000, напечатанное под отрывком карты?
    Ответ:
    1 юнит на карте равен 50 000 юнитов на земле. Значение 1:50 000 — это соотношение между расстоянием, указанным на карте, и фактическим расстоянием на земле.

    (ii) Вычислите расстояние в км вдоль пути телеги между Джегалом в квадрате сетки 8590 и Одхавой в квадрате сетки 8892.
    Ответ:
    Расстояние 3,6 км.

    (g) Какая часть карты более развита в сельском хозяйстве? Обоснуйте свой ответ.
    Ответ:
    Юго-западная часть из-за меньшего количества кустов и большого количества колодцев.

    (h) (i) Дайте четырехзначную ссылку на сетку слияния реки Сипу и северной Варки.
    (ii) Назовите два типа растительности в данном отрывке карты.
    Ответ:
    8392.
    (i) Открытые смешанные джунгли, открытые кустарники и открытые джунгли.
    (ii) Дендритный.

    (i) Какова схема дренажа в квадрате сетки 8599?
    (j) (i) Сколько типов треков показано в отрывке карты? Назови их.
    Ответ:
    Отслеживание тележек и отслеживание упаковок.

    (ii) Какой вид транспорта обычно используется в южной части этой карты?
    Ответ:
    Тележки.

    (k) Какова общая схема расселения? Почему?
    Ответ:
    Сгруппированные или структурированные поселения из-за большой площади возделываемых земель и наличия воды, которые легко соединяются с другими деревнями через рельсы телеги.

    Вопрос 2.
    Изучите отрывок из Карты Обзора Индии № 45 / D 10 (карта C дана в конце книги) и ответьте на следующие вопросы:
    (a) Дайте значение условным обозначениям На карте нанесены следующие шестизначные обозначения сетки:
    (i) 197036
    Ответ:
    197036-6r — относительная высота насыпи.

    (ii) 1
    (2)
    Ответ:
    1
    -A Храм.

    (b) Дайте четырехзначную ссылку на сетку по одному яркому примеру каждого из следующих:
    (i) Трубопровод в северо-восточной части карты.
    Ответ:
    2209.
    (ii) Обнажение скал в юго-западной части карты.
    Ответ:
    1402.

    (c) Какое направление по компасу на Багери, 2002 г., от Хануманджи-ка-Мандир, 2208?
    Ответ:
    Юго-запад.

    (d) Назовите тип схемы дренажа в квадрате сетки 2209.
    Ответ:
    Решетчатый.

    (e) Как называются коричневые линии, покрывающие большую часть фрагмента карты? Что обозначают эти коричневые линии?
    Ответ:
    Контуры. Они указывают высоту / холмистую местность.

    (f) Что означают слова «падение с высоты 25 м» в квадрате сетки 2307?

    Ответ:
    Это означает, что водопад падает с 25 м или высота водопада 25 м над уровнем моря.

    (g) (i) Какой будет главный источник водоснабжения для горы Абу?
    Ответ:
    Водохранилище Нахи Талао или Кодра является основным источником водоснабжения горы Абу.

    (ii) Объясните причину своего ответа.
    Ответ:
    Многолетний источник воды, вокруг которого возникло поселение, обозначен синим цветом Талао.

    h) Нет дорог, соединяющих гору Абу с поселениями на юго-западе страны. Почему?
    Ответ:
    Нет дорог, соединяющих гору Абу с поселениями на юго-западе, потому что средняя часть региона представляет собой высокую холмистую местность, обозначенную коричневыми линиями, где строительство дорог невозможно.

    (i) Вычислите площадь земли, заключенную в пределах восточных 17 и 21 и северных направлений 05 и 09.
    Ответ:
    16 км 2 .

    (j) Комментарий к природе колодца в квадрате сетки 1906 г.
    Ответ:
    Вода из колодца солоноватая или соленая, что не подходит для питья и сельского хозяйства.

    (k) Назовите любые два типа лесных регионов, упомянутых в отрывке карты.
    Ответ:
    Довольно густые смешанные джунгли и Густые смешанные джунгли с бамбуком.

    (l) Укажите в километрах протяженность всего региона, изображенного на карте, с севера на юг. Покажи свою работу.
    Ответ:
    Длина 10 см. Масштаб карты 1 см = 0,5 км
    ∴ 10 см = 0.5 × 10 = 5 км

    (м) Какова схема поселения самого большого города на предоставленной вам карте?
    Ответ:
    Nucleated / Nodal.

    (n) Опишите природу дорог, соединяющих Гаутама Махариши Манди 2007 и Шри Вашист Ашрам 2207.
    Ответ:
    FootPath.

    Вопрос 3.
    Изучите выдержку из Карты Обзора Индии № 45 D / 7 и ответьте на следующие вопросы:
    (a) Дайте ссылку на шестизначную сетку:
    (i) ∆ 592
    Ответ:

    0

    (ii) Храм, где в феврале проводится ежегодная ярмарка.
    Ответ:

    3

    (b) (i) Укажите общее направление Арадо Н.
    Ответ:
    с севера на юг.

    (ii) Что обозначает синяя линия в AradoN?
    Ответ:
    Многолетний канал.

    (c) Рассчитайте расстояние в километрах по колее телеги между Велавасом (868895) и Ранолом (883888).
    Ответ:
    2,5 км.

    (d) (i) Что означает 7r в квадрате сетки 8689?
    Ответ:
    Относительная высота

    (ii) Укажите основной источник орошения в Дхариаваде (Фатепура).
    Ответ:
    Уэллс.

    (e) Какой тип осадков выпадает в регионе, показанном на фрагменте карты? Обоснуйте свой ответ одной причиной.
    Ответ:
    Сезонные осадки, так как рельсы телеги проходят прямо через реку, что указывает на то, что русло реки сухое и может быть пересечено телегами, запряженными волами, в сухой сезон.

    (f) Назовите и объясните термин, используемый для коричневых линий в отрывке карты.
    Ответ:
    Контурные линии. Это воображаемые линии, соединяющие два места на одинаковой высоте над уровнем моря.

    (g) Назовите любые два типа растительности, найденные на отрывке карты. Дайте ссылку на четырехзначную сетку для каждого.
    Ответ:

    • Открытые смешанные джунгли (9187).
    • Открытые джунгли (8600).

    (h) (i) Какой главный рельефный элемент виден в квадрате сетки 9284?
    Ответ:
    Коническая горка.

    (ii) Какой характерный элемент можно увидеть в русле реки по координатной сетке 9380?
    Ответ:
    Остров.

    (i) Укажите схему:
    (i) Населенные пункты в квадрате сетки 8989 и
    Ответ:
    (i) Линейное поселение.

    (ii) Дренаж в квадрате сетки 9185
    Ответ:
    Схема радиального дренажа.

    (j) Какой вывод можно сделать об общем занятии жителей региона, показанного в западной половине отрывка карты? Назовите одну причину, чтобы оправдать свой ответ.
    Ответ:
    Основным занятием является сельское хозяйство, так как большая часть его земель возделывается, что обозначено желтым цветом.

    Вопрос 4.
    Изучите выдержку из Карты Обзора Индии № 45 D / 7 и ответьте на следующие вопросы:
    (a) (i) Как называется коричневая линия в квадрате сетки 1516?
    Ответ:
    Коричневая линия в квадрате сетки 1516 называется контурной линией.

    (ii) Что означает цифра, написанная вдоль этой линии?
    Ответ:
    Цифра 280 означает, что высота этой контурной линии над уровнем моря составляет 280 метров.

    (b) Назовите два основных вида транспорта, которыми пользуются люди в районе, показанном на фрагменте карты.
    Ответ:
    Двумя основными видами транспорта являются железные дороги и тележки.

    (c) На каком берегу Сипу Нади находится деревня Гулабгандж (1920 г.)? Откуда в поселке водоснабжение круглый год?
    Ответ:
    Гулабгандж расположен на левом берегу Сипу Нади.Эта деревня круглый год снабжается водой из многолетних колодцев.

    (d) Что подразумевается под термином «Линия огня»? Учтите необходимость Линии Огня в районе джунглей данного фрагмента карты.
    Ответ:
    Линия огня — это вырубка, сделанная в лесу для предотвращения распространения лесных пожаров.
    Противопожарные линии необходимы в лесах, так как леса подвержены пожарам в жаркое засушливое время года. Эти линии огня предотвращают распространение огня на другие районы.

    (e) Каков характер канала, показанного на фрагменте карты? Измерьте в километрах общую длину канала.
    Ответ:
    Показанный канал является многолетним. Общая длина канала 5,7 км.

    (f) Чем рисунок дренажа в квадрате сетки 2118 отличается от рисунка в квадрате сетки 2114?
    Ответ:
    В сетке 2118 рисунок дренажа является радиальным, но в сетке 2114 рисунок дренажа является дендритным.

    (g) Какое из населенных пунктов на отрывке карты является самым важным? Назовите одну причину в поддержку своего ответа.
    Ответ:
    Анадра — самый важный населенный пункт на карте.Это потому, что в нем есть много объектов, таких как Police Chauki, которых нет в других поселениях.

    (h) (i) Укажите направление по компасу на Удварию (2423) от Гулабганджа (1920).
    Ответ:
    Направление Удварии от Гулабганджа — северо-восток.

    (ii) Назовите одно поразительное различие между этими двумя поселениями.

    Ответ:
    Удвария — это линейное / ленточное поселение, а Гулабгандж — компактное или ячеистое поселение.

    Вопрос 5.
    Изучите выдержку из Карты Обзора Индии №45D / 10 и ответьте на следующие вопросы:
    (a) Дайте шестизначную сетку координат: —
    (i) Храм в деревне Дхавли
    Ответ:
    Шестизначная сетка храма в деревне Дхавли — 111073.
    Примечание: [В Дхалви есть два храма (i) 111073 (ii) 107077 (через реку)]

    (ii) Δ 480
    Ответ:
    (ii) Для A 480 это 137046.

    (b) На что указывает следующее?
    (i) 4r в квадрате сетки 1903
    Ответ:
    Относительная высота сухого резервуара составляет 4 метра.

    (ii) Слово «солоноватая» в квадрате сетки 1403.
    Ответ:
    Соленая вода, не пригодная для употребления в пищу людьми или животными. Он также может быть непригодным для выращивания из-за того, что он соленый или солоноватый.

    (c) (i) В квадрате сетки 1909 несколько контуров сливаются в одной точке. Что это собой представляет?
    Ответ:
    Крутой обрыв или внезапное падение воды.

    (ii) Укажите значение красных пунктирных линий на фрагменте карты.
    Ответ:
    Это вьючная тропа, используемая вьючными животными.

    (d) Назовите тип дождя, выпавший в регионе, показанном на фрагменте карты. Укажите причину в поддержку своего ответа.
    Ответ:
    Сезонное количество осадков из-за наличия сухих резервуаров и сезонных водотоков.

    (e) Чем рисунок дренажа в квадрате сетки 1606 отличается от рисунка дренажа в квадрате сетки 1708?
    Ответ:
    В квадрате сетки 1606 рисунок дренажа является радиальным, поскольку многочисленные потоки выходят из центрального нагорья, а в квадрате сетки 1708 это решетчатая решетка, потоки, соединяющиеся с рекой под прямым углом.

    (f) (i) Какова схема заселения в квадрате 1904 года?
    Ответ:
    Рассеянный тип поселения.

    (ii) Назовите населенный пункт, в котором есть почтовое отделение.
    Ответ:
    Поселение Даттани.

    (g) Назовите две формы рельефа, представленные узором из контуров в квадрате сетки 1608.
    Ответ:
    Две формы рельефа, представленные узором из контуров в квадрате сетки 1608, представляют собой холмистый гребень (скалистый гребень) на западной стороне и плато. типовые особенности на южной стороне.

    (h) Вычислите наземное расстояние в километрах вдоль дороги с металлическим покрытием между дамбой в 1502 и дистанционным камнем, отмеченным «14» в квадрате сетки 1203.
    Ответ:
    Расстояние составляет 4 км.

    (i) Укажите две причины отсутствия человеческого жилья в северо-восточной части фрагмента карты.
    Ответ:

    • Холмистая местность
    • Многочисленные ручьи.

    (j) (i) Укажите разницу в высоте между максимальной высотой пятна на извлечении карты и высотой контура в квадрате сетки 1006.
    Ответ:
    Перепад высот 763 метра.

    (ii) Какое направление по компасу у Патлавы ка Голиа (590) от Качоли Дунгар (443)?
    Ответ:
    Северо-Восток

    Раздел B
    Карта Индии

    Вопрос 1.
    На контурной карте Индии отметьте и назовите следующее:
    (a) Восточные и Западные Гаты
    (b) Индо-Гангские равнины
    (c) Залив Камбхат
    (d) Перевал Натху-Ла (e) Тропик рака
    (f) Направление северо-восточного муссона
    (g) Морские месторождения нефти
    (h) Место добычи угля в Джаркханде
    ( i) Район с красной почвой
    (j) Хайдарабад
    Ответ:

    Вопрос 2.
    На контурной карте Индии отметьте и назовите следующее:
    (a) Хребет Виндхья
    (b) Побережье Конкан
    (c) Приток Ганги (d) Пролив Полк
    (e) Перевал Каракорум
    (f) Стандартный Меридиан Индии — 82 ° 30 ′ E
    (h) IronoreinSinghbhum
    (i) Аллахабад
    (j) Штат с самой высокой плотностью населения
    Ответ:

    Вопросы по карте Индии

    I. На контурной карте 1 Индии отметьте следующее:

    Вопросы
    1. Заштрихуйте важную область, где находится железо в Индии,
    2.Заштрихуйте и назовите плато в Индии, известное своими черноземами,
    3. Заштрихуйте и назовите область, где в Индии выпадает менее 25 см осадков.
    4. Отметьте и назовите Индийский стандартный меридиан.
    5. Отметьте и назовите равнины Инда.
    6. Заштрихуйте и назовите холмы Гаро, Хаси и Джайнтия.
    7. Отметьте стрелками циклон, который дает осадки в зимний сезон над северо-западными частями Индии.
    8. Марка и имя (i) Сринагар и (ii) Кочи.
    Ответ:

    II.На контурной карте Индии 2 отметьте следующее:

    Вопросы
    1. Отметьте и назовите Сундарбанс.
    2. Отметьте город Ченнаи.
    3. Марка и название Гималаев.
    4. Обозначьте жирной линией и назовите хребет Сатпура.
    5. Марка и название (i) реки Инд и (ii) реки Брахмапутра.
    6. Заштрихуйте и назовите равнины Раджастхана.
    7. Отметьте и назовите реку Коси.
    8. Отметьте и назовите стрелками направление ветров, дующих в Керале в летний сезон с Аравийского моря.
    9. Заштрихуйте и назовите плато Чотанагпур.
    Ответ:

    III. На контурной карте Индии 3 отметьте следующее:

    Вопросы
    1. Назовите и отметьте стрелками направление тропического циклона над Бенгальским заливом.
    2. Отметьте и назовите два региона в Индии, каждый из которых богат месторождениями
    (i) Уголь
    (ii) Нефть
    (iii) Железная руда
    (iv) Бокситы
    (v) Марганец.
    3. Отметьте и назовите хребет Каракорум, а также Западные и Восточные Гаты.
    4. Отметьте и назовите один регион в Индии для каждого из следующих
    (i) Аллювиальная почва
    (ii) Латеритная почва
    (iii) Красная почва.
    5. Заштрихуйте область в Индии, где ежегодно выпадает 100-200 см осадков.
    6. Отметьте и назовите
    (i) Дели и
    (ii) Вишакхапатнам.
    7. Отметьте и назовите Эверест и Восточные Гаты.
    8. Отметьте и назовите Реку Маханади и Реку Ямуну.
    9. Отметьте перевалы Каракорум и Натху Ла.
    10. Отметьте тропик Рака и стандартный меридиан.
    11. Отметьте направление юго-западного и северо-восточного муссонов. Ответ
    Ответ:

    IV. На контурной Карте 4 Индии отметьте следующее.

    Вопросы
    1. Отметьте следующие города — (i) Хайдарабад, (ii) Аллахабад (c) Калькутта.
    2. Отметьте и назовите две реки в Индии, впадающие в Аравийское море.
    3. Заштрихуйте и назовите область в Индии, которая получает максимальное количество осадков в результате северо-восточного муссона.
    4. Оттенок и название хребтов Аравалли.
    5. Затените ровные равнины Северной Индии.
    6. Отметьте и назовите Нилгири.
    7. Отметьте стрелками направление ветра, дующего зимой в Индии.
    8.Заштрихуйте и назовите область в Индии, которая получает дождь 1 из «трех источников».
    Ответ:

    V. На контурной карте Индии 5 отметьте следующее:

    Вопрос 1.
    Следующие города и поселки: Мумбаи, Калькутта, Ченнаи, Хайдарабад, Бангалор, Нью-Дели, Кочи, Сринагар и Вишакхапатнам.

    Вопрос 2.
    Отметьте и назовите следующие реки: Рави, Беас, Ченаб, Джелум, Сатледж, Ганга, Нармада, Годавари, Тапти, Кришна и Кавери.

    Вопрос 3.
    Заштрихуйте и назовите густонаселенные и малонаселенные районы.
    Ответ:

    Еще несколько дополнительных вопросов

    Вопрос 1.
    На контурной карте Индийского субконтинента, предоставленной вам в конце этого документа с вопросами:
    (a) Обозначьте реку Рави.
    (b) Обозначьте жирной линией и назовите холмы Сатпура.
    (c) Назовите реку Годавари.
    (d) Отметьте точкой и назовите в соответствующем месте столицу штата Мадхья-Прадеш.
    (e) Заштрихуйте и назовите место, где встречаются мангровые леса.
    (f) Распечатайте Уттар-Прадеш в надлежащем месте.
    (g) Обозначьте жирной линией и назовите Сулайманский хребет.
    (i) Заштрихуйте область с зимними дождями в Южной Индии.
    (j) Распечатайте ЗОЛОТОЙ мир на одной территории, где оно добывается. Проведите линию 10 ° северной широты, которая проходит недалеко от Коччи (Кочин).
    (k) Напечатайте жирным шрифтом «W» в регионе Пакистана, где выращивают пшеницу, и «T» для табака в одном регионе в западной Индии, где он широко выращивается.
    (l) Затените и назовите Ранн Катча.
    Ответ:

    Вопрос 2.
    На контурной карте Индийского субконтинента, предоставленной вам в конце этого вопросника:
    (a) Отметьте и назовите хребет Махадео.
    (b) Отметьте и обозначьте реку Чамбал.
    (c) Заштрихуйте и промаркируйте Рэнн из Катча.
    (d) Затенение и маркировка основного района производства кофе в Южной Индии.
    (e) Печать C над одним важным регионом, где добывается медь в Индии.
    (f) С помощью стрелок назовите ветры, которые приносят дожди на побережье Тамил Наду зимой.
    (g) Отметьте и назовите долготу, от которой Индия берет свое стандартное время.
    (h) Отметьте точкой и укажите название
    (i) Исламабад. (i) Калькутта.
    (i) Отметьте и обозначьте реку Сатледж.
    (j) Отметьте и назовите многоцелевой проект, построенный на реке Сон.
    (k) Заштрихуйте и назовите регион в Северной Индии, где выпадает менее 50 см осадков.
    Ответ:

    Вопрос 3.
    На контурной карте Индийского субконтинента, предоставленной вам в конце этого вопросника:
    (a) Обозначьте жирной линией горный хребет Сатпура.
    (b) Назовите реку Маханади.
    (c) Затените и назовите Ранн Катча.
    (d) Отметьте и назовите Реку Джелум.
    (e) Отметьте точкой и назовите город Лахор в соответствующем месте.
    (f) Напечатайте «TRF» в одном районе, где есть тропические дождевые леса.
    (g) Отметьте точкой и назовите Дакка.
    (h) Напечатайте «COTTON» на одном участке, где он выращивается к югу от тропика рака в Индии.
    (i) Выведите «I.O.» на одном участке, где добывается железная руда в Индии.
    (j) Отметьте точкой и назовите Аллахабад.
    (k) Заштрихуйте и напишите «DP» над одним районом с плотным населением к северу от тропика рака.
    (l) Отметьте точкой и назовите самый большой город на западном побережье.
    Ответ:

    Вопрос 4.
    На контурной карте Индийского субконтинента, предоставленной вам в конце этого вопросника, —
    (a) Отметьте жирной линией и надписью:
    (i) Хребет Каракорум ,
    (ii) Восточные Гаты.
    (iii) Нилгири.

    (b) Заштрихуйте и назовите по одному региону для каждого из следующих:
    (i) Растительность пустыни.
    (ii) Чернозём.
    (iii) Выращивание табака.

    (c) Марка и этикетка:
    (i) Хайберский перевал.
    (ii) Столица Ассам.
    (iii) Река Годавари.

    (d) (i) Идентифицировать, заштриховать и назвать:
    1. Район с малонаселенностью (менее 50 человек на кв. Км) в Северо-Восточной Индии.
    2. Район «Экваториальные леса».
    (ii) Отметьте одной длинной стрелкой направление летнего муссона над Бенгальским заливом.
    Ответ:

    Вопрос 5.
    На контурной карте Индийского субконтинента, предоставленной вам в конце этого вопросника:
    (a) Отметьте и подпишите
    (i) Сулейманские хребты.
    (ii) Западные Гаты.
    (iii) перевал Болан.

    (b) Заштрихуйте и назовите по одному региону для каждого из следующего:
    (i) Томская и кустарниковая растительность в Индии.
    (ii) Выращивание пшеницы в Пакистане.
    (iii) Выращивание кофе в Южной Индии.

    (c) Оттенок и этикетка:
    (i) Район красной почвы в Индии.
    (ii) Пустыня Тар.
    (iii) Один регион в Индии, где выпадают зимние дожди.

    (d) Идентификация и название:
    (i) Столица Бангладеш
    (ii) Река Ямуна.
    (iii) Портовый город Пакистана.
    Ответ:

    Вопрос 6.
    На предоставленной вам контурной карте отметьте и подпишите следующее:
    (a) Гора Эверест.
    (b) 82 \ (\ frac {1} {2} \) ° E долгота
    (c) Река Инд.
    (d) Полуостров Катхиавар.
    (e) Залив Маннар.
    (f) Река Брахмапутра.
    (g) Столица Непала.
    (h) Дакка.
    (i) Важный порт Пакистана.
    (j) Направление летнего муссона над Шри-Ланкой.
    Ответ:

    ICSE Geography Class 10 Банк вопросов

    ГЛАВА 3 — ЭЛЕМЕНТЫ ТОПОГРАФИИ

    ГЛАВА 3 — ЭЛЕМЕНТЫ ТОПОГРАФИИ



    3.1 Склоны
    3.2 Высота точки
    3.3 Изолинии
    3.4 Карты



    3.1.1 Определение
    3.1.2 Метод выражения
    склоны
    3.1.3 Поперечные откосы


    3.1.1 Определение

    Уклон — это подъем или опускание поверхности земли. Фермеру или ирригатору важно определить уклоны на земле.

    Склон легко узнать на холмистой местности. Начинайте подъем от подножия холма к вершине, это называется восходящим спуском (см. Рис. 46, черная стрелка). Спускайтесь под гору, это нисходящий спуск (см. Рис.46, белая стрелка).

    Рис. 46. Подъем и спуск

    Плоские участки никогда не бывают строго горизонтальными; на кажущейся ровной местности есть пологие склоны, но зачастую они едва заметны невооруженным глазом. Для выявления этих так называемых «пологих склонов» необходимо точное обследование местности.

    3.1.2 Метод обозначения уклонов

    Наклон поля выражается в виде отношения. Это расстояние по вертикали или разница в высоте между двумя точками в поле, деленная на расстояние по горизонтали между этими двумя точками.Формула:

    ….. (14a)

    Пример приведен на рис. 47.

    Рис. 47. Размеры откоса

    Наклон также может быть выражен в процентах; тогда используется формула:

    ….. (14b)

    Используя те же измерения, что и на рис.47:

    Наконец, наклон можно выразить в промилле; тогда используется формула:

    ….. (14c)

    с цифрами из того же примера:

    ПРИМЕЧАНИЕ :

    Уклон в ‰ = уклон в% x 10

    ВОПРОС

    Каков уклон в процентах и ​​промилле поля с горизонтальной длиной 200 м и разницей в высоте, равной 1.5 м между верхом и низом?

    ОТВЕТ

    Наклон поля в = наклон поля в% x 10 = 0,75 x 10 = 7,5 ‰

    ВОПРОС

    Какова разница в высоте между верхом и низом поля при длине поля по горизонтали 300 м и уклоне 2 ‰.

    ОТВЕТ

    таким образом: перепад высот (м) = 0.002 х 300 м = 0,6 м.

    В следующей таблице показан диапазон уклонов, обычно встречающихся на орошаемых полях.

    Наклон

    %

    Горизонтальный

    0 — 0,2

    0–2

    Очень плоский

    0.2 — 0,5

    2–5

    Квартира

    0,5 — 1

    5-10

    Умеренная

    1 — 2,5

    10-25

    Крутой

    более 2,5

    более 25

    Рис.48а. Крутой спуск

    Рис. 48б. Плоский склон

    3.1.3 Поперечный спуск

    Положите книгу на стол и приподнимите одну сторону ее на 4 см от стола (рис. 49a). Теперь наклоните книгу вбок (6 см) так, чтобы только один ее угол касался стола (рис. 49b).

    Рис. 49а. Главный уклон

    Рис. 49b. Главный и поперечный уклон

    Толстая стрелка указывает направление того, что можно назвать основным уклоном; тонкая стрелка указывает направление поперечного откоса, последний пересекает направление основного откоса.

    Изображение основного и поперечного откосов орошаемого поля показано на Рис. 50.

    Рис. 50. Главный и поперечный уклон орошаемого поля


    3.2.1 Определение
    3.2.2 Репер и
    средний уровень моря


    3.2.1 Определение

    На рисунке 51 точка A находится наверху бетонного моста. Любая другая точка в окружающей области выше или ниже точки A, и можно определить расстояние по вертикали между ними.Например, B выше, чем A, а расстояние по вертикали между A и B составляет 2 м. Точка C ниже точки A, а расстояние по вертикали между точками A и C составляет 1 м. Если точка A выбрана в качестве опорной точки или опорной точки, высота любой другой точки поля может быть определена как расстояние по вертикали между этой точкой и A.

    Рис. 51. Контрольная точка или точка отсчета «A»

    Таким образом, высота или превышение точки B по отношению к точке отсчета A составляет 2 м, а высота точки C также связана с точкой отсчета A , составляет 1 м.

    В качестве напоминания о том, что точка находится выше или ниже нулевой точки, ее отметке предшествует знак + (плюс), если она выше нулевой точки, или — (минус), если она ниже нулевой точки.

    Следовательно, по отношению к точке A высота точки B составляет +2 м, а высота точки C — -1 м.

    3.2.2 Репер и средний уровень моря

    Репер — это постоянная отметка, установленная в поле для использования в качестве контрольной точки. Репером может быть бетонное основание, в котором закреплен железный стержень, указывающий точное место реперной точки.

    Репер также может быть постоянным объектом на ферме, например, вершиной бетонной конструкции.

    В большинстве стран топографические отделы создали национальную сеть реперов с официально зарегистрированными отметками. Все высоты реперов даны относительно одной национальной опорной плоскости, которая в целом является средним уровнем моря (MSL) (см. Рис. 52).

    Рис. 52. Репер (B.M.) и средний уровень моря (M.S.L.)

    ПРИМЕР

    На рисунке 52 высота точки A по отношению к реперу (BM) составляет 5 метров.Высота БМ относительно среднего уровня моря (СММ) составляет 10 м. Таким образом, высота точки A относительно ПМР составляет 5 м + 10 м = 15 м и называется пониженным уровнем (RL) A.

    ВОПРОС

    Каков пониженный уровень точки B на рисунке 52.

    ОТВЕТ

    Превышение точки Б относительно БМ = 3 м.

    Высота БМ относительно ПДС = 10 м.

    Таким образом, приведенный уровень В = 3 м + 10 м = 13 м.

    ВОПРОС

    Какая разница в высоте между точками A и B? Что это собой представляет?

    ОТВЕТ

    Разница в высоте между A и B — это уменьшенный уровень A минус уменьшенный уровень B = 15 м — 13 м = 2 м, который представляет собой вертикальное расстояние между A и B.

    Контурная линия — это воображаемая горизонтальная линия, соединяющая все точки поля с одинаковой высотой. Контурная линия является воображаемой, но ее можно визуализировать на примере озера.

    Уровень воды в озере может подниматься и опускаться, но поверхность воды всегда остается горизонтальной. Уровень воды на береговой линии озера образует контурную линию, поскольку достигает точек, находящихся на одной высоте (рис. 53a).

    Фиг.53а. Береговая линия озера образует контур

    Предположим, что уровень воды в озере поднимается на 50 см выше своего первоначального уровня. Контурная линия, образованная береговой линией, изменяется и принимает новую форму, теперь соединяя все точки на 50 см выше первоначального уровня озера (рис. 53b).

    Рис. 53b. При повышении уровня воды образуется новая контурная линия

    Контурные линии — полезные средства для иллюстрации топографии поля на плоской карте; высота каждой изолинии указана на карте, чтобы можно было идентифицировать холмы или впадины.


    3.4.1 Описание карты
    3.4.2 Интерпретация
    контурных линий на карте
    3.4.3 Ошибки в
    контурные линии
    3.4.4 Масштаб карты


    3.4.1 Описание карты

    Рис. 54 представляет собой трехмерный вид поля с его холмами, долинами и впадинами; контурные линии также были обозначены.

    Рис. 54. Трехмерное изображение

    Такое представление дает очень хорошее представление о том, как поле выглядит в действительности.К сожалению, рисование требует большого мастерства и практически бесполезно при проектировании дорог, ирригационных и дренажных инфраструктур. Намного более точное и удобное представление поля, на котором могут быть нанесены все данные, относящиеся к топографии, является карта (рис. 55). Карта — это то, что вы видите, глядя на трехмерное изображение (Рис. 54) сверху.

    3.4.2 Расшифровка контурных линий на карте

    Расположение контурных линий на карте дает прямое представление об изменениях в топографии поля (рис.55).

    Рис. 55. Двумерный вид или карта

    В холмистой местности линии изолиний близки друг к другу, а на пологих склонах они шире. Чем ближе линии контура, тем круче наклон. Чем шире контурные линии, тем более пологие откосы.

    На холме контурные линии образуют круги; при этом значения их высоты увеличиваются от края к центру.

    В углублении контурные линии также образуют круги; однако значения их возвышения уменьшаются от края к центру.

    3.4.3 Ошибки в контурных линиях

    Линии разной высоты никогда не пересекаются. Пересечение линий обратного отсчета означало бы, что точка пересечения находится на двух разных отметках, что невозможно (см. Рис. 56).

    Рис. 56. НЕПРАВИЛЬНО; пересечение контурных линий

    Контур непрерывный; где-то на карте никогда не может быть изолированного отрезка контурной линии, как показано на рисунке 57.

    Фиг.57. НЕПРАВИЛЬНО; изолированный кусок контурной линии

    3.4.4 Масштаб карты

    Чтобы карта была полной и действительно полезной, она должна иметь определенный масштаб. Масштаб — это отношение расстояния между двумя точками на карте и их реального расстояния на поле. Масштаб 1 к 5000 (1: 5000) означает, что 1 см, измеренный на карте, соответствует 5000 см (или 50 м в метрах) на поле.

    ВОПРОС

    Каково реальное расстояние между точками A и B на поле, когда эти две точки равны 3.На расстоянии 5 см на карте масштабом от 1 до 2 500? (см. рис.58)

    Рис. 58. Измерение расстояния между A и B

    ОТВЕТ

    Масштаб 1: 2 500, что означает, что 1 см на карте соответствует 2 500 см в действительности. Таким образом, 3,5 см между точками A и B на карте соответствует 3,5 x 2 500 см = 8 750 см или 87,5 м на поле.


    Практическая работа по географии Решения класса 11 Глава 5 Топографические карты

    Практическая работа по географии Решения класса 11 Глава 5 Топографические карты

    Класс 11 Практическая работа по географии Глава 5 Учебник NCERT Решенные вопросы

    1.Ответьте на следующие вопросы примерно 30 словами.

    Вопрос 1 (i).
    Что такое топографические карты?
    Ответ:
    Они также известны как карты общего назначения. Они нарисованы в относительно большом масштабе. На этих картах показаны важные природные и культурные особенности, такие как рельеф, растительность, водоемы, возделываемые земли, поселения, транспортные сети и т. Д. Другими словами, это карта небольшой территории, нарисованная в большом масштабе, с подробным изображением деталей поверхности и естественный и искусственный.Рельеф на этой карте показан контурами.

    Вопрос 1 (ii).
    Назовите организацию, которая готовит топографические карты Индии. Ответ
    :
    Топографические карты серии «Индия» и «Соседние страны» были подготовлены Службой Обзора Индии до начала Дели Обзорной конференции в 1937 году. С тех пор подготовка карт для соседних стран была прекращена, и Обзор Индии ограничился подготовить и опубликовать топографические карты Индии в соответствии со спецификациями, установленными для Международной серии карт мира.

    Вопрос 1 (iii).
    Какие шкалы обычно используются для картографирования нашей страны в Обзоре Индии?
    Ответ:
    Топографические карты Индии подготовлены в масштабе 1: 10,00,000, 1: 250,000, 1: 1,25,000,1: 50,000 и 1: 25,000, обеспечивая широтное и долготное покрытие 4 ° x 4 °, 1. ° x 1 °, 30 ′ x 30 ′, 15 ′ x 15 ′ и 5 ′ x 7 ′ 30 ″ соответственно.

    Вопрос 1 (iv).
    Что такое контуры?
    Ответ:
    Контуры — это воображаемые линии, соединяющие места, имеющие одинаковую высоту над средним уровнем моря.Другими словами, это воображаемые линии, соединяющие все точки одинаковой высоты или высоты над средним уровнем моря. Их еще называют «линиями уровня». Карта, показывающая рельеф местности по контурам, называется контурной картой. Метод отображения рельефа через контур очень полезен и универсален. Контурные линии на карте дают полезное представление о топографии местности.

    Вопрос 1 (v).
    На что указывает расстояние между контурами?
    Ответ:
    Расстояние в контурах обозначает уклон.

    • близко расположенные контуры обозначают крутые склоны.
    • Широко расставленные контуры обозначают пологий уклон.

    Вопрос 1 (vi).
    Что такое условные знаки?
    Ответ:
    Существуют определенные внутренние стандартные символы, знаки и цвета, которые используются для изображения поселений, зданий, автомобильных и железных дорог и являются важными культурными объектами, отображаемыми на топографических листах. Когда эти особенности отображаются с помощью обычных знаков, символов и цветов, становится легко понять и интерпретировать карту.Обычные знаки и символы признаны во всем мире, поэтому любой может читать любую карту в любой точке мира, не зная языка этой конкретной страны.

    2. Сделайте короткие заметки по-
    Вопрос 2 (i)
    Контуры
    Ответ:
    Вот некоторые основные особенности контурных линий:

    • Контурная линия проводится для обозначения мест одинаковой высоты.
    • Контурные линии и их формы представляют высоту, уклон или уклон рельефа.
    • Близко расположенные контуры представляют крутые склоны, а широко разнесенные контуры представляют пологий склон.
    • Когда две или более горизонтальных линии сливаются друг с другом, они представляют элементы вертикальных склонов, такие как скалы или водопады.
    • Два контура разной высоты обычно не пересекаются.

    Вопрос 2 (ii).
    «Маргинальная информация» на топографических листах
    Ответ:
    Маргинальная информация: включает номер топографического листа, его местоположение, привязку к сетке, его протяженность в градусах и минутах, масштаб, охватываемые районы и т. Д. Другими словами, он предоставляет информацию связанных с тем, что именно показывает топографический лист.Без этой второстепенной информации невозможно извлечь какое-либо значение из топографического листа.

    Вопрос 2 (iii).
    Обзор Индии
    Ответ:
    Обзор Индии готовит топографические карты Индии для всей страны. Топографические карты из серии «Индия» и «Соседние страны» были подготовлены Службой Обзора Индии до начала Дели Обзорной конференции в 1937 году. С этого момента подготовка карт для соседних стран была прекращена, и Обзор Индии ограничился подготовкой и публикацией. топографические карты Индии в соответствии со спецификациями Международной серии карт мира.

    3. Объясните, что означает «интерпретация карты» и какая процедура используется для ее интерпретации.
    Ответ:
    Интерпретация карты включает изучение факторов, которые объясняют причинно-следственную связь между несколькими элементами, показанными на карте. Например, распределение естественной растительности и обрабатываемых земель можно лучше понять на фоне рельефа и дренажа. Аналогичным образом, распределение населенных пунктов может быть изучено в связи с уровнями системы транспортной сети и характером топографии.

    Получение точных значений из карт называется интерпретацией карт. Знание языка карты и чувство направления необходимы при чтении и интерпретации топ-листов. Сначала мы должны найти северную линию и масштаб карты и соответственно сориентироваться. Мы должны хорошо знать легенды / ключи, приведенные на карте, с изображением различных объектов. Все топ-листы содержат таблицу с условными знаками и символами, используемыми на карте. Мы должны быть знакомы с условными символами, знаками и цветами.

    При интерпретации карты следует следующая процедура:

    • Определение местоположения области в Индии по порядковому номеру топографического листа. Это дало бы представление об общих характеристиках основных и второстепенных физико-географических подразделений области.
    • Найдите масштаб карты и интервал изолиний, которые дадут протяженность и общий рельеф местности.
    • Найдите на копировальных листах следующие элементы.
      • Основные формы рельефа — как показано контурами и другими графическими элементами.
      • Дренажные и водные объекты — главная река и ее важные притоки.
      • Землепользование — т.е. леса, сельскохозяйственные угодья, отходы, заповедник, парк, школа и т. Д.
      • Схема населенных пунктов и транспорта.
    • Объясните схему распределения каждой из функций по отдельности, обратив внимание на наиболее важный аспект.
    • Наложите пары этих карт и запишите отношения, если таковые имеются, между двумя образцами. Например, если контурная карта накладывается на карту землепользования, она обеспечивает взаимосвязь между степенью уклона и типом используемой земли.
    • Аэрофотоснимки и спутниковые изображения той же местности и того же масштаба также можно сравнить с топографической картой для обновления информации.

    4. Если вы интерпретируете культурные особенности на топографическом листе, какую информацию вы хотели бы искать и как бы вы получили эту информацию? Обсудите с помощью подходящих примеров.
    Ответ:
    Поселения, здания, дороги и железные дороги являются важными культурными объектами, отображаемыми на топографических листах с помощью обычных знаков, символов и цветов.Расположение и характер распределения различных объектов помогают понять область, показанную на карте. Распределение населенных пунктов можно увидеть на карте по их местонахождению, схеме расположения, выравниванию и плотности.

    Природу и причины различных моделей поселений можно четко понять, сравнив карту поселения с контурной картой. Схема транспорта и связи Характер рельефа, населения, размера и освоения ресурсов области напрямую влияет на средства транспорта и связи, а также на их плотность.Они обозначаются обычными знаками и символами. Транспортные средства и средства связи предоставляют полезную информацию о местности, указанной на карте.

    Населенные пункты, род занятий, средства связи и транспорта, структура землепользования — вот некоторые из культурных особенностей, которые показаны на топографическом листе с использованием обычных знаков, цветов и символов. Нам необходимо собрать информацию для всего этого, мы можем получить всю эту информацию через Survey of India. Транспортные средства включают национальные или государственные автомагистрали, окружные дороги, пути для телеги, верблюжьи пути, пешеходные дорожки, железные дороги, водные пути, основные линии связи, почтовые отделения и т. Д.Поселения изучаются по двум направлениям: сельские поселения и городские поселения.

    Общее занятие жителей района можно определить по землепользованию и типу населенного пункта. Например, в сельской местности основным занятием большинства людей является сельское хозяйство; в районах проживания племен преобладают лесозаготовки и примитивное земледелие, а в прибрежных районах практикуется рыболовство. Точно так же в больших и малых городах услуги и бизнес кажутся основными занятиями людей.

    5. Нарисуйте условные знаки и символы для следующих объектов:

    1. Международная граница
    2. Контрольная отметка
    3. Деревни
    4. Металлическая дорога
    5. Тропа с мостами
    6. Места поклонения
    7. Железнодорожная линия.

    Ответ:

    УПРАЖНЕНИЕ A

    Изучите контурный узор и ответьте на следующие вопросы.

    1. Назовите географический объект, образованный контурами.
    Ответ:
    Плато

    2.Найдите на карте интервал изолиний.
    Ответ:
    100 метров

    3. Найдите на карте расстояние между E и F и преобразуйте его в наземное расстояние.
    Ответ:
    2 см = 4 км по земле

    4. Назовите тип уклона между A и B; C и D, E и F.
    Ответ:
    A и B пологий склон
    C и D крутой склон
    E и F пологий склон

    5. Найдите направление E, D и F от G.
    Ответ:
    От точки G точка E находится на западе, D — на севере, а F — на юге.

    УПРАЖНЕНИЕ B

    Изучите отрывок из топографической карты № 63К / 12. как показано на рисунке ниже, и ответьте на следующие вопросы.

    1. Преобразуйте 1: 50 000 в указание масштаба.
    Ответ:
    1 см на карте показывает 50000 см на земле.

    2. Назовите основные населенные пункты области.
    Ответ:
    Качхва, Прем Ка Пура, Бхатаули, Бахрейн.

    3. Какое направление течения реки Ганга?
    Ответ:
    Направление течения реки Ганга с северо-запада на юго-восток.

    4. На каком из берегов реки Ганга, Бхатаули находится?
    Ответ:
    Находится посреди реки Ганга.

    5. Какова структура сельских поселений на правом берегу реки Ганга?
    Ответ:
    Вдоль правого берега реки Ганга виден компактный рисунок сельских поселений.

    6. Назовите села / населенные пункты, в которых находится почтовое отделение / почтово-телеграфное отделение?
    Ответ:
    Поселки с указанием ЗП и ВОМ имеют почтовое или почтово-телеграфное отделение.

    7. К чему относится желтый цвет в области?
    Ответ:
    Желтый цвет в местности относится к равнинам.

    8. Каким транспортным средством пересекают реку жители деревни Бхатаули?
    Ответ:
    Лодки переходили реку жители деревни Бхатаули.

    УПРАЖНЕНИЕ C

    Изучите отрывок для топографической карты 63K / 12, показанный на приведенном ниже рисунке, и ответьте на следующие вопросы.

    1. Укажите высоту самой высокой точки на карте.
    Ответ:
    208 метров.

    2. В какой части карты протекает река Джанитихва Нади 0
    Ответ:
    Река Джамтихва Нади протекает через юго-восточную четверть карты.

    3. Какое крупное поселение находится на востоке Куардари-Нала?
    Ответ:
    Поселение Бандхва находится на востоке Куардари-Нала.

    4. Какой тип поселения находится в этой местности?
    Ответ: В этом районе есть
    линейных населенных пунктов. ;

    5.Назовите географический объект, представленный белыми пятнами в центре Сипиу Нади.
    Ответ:
    Это равнины.

    6. Назовите два типа растительности, показанные на части топографического листа.
    Ответ:
    Тропическая лиственная растительность.

    7. В каком направлении течет Куардари?
    Ответ:
    Он течет с юга на север.

    8. В какой части листа находится плотина Нижняя Хаджури?
    Ответ:
    В южной части листа находится Нижняя Хаджурская плотина.

    Практическая работа по географии 11 класса Глава 5 Дополнительные вопросы NCERT

    Практическая работа 11 класса по географии Глава 5 Вопросы с несколькими вариантами ответов

    Вопрос 1.
    Что из перечисленного не является типом j поселений?
    (а) Компактный
    (б) Дисперсный
    (в) Линейный
    (г) Полярный.
    Ответ:
    (d) Полярный

    Вопрос 2.
    Что из перечисленного не относится к городам городского типа?
    (a) Горные города
    (b) Столицы
    (c) Красивые города
    (d) Религиозные города.
    Ответ:
    (c) Города красоты

    Вопрос 3.
    Широко протяженная плоская возвышенность с относительно более крутыми склонами, возвышающаяся над прилегающей равниной или морем, называется a:
    (a) Гора
    (b) Долина
    (в) Отрог
    (г) Плато.
    Ответ:
    (d) Плато

    Вопрос 4.
    Внезапный и более или менее перпендикулярный спуск воды со значительной высоты в русле реки называется a:
    (a) Отрог
    (b) Долина
    ( в) Водопад
    (г) Плато.
    Ответ:
    (c) Водопад

    Вопрос 5.
    Язычок земли, выступающий с возвышенности в нижнюю, называется a:
    (a) Отрог
    (b) Долина
    (c) Водопад
    (d) Плато .
    Ответ:
    (а) Шпор.

    Класс 11 Практическая работа по географии Глава 5 Очень короткий тип ответа Вопросы

    Вопрос 1.
    Чем карты рельефа отличаются от топографических карт?
    Ответ:

    Топографические карты Карты рельефа
    На этих картах показаны важные природные и культурные особенности, такие как рельеф, растительность, водоемы, обрабатываемые земли, поселения, транспортные сети и т. Д. Поверхность Земли неоднородна и варьируется от гор до холмов, плато и равнин. Возвышения и впадины земной поверхности известны как физические особенности или особенности рельефа земли. Карта, показывающая эти особенности, называется рельефной картой.
    Это карты общего назначения. Это так называемые карты специального назначения.
    Они нарисованы в относительно большем масштабе. Они нарисованы в относительно меньшем масштабе.

    Вопрос 2.
    Какие методы используются для отображения топографии?
    Ответ:
    На протяжении многих лет использовался ряд методов для отображения рельефа поверхности Земли на картах. Эти методы включают штриховку, затенение холмов, оттенки слоев, контрольные отметки, а также высоту и контуры пятен. Однако контуры и высоты пятна преимущественно используются для изображения рельефа местности на всех топографических картах.

    Вопрос 3.
    Что такое Hachure?
    Ответ:
    Небольшие прямые линии, проведенные на карте в направлении максимального уклона, проходящие через контуры.Они дают представление о различиях уклона земли.

    Вопрос 4.
    Определить горизонтальные линии.
    Ответ:
    Это воображаемые линии, соединяющие все точки одинаковой высоты или высоты над средним уровнем моря. Их еще называют «линиями уровня».

    Вопрос 5.
    Различия между вертикальным интервалом и горизонтальным расстоянием.
    Ответ:

    Основа Интервал по вертикали Расстояние по горизонтали
    Значение Интервал между двумя последовательными контурами называется интервалом по вертикали. Расстояние по горизонтали — это расстояние между двумя контурами по горизонтали.
    Изменить Вертикальный интервал между двумя последовательными горизонтальными линиями остается постоянным. Он варьируется от места к месту в зависимости от характера склона. Он больше, когда уклон пологий, и уменьшается с увеличением уклона.
    Выражение Выражается как VT. Горизонтальное расстояние, также известное как горизонтальный эквивалент (HE).

    Вопрос 6.
    Что такое топографическая карта?
    Ответ:
    Карта небольшой области, нарисованная в большом масштабе, с подробными изображениями природных и искусственных элементов поверхности. Рельеф на этой карте показан контурами.

    Вопрос 7.
    Объясните различные типы населенных пунктов, показанные на топографическом листе.
    Ответ:
    На карте можно выделить четыре типа сельских поселений.
    (а) Компактный
    (б) Разрозненный
    (в) Линейный
    (г) Круговой.

    Городские поселения выделены как

    • Город на перекрестке
    • Узловая точка
    • Торговый центр
    • Горная станция
    • Прибрежный курортный центр
    • Порт
    • Производственный центр с пригородными деревнями или городами-спутниками
    • Столица

      Религиозный центр.

    Вопрос 8.
    Как плотность заселения связана с продовольствием?
    Ответ:
    Плотность заселения напрямую связана с обеспеченностью продуктами питания.

    Вопрос 9.
    Что такое линейные поселения?
    Ответ:
    Иногда сельские поселения образуют ряды, т. Е. Располагаются вдоль речной долины, дороги, набережной, береговой линии — их называют линейными поселениями.

    Класс 11 Практические занятия по географии Глава 5 Тип краткого ответа Вопросы

    Вопрос 1.
    Каковы особенности контуров?
    Ответ:
    Некоторые основные характеристики контурных линий:

    • Контурная линия проводится для обозначения мест одинаковой высоты.
    • Контурные линии и их формы представляют высоту, уклон или уклон рельефа.
    • Расстояние между горизонтальными линиями обозначает уклон. Близко расположенные контуры представляют крутые склоны, а широко разнесенные контуры представляют пологий склон.
    • Когда две или более горизонтальных линии сливаются друг с другом, они представляют элементы вертикальных склонов, такие как скалы или водопады.
    • Два контура разной высоты обычно не пересекаются.

    Вопрос 2.
    Объясните, как мы интерпретируем топографический лист?
    Ответ:
    Важно знать язык карты и чувство направления, которые необходимы при чтении и интерпретации топографических листов. Сначала мы должны найти северную линию и масштаб карты и соответственно сориентироваться. Мы должны хорошо знать легенды / ключи, приведенные на карте, с изображением различных объектов. Все топ-листы содержат таблицу с условными знаками и символами, используемыми на карте. Мы должны быть знакомы с условными символами, знаками и цветами.

    Вопрос 3.
    Под какими заголовками интерпретируется топографический лист? Вкратце объясните каждое.
    Ответ:
    Топографический лист обычно интерпретируется следующим образом:

    • Маргинальная информация: включает номер топографического листа, его местоположение, привязку к сетке, его протяженность в градусах и минутах, масштаб, охватываемые районы и т. Д.
    • Рельеф района: общая топография территории изучается для определения различных форм рельефа, включая пики, гребни, отроги и общее направление склона.
    • Дренаж Района: Нам также необходимо интерпретировать важные реки и их притоки, а также тип и протяженность образованных ими долин, типы дренажной системы, т. Е. Дендритный, радиальный, кольцевой, решетчатый, внутренний и т. Д.
    • Землепользование: включает использование земель в различных категориях, таких как естественная растительность и лес, часть территории которой покрыта лесами, будь то густой или редкий лес, а также категории лесов, найденных там, например, зарезервированные, охраняемые, классифицированные / неклассифицированные.
    • Транспорт и связь: Транспортные средства включают национальные или государственные автомагистрали, окружные дороги, пути для телеги, верблюжьи пути, пешеходные дорожки, железные дороги, водные пути, основные линии связи, почтовые отделения и т. Д., Каждая из которых представлена ​​на топографическом листе.
    • Населенный пункт: Населенные пункты изучаются под сельскими поселениями и городскими поселениями.
    • Род занятий: Общее занятие жителей района можно определить по землепользованию и типу населенного пункта.

    Вопрос 4.
    Какие факторы определяют местонахождение населенных пунктов?
    Ответ:
    Расположение населенных пунктов определяется различными факторами, например,

    • Источник воды
    • Обеспечение продуктами питания
    • Характер рельефа
    • Природа и характер занятия
    • Оборона.

    Вопрос 5.
    Напишите небольшую заметку по интерпретации карты.
    Ответ:
    Интерпретация карты включает изучение факторов, которые объясняют причинно-следственную связь между несколькими элементами, показанными на карте.Например, распределение естественной растительности и обрабатываемых земель можно лучше понять на фоне рельефа и дренажа. Аналогичным образом, распределение населенных пунктов может быть изучено в связи с уровнями системы транспортной сети и характером топографии.

    Класс 11 Практическая работа по географии Глава 5 Длинный тип ответа Вопросы

    Вопрос 1.
    Объясните этапы рисования, поперечный разрез по их контурам в различных топографических формах рельефа.
    Ответ:
    Чтобы нарисовать поперечные сечения различных рельефных элементов по их контурам, можно выполнить следующие шаги:

    • Нарисуйте прямую линию, пересекающую контуры на карте, и отметьте ее как XY.
    • Возьмите полоску белой бумаги или диаграммы и поместите ее край вдоль линии XY.
    • Отметьте положение и значение каждого контура, пересекающего линию XY.
    • Выберите подходящий масштаб по вертикали, например V2 см = 100 метров, чтобы провести горизонтальные линии, параллельные друг другу и равные длине XY.Количество таких линий должно быть равно или больше общего количества горизонтальных линий.
    • Обозначьте соответствующие значения, соответствующие значениям контура по вертикали поперечного сечения. Нумерацию можно начать с наименьшего значения, представленного контурами.
    • Поместите край отмеченной бумаги вдоль горизонтальной линии в нижней линии поперечного сечения таким образом, чтобы XY бумаги соответствовала XY карты, и отметьте точки контура.
    • Нарисуйте перпендикуляры от линии XY, пересекающей контурные линии, к соответствующей линии в основании поперечного сечения.
    • Плавно соедините все точки, отмеченные на разных линиях у основания поперечного сечения.

    Вопрос 2.
    Под какими заголовками поясняются топографические карты?
    Ответ:
    Топографический лист обычно интерпретируется следующим образом:

    • Маргинальная информация: включает номер топографического листа, его местоположение, привязку к сетке, его протяженность в градусах и минутах, масштаб, охватываемые районы и т. Д.
    • Рельеф района: общая топография территории изучается для определения различных форм рельефа, включая пики, гребни, отроги и общее направление склона.Эти особенности изучаются в следующих разделах:
      • Холм: с вогнутым, выпуклым, крутым или пологим склоном и формой.
      • Плато: широкое, узкое, плоское, волнистое или рассеченное.
      • Равнина: ее типы, т. Е. Аллювиальные, ледниковые, карстовые, поастальные, болотистые и т. Д.
      • Горы: общая высота, вершины, перевалы и т. Д.
    • Дренаж территории: нам также необходимо интерпретировать важные реки и их притоков, а также типа и протяженности образованных ими долин, типов водосбора, т.е.е. дендритная, радиальная, кольцевая, решетчатая, внутренняя и т. д.
    • Землепользование: включает использование земель в различных категориях, таких как естественная растительность и лес, какая часть территории покрыта лесом, будь то густой или редкий лес, и категории лесов, найденные там, такие как Зарезервированные, Защищенные, Классифицированные / Неклассифицированные.
    • Транспорт и связь: транспортные средства включают национальные или государственные автомагистрали, окружные дороги, пути для телеги, верблюжьи пути, пешеходные дорожки, железные дороги, водные пути, основные линии связи, почтовые отделения и т. Д.топографический лист представляет каждое из них.
    • Поселение: Поселения изучаются по следующим разделам:
      • Сельские поселения: Типы и структура сельских поселений: компактные, полукомпактные, рассредоточенные, линейные и т. Д.
      • Городские поселения: Тип городских поселений и их функции , т. е. столицы, административные города, религиозные города, портовые города, горные станции и т. д.
    • Род занятий: Общее занятие жителей района можно определить с помощью землепользования и типа поселения.Например, в сельской местности основным занятием большинства людей является сельское хозяйство; в районах проживания племен преобладают лесозаготовки и примитивное земледелие, а в прибрежных районах практикуется рыболовство. Точно так же в больших и малых городах услуги и бизнес кажутся основными занятиями людей.

    Вопрос 3.
    Объясните идентификацию культурных объектов по топографическим листам.
    Ответ:
    Поселения, здания, дороги и железные дороги — важные культурные объекты, показанные на топографических листах с помощью обычных знаков, символов и цветов.Расположение и характер распределения различных объектов помогают понять область, показанную на карте.
    Типы населенных пунктов: На карте можно выделить четыре типа сельских поселений:

    1. Компактные
    2. Разрозненные
    3. Линейные
    4. Круговые

    Городские центры выделяются как:

    • Город на перекрестке
    • Узловая точка
    • Рыночный центр
    • Горная станция
    • Прибрежный курортный центр
    • Порт
    • Производственный центр с пригородными деревнями или городами-спутниками
    • Столица
    • Религиозный центр

    Местоположение поселений:
    Следует внимательно изучить со ссылкой на контурная и дренажная карта.Плотность заселения напрямую связана с обеспеченностью продуктами питания. Иногда сельские поселения образуют ряды, т. Е. Расположены вдоль речной долины, дороги, набережной, береговой линии — их называют линейными поселениями. В случае городского поселения город-перекресток имеет веерообразную форму, дома располагаются вдоль дороги, а перекресток находится в центре города и на главной рыночной площади. В узловом городе дороги расходятся во все стороны.

    Транспорт и связь:
    Рельеф, население, размер и структура освоения ресурсов в районе напрямую влияют на средства транспорта и связи, а также на их плотность.Они обозначаются обычными знаками и символами. Транспортные средства и средства связи предоставляют полезную информацию о местности, указанной на карте.

    Класс 11 Практическая работа по географии Глава 4 Вопросы Viva

    Вопрос 1.
    Для чего нужны топографические карты?
    Ответ:
    Они служат для базовых карт, а
    используются для рисования всех остальных карт.

    Вопрос 2.
    Кто готовит топографические карты в Индии для всей страны?
    Ответ:
    Обзор Индии готовит топографические карты Индии для всей страны.

    Вопрос 3.
    Каковы особенности рельефа земли? Ответ: Высота и впадины земной поверхности известны как физические особенности или особенности рельефа земли.

    Вопрос 4.
    Назовите некоторые методы, используемые для изображения рельефа земли. Какие из них наиболее распространены?
    Ответ:
    На протяжении многих лет использовался ряд методов для отображения рельефа поверхности Земли на картах. Эти методы включают штриховку, штриховку холмов, оттенки слоев, контрольные отметки, а также высоту и контуры пятен.Однако контуры и высоты пятна преимущественно используются для изображения рельефа местности на всех топографических картах.

    Вопрос 5.
    Назовите откос, если контуры показывают следующие особенности:

    • Контуры на этом типе откоса широко разнесены в нижней части и близко расположены в верхней части.
    • Контуры широко расставлены в верхней части и близко расположены в нижней части.
    • Контуры, представляющие этот тип наклона, далеки друг от друга.
    • Контуры близко расположены Ответ:
      • Вогнутый склон
      • Выпуклый склон
      • Пологий склон
      • Крутой склон.
    Примечания к топографическим картам
    • Топографические карты, также известные как карты общего назначения, составлены в относительно большом масштабе. На этих картах показаны важные природные и культурные особенности, такие как рельеф, растительность, водоемы, обрабатываемые земли, поселения, транспортные сети и т. Д.Они служат для создания базовых карт и используются для рисования всех остальных карт.
    • Эти карты подготовлены и опубликованы Национальной картографической организацией каждой страны. Например, Survey of India готовит топографические карты Индии для всей страны.
    • Топографические карты составлены в виде серий карт разного масштаба. Следовательно, в данной серии все карты используют одну и ту же точку отсчета, масштаб, проекцию, условные знаки, символы и цвета.
    • Топографические карты Индии подготавливаются в двух сериях: серии «Индия и прилегающие страны» и «Международная серия карт мира».
    • Топографические карты из серии Индии и смежных стран были подготовлены Службой Обзора Индии до начала Дели Обзорной конференции в 1937 году. С этого момента подготовка карт для соседних стран была прекращена, и Обзор Индии ограничился подготовкой и опубликовать топографические карты Индии в соответствии со спецификациями, установленными для Международной серии карт мира.
    • Топографические карты Индии подготовлены в масштабах 1: 10,00,000, 1: 250,000, 1: 1,25,000, 1: 50,000 и 1: 25,000, обеспечивая широтное и долготное покрытие 4 ° x 4 °, 1 ° x. 1 °, 30 ′ x 30 ′, 15 ′ x 15 ′ и 5 ′ x 7 ′ 30 ″ соответственно.
    • На протяжении многих лет для отображения рельефа поверхности Земли на картах использовался ряд методов. Эти методы включают штриховку, затенение холмов, оттенки слоев, контрольные отметки, а также высоту и контуры пятен. Однако контуры и высоты пятна преимущественно используются для изображения рельефа местности на всех топографических картах.
    • Контурные линии на карте дают полезное представление о топографии местности. Ранее для рисования контуров на топографических картах использовались методы топографической съемки и нивелирования. Однако изобретение фотографии и последующее использование аэрофотосъемки заменили традиционные методы съемки, нивелирования и картирования.
    • Склоны можно в целом разделить на пологие, крутые, вогнутые, выпуклые, неровные или волнистые. Контуры различных типов откосов демонстрируют четкую схему расположения.
    • Когда две или более горизонтальных линии сливаются друг с другом, они представляют элементы вертикальных склонов, такие как скалы или водопады. Два контура разной высоты обычно не пересекаются.
    • Все топографические особенности показывают разную степень уклона. Например, плоская равнина имеет более пологий склон, а скалы и ущелья связаны с крутыми склонами. Точно так же долины и горные хребты также характеризуются различной степенью уклона, от крутых до пологих.Следовательно, расстояние между контурами важно, поскольку оно указывает на наклон.
    • Поселки, здания, дороги и железные дороги являются важными культурными объектами, отображаемыми на топографических листах с помощью обычных знаков, символов и цветов. Расположение и характер распределения различных объектов помогают понять область, показанную на карте.
    Топографические карты Важные термины
    • Топографические карты: Они также известны как карты общего назначения. Они нарисованы в относительно большом масштабе.На этих картах показаны важные природные и культурные особенности, такие как рельеф, растительность, водоемы, возделываемые земли, поселения, транспортные сети и т. Д.
    • Карты рельефа: Поверхность Земли неоднородна и варьируется от гор к холмам, плато и равнинам. . Возвышения и впадины земной поверхности известны как физические особенности или особенности рельефа земли. Карта, показывающая эти особенности, называется рельефной картой.
    • Серия «Индия и прилегающие страны»: Топографические карты серии «Индия и прилегающие страны» были подготовлены Службой Обзора Индии до начала проведения Дели Обзорной конференции в 1937 году.С этого момента подготовка карт для соседних стран была прекращена, и Survey of India ограничилось подготовкой и публикацией топографических карт Индии в соответствии со спецификациями, установленными для Международной серии карт мира.
    • Международная серия карт мира: Топографические карты в рамках Международной серии карт мира предназначены для создания стандартизированных карт всего мира в масштабе 1: 10,00,000 и 1: 250,000.
    • Контуры: это воображаемые линии, соединяющие все точки одинаковой высоты или высоты над средним уровнем моря.Их еще называют «линиями уровня».
    • Контурная карта: Карта, показывающая рельеф местности по контурам, называется контурной картой.
    • Интервал контура: интервал между двумя последовательными контурами. Он также известен как вертикальный интервал, обычно обозначаемый как V. I. Как правило, он постоянен для данной карты.
    • Поперечный разрез: вид сбоку на землю, разрезанную вертикально по прямой линии. Он также известен как раздел или профиль.
    • Hachures: Небольшие прямые линии, проведенные на карте в направлении максимального уклона, пересекающие контуры.Они дают представление о различиях уклона земли.
    • Пологий уклон: когда градус или угол наклона объекта очень мал, уклон будет пологим.
    • Крутой склон: если градус или угол наклона объекта большой, а контуры расположены близко друг к другу, они указывают на крутой уклон.
    • Вогнутый откос: Склон с пологим уклоном в нижних частях рельефа и крутым в его верхних частях называется вогнутым откосом.
    • Выпуклый склон: Выпуклый склон довольно пологий в верхней части и крутой в нижней части.В результате контуры широко разнесены в верхней части и близко расположены в нижней части.
    • Конический холм: Он почти равномерно возвышается над окружающей местностью. Конический холм с равномерным уклоном и узкой вершиной представлен концентрическими контурами, расположенными почти через равные промежутки.
    • Плато: Широко протяженная плоская возвышенность с относительно более крутыми склонами, возвышающаяся над прилегающей равниной или морем, называется плато.
    • Долина: Геоморфический объект, расположенный между двумя холмами или хребтами и образованный в результате боковой эрозии рекой или ледником, называется долиной.
    • Шпора: язычок земли, выступающий с возвышенности в нижнюю, называется шпорой. Он также представлен V-образными контурами, но в обратном порядке.
    • Утес: очень крутой или почти перпендикулярный обрыв рельефа. На карте обрыв можно определить по тому, как его контуры проходят очень близко друг к другу, в конечном итоге сливаясь в одно целое.
    • Водопад: Внезапный и более или менее перпендикулярный спуск воды со значительной высоты в русле реки называется водопадом.
    • Пороги: Когда водопад следует за каскадным потоком или предшествует ему, он образует пороги вверх или вниз по течению от водопада.
    Класс 11 География Решения NCERT

    Определение рельефа в географии

    В географии рельеф местности — это разница между самой высокой и самой низкой отметками. Например, с горами и долинами в этом районе впечатляет местный рельеф национального парка Йосемити. Двумерная карта рельефа отображает топографию данной области.На картах физического рельефа на самом деле есть возвышенности, которые представляют разные высоты. (Возможно, вы видели их в школе.) Однако, если вы собираетесь в поход, носить их в кармане не очень удобно.

    Плоские карты

    Плоские карты отображают рельеф по-разному. На старых плоских картах вы можете видеть области с линиями разной толщины, чтобы обозначить вариации крутизны локаций. При использовании этой техники, известной как «штриховка», чем толще линии, тем круче область.По мере развития картографирования штриховка была заменена заштрихованными участками, которые отображали вариации крутизны местности. Эти типы карт могут также отображать обозначения высоты в различных местах на карте, чтобы дать зрителям некоторый контекст.

    Различия в высоте на плоских картах также могут быть представлены с использованием разных цветов — обычно от светлого до темного для восходящих высот, причем самые темные области находятся дальше всего над уровнем моря. Недостатком этого метода является то, что контуры на земле не видны.

    Чтение топографических карт

    Топографические карты, которые также являются типами плоских карт, используют горизонтальные линии для обозначения высоты. Эти линии соединяют точки, которые находятся на одном уровне, поэтому вы знаете, что, перемещаясь с одной линии на другую, вы либо поднимаетесь, либо опускаетесь по высоте. На линиях также есть номера, указывающие, какая отметка представлена ​​точками, соединенными этой линией. Между линиями сохраняется постоянный интервал, например 100 футов или 50 метров, который будет отмечен в легенде карты.По мере того, как линии сближаются, земля становится круче. Если числа становятся меньше по мере того, как вы приближаетесь к центру области, они представляют собой место впадины и имеют отметки решетки, чтобы отличить их от холмов.

    Общие способы использования топографических карт

    Вы найдете топографические карты в магазинах спортивных товаров или на интернет-сайтах для любителей активного отдыха. Поскольку на топографических картах также отображаются глубины воды, расположение порогов, водопадов, плотин, точек доступа к лодочным аппарелям, прерывистые потоки, лесистые болота и болота, песок vs.гравийные пляжи, песчаные отмели, дамбы, волнорезы, опасные скалы, дамбы и мангровые заросли — они чрезвычайно полезны для отдыхающих, туристов, охотников и всех, кто занимается рыбалкой, рафтингом или катанием на лодках. На топографических картах также показаны наземные и заглубленные трубопроводы, а также инженерные сети и телефонные столбы, пещеры, крытые водоемы, кладбища, шахтные стволы, карьеры, палаточные лагеря, станции рейнджеров, зоны зимнего отдыха и грунтовые дороги, которые, скорее всего, не появятся. на вашей основной дорожной карте.

    В то время как топография относится к суше, карта, которая показывает переменную глубину воды, называется батиметрической картой или картой .В дополнение к отображению глубин линиями, как на топографической карте, эти типы карт могут также отображать различия в глубинах с помощью цветового кодирования. Серфингисты могут просматривать батиметрические карты пляжей, чтобы определить места, где волны, вероятно, будут больше, чем в других местах (крутой подъем вблизи пляжа означает большие волны).

    Чтение: Карты | Геология

    Карты — важный инструмент в геологии. Карты так же важны в геологии, как письменные тексты при изучении литературы.Изучая карты, геолог может увидеть форму и геологию земной поверхности и определить геологические структуры, скрытые под поверхностью. Геологов обучают чтению и составлению карт. Многие геологи имеют опыт составления карт какой-либо части земной поверхности.

    Для умелого чтения карт требуется некоторое обучение. От вас не ожидается, что вы станете экспертом-геологом в чтении карт. Однако ожидается, что вы разовьете свои навыки чтения карт, когда будете использовать карты для изучения геологии.

    Топографические карты

    Рисунок 1. Карта Йеллоустона.

    Топографическая карта (как на рисунке 1) — это один из типов карт, используемых геологами. Топографические карты показывают трехмерную форму земли и особенности ее поверхности. Топографические карты также используются туристами, проектировщиками, принимающими решения о зонировании и разрешениях на строительство, государственными учреждениями, участвующими в планировании землепользования и оценками опасностей, а также инженерами-строителями. Топографические карты, составленные и изданные U.S. Геологическая служба изображает сетки, которые используются в документах для определения местоположения недвижимости, поэтому домовладельцы и владельцы недвижимости иногда считают полезным обращаться к топографическим картам своего района.

    Большинство топографических карт используют контурные линии для обозначения высоты над уровнем моря. Контурные линии показывают форму земли в вертикальном направлении, позволяя изобразить трехмерную форму земли на двухмерном листе бумаги или экране компьютера. Когда вы знаете, как читать контурные линии, вы можете смотреть на них на топографической карте и визуализировать горы, равнины, хребты или долины, которые они изображают.

    Топографические карты важны в геологии, потому что они детально изображают поверхность земли. Этот вид поверхности показывает образцы, которые предоставляют информацию о геологии под поверхностью.

    Формы рельефа земли являются результатом поверхностных процессов, таких как эрозия или седиментация, в сочетании с внутренними геологическими процессами, такими как подъем магмы с образованием вулкана или гребня коренных пород, вытесненных вверх вдоль разлома. Изучая форму земной поверхности с помощью топографических карт, геологи могут понять природу поверхностных процессов в данной области, включая зоны, подверженные оползням, места, подвергающиеся эрозии, и места, где накапливаются наносы.Они также могут найти ключи к разгадке геологической структуры и геологической истории местности.

    В дополнение к топографической карте для полного понимания основной геологической структуры и истории области требуется заполнение геологической карты и разрезов. Топографическая карта представляет собой систему координат, на которой строится большинство геологических карт.

    Чтение топографической карты

    Чтение топографической карты требует знания того, как она отображает трехмерную форму земли, чтобы, глядя на топографическую карту, вы могли визуализировать форму земли.Чтобы читать топографическую карту, нужно понимать правила контурных линий.

    Правила для контурных линий

    • Контурная линия соединяет все точки области карты, которые находятся на определенной высоте. Например, каждая точка на контурной линии длиной 600 футов представляет собой точку на Земле, которая находится на высоте 600 футов над уровнем моря. Вы можете визуализировать контурную линию как береговую линию, которая существовала бы, если бы океан покрыл землю до этой отметки.
    • Интервал изолиний — это расстояние по вертикали, , также известное как перепад высот, между соседними горизонтальными линиями.На карте с 40-футовым интервалом изолиний вертикальное расстояние между двумя горизонтальными линиями, которые находятся рядом друг с другом, составляет 40 футов, независимо от горизонтального расстояния между двумя линиями на карте.
    • Изолинии не пересекаются друг с другом, потому что точка на поверхности земли не может находиться на двух разных отметках. (Однако в редких случаях, когда на топографической карте появляется вертикальный обрыв, контурные линии вдоль обрыва могут соединяться в одну линию.)
    • Круги с замкнутыми контурами обычно обозначают холмы.
    • Углубления, у которых нет выхода, обозначаются замкнутыми контурами с короткими линиями, выходящими из них и направленными к центру. (Короткие линии, выходящие из контурных линий, называются штрихами, штриховками или делениями.)
    • Контурные линии на стандартных топографических картах Геологической службы США имеют коричневый цвет — xcept на поверхности ледников, где контурные линии синие.
    • Высота точки на карте, которая не находится на горизонтальной линии, должна оцениваться как больше, чем отметка ближайшей горизонтальной линии под ней, и меньше, чем отметка ближайшей горизонтальной линии над ней. Например, точка, расположенная на полпути между контурами 5440 футов и 5480 футов, будет находиться на высоте приблизительно 5460 футов.
    • Изолинии изгибаются вверх по течению, когда пересекают долину. Это приводит к «Правилу V» : там, где они пересекают потоки, изолинии образуют V, указывающие вверх по потоку.
    • Если контуры близки друг к другу, рельеф крутой; там, где контурные линии далеко друг от друга, уклон будет пологим или пологим.
    • Рельеф на ландшафте — это разница высот между двумя заданными точками. Максимальный рельеф на топографической карте — это разница высот между самой высокой и самой низкой точками на карте.
    Карта четырехугольника, широты и долготы

    Стандартные топографические карты Геологической службы США покрывают четырехугольник. Четырехугольник карты охватывает долю градуса долготы с востока на запад и такую ​​же долю градуса широты с севера на юг. Поскольку линии градусов долготы (также называемые меридианами) в Северном полушарии приближаются все ближе и ближе друг к другу, чем ближе они подходят к Северному полюсу, тогда как линии градусов широты остаются на том же расстоянии друг от друга, когда они окружают Землю, карты четырехугольника охватывают меньшее расстояние на восток. -на запад, чем с севера на юг.

    Широта — это то, насколько далеко к северу или югу от экватора находится точка на Земле, измеряемая в градусах, от 0 ° на экваторе до 90 ° на полюсах. При указании широты всегда указывайте, находится ли она в северном полушарии (N) или в южном полушарии (S).

    Долгота — это то, насколько далеко на восток или запад, максимум до 180 °, находится точка на Земле от нулевого меридиана. Главный меридиан (0 ° долготы) — это линия с севера на юг, проходящая через Гринвич, Англия. При указании долготы укажите, находится ли она в западном полушарии (W) или в восточном полушарии (E).

    Меридианы, линии долготы, проходят от Южного полюса до Северного полюса, сходясь (сходясь) на полюсах. Поскольку меридианы сходятся на полюсах, градусы долготы становятся все меньше и меньше около каждого полюса. Напротив, градус широты остается примерно 69 миль в поперечнике, независимо от того, насколько близко или далеко он находится от полюсов или экватора.

    Градусы широты и долготы делятся на угловые минуты и угловые секунды. В этом контексте их обычно называют просто минутами и секундами, но следует иметь в виду, что эти минуты и секунды представляют собой единицы углов, а не единиц времени.Эти устройства, которые делят углы на более мелкие части, работают следующим образом:

    1. В 1 градусе 60 угловых минут.
    2. Минуты обозначаются одним апострофом: ‘.
    3. В символах 60 ′ = 1 ° означает, что в 1 градусе 60 минут.
    4. В 1 угловой минуте 60 угловых секунд.
    5. Чтобы преобразовать угловые минуты в десятичную дробь градуса, умножьте количество угловых минут на 1 ° / 60 ′. Например, чтобы преобразовать 15 ′ в десятичную дробь градуса, 15 ′ x 1 ° / 60 ′ = 0.25 °. Проще говоря, просто разделите количество угловых минут на 60, чтобы преобразовать их в десятичные градусы.
    6. Символ угловой секунды — двойной апостроф или кавычка: «.
    7. В символах 60 ″ = 1 ′ означает, что в 1 минуте 60 секунд.

    Два обычных размера четырехугольника: 7,5 минут (1/8 градуса) и 15 минут (1/4 градуса).

    Название, размер и широта-долгота четырехугольника топографической карты

    На изображении выше показан северо-восточный угол топографической карты четырехугольника можжевельника, который пересекает границу штатов Орегон и Вашингтон.Название четырехугольника происходит от названия места на карте. Найдите в этом углу карты следующую информацию:

    1. Название четырехугольника
    2. Состояние (а), в котором находится четырехугольник
    3. Размер четырехугольника
    4. Название и дробный масштаб карты четырехугольника, расположенной рядом с северо-востоком от углового угла.
    5. Долгота восточной границы карты
    6. Широта северной границы карты

    Показать ответ

    1. Можжевельник
    2. Орегон и Вашингтон
    3. 7.5 минут
    4. Валлула, 1: 125 000
    5. 119 ° 00 ′
    6. 46 ° 00 ′

    Масштаб карты, интервал изолиний и магнитное склонение

    Важная информация отображается внизу карты четырехугольника USGS, включая масштаб карты, интервал изолиний и магнитное склонение. Изображение вверху получено из нижней части 7,5-минутного четырехугольника Juniper. В нем, среди прочего, указано:

    1. Масштаб карты. Масштаб карты указан в дробном масштабе 1: 24 000.Это означает, что 1 дюйм на карте соответствует 24 000 дюймов в реальном мире, представленном на карте, или 1 см равен 24 000 см; Другими словами, расстояния на карте были уменьшены в 24 000 раз по сравнению с их реальными размерами. Под дробным масштабом масштаб карты также отображается по-другому, в виде столбиков с использованием трех различных единиц. Одна из шкал — в милях, одна — в тысячах футов и одна — в километрах.
    2. Интервал изолиний, разница в высоте между соседними горизонтальными линиями на карте, указан под масштабом карты как 20 футов.
    3. Также есть напоминание о том, что отметки, показанные на карте, — это отметки над средним (средним) уровнем моря на Земле.
    4. (Вы можете заметить, что эта карта делает что-то необычное для топографической карты. Она показывает глубины реки Колумбия в футах ниже поверхности реки, когда река подпирается в своем резервуаре за плотиной до нормального уровня поверхности бассейна 340 футов над уровнем моря.)
    5. Слева от линейчатой ​​шкалы магнитное склонение показано стрелкой, расходящейся от линии, ориентированной на истинный север.Истинный север — это направление к географическому Северному полюсу. Географический Северный полюс — это то место, где находится северный конец оси вращения Земли. Северный магнитный полюс находится на северо-востоке Канады. В 1962 году магнитный Северный полюс, измеренный от четырехугольника можжевельника, находился в 20,5 ° к востоку от истинного севера. Если вы взяли магнитный компас к четырехугольнику можжевельника в 1962 году, его стрелка на север указала бы на 20,5 ° к востоку от истинного севера, поэтому вам пришлось бы настроить магнитный компас, чтобы компенсировать склонение.Северный магнитный полюс отклоняется на несколько миль каждый год, а в 1962 году было определено магнитное склонение 20,5 ° к востоку от истинного севера; сейчас может быть немного иначе.

    Построение топографического профиля

    Одним из важных инструментов, которые можно использовать для извлечения вертикальной информации с топографической карты и более четкого просмотра формы земной поверхности, которую она представляет, является топографический профиль.

    Построение топографического профиля позволяет визуализировать вертикальную составляющую ландшафта.Топографический профиль похож на вид на пейзаж, который вы видите, когда стоите на земле, глядя на холмы и долины сбоку, а не сверху.

    Для топографической карты, подобной приведенной ниже, вот как построить топографический профиль.

    Шаг 1

    Определите линию профиля, линию через ту часть карты, которую вы хотите видеть в виде профиля или поперечного сечения. В зависимости от того, какую часть карты вы хотите видеть в профиле, вы можете провести линию профиля в любом направлении по вашему выбору, через любую часть карты по вашему выбору.Для карты, используемой в этом примере, мы решили нарисовать профиль от A до A ’, как показано на схеме ниже, чтобы увидеть всю длину холма в профиле.

    Шаг 2

    Нарисуйте сетку, которая будет содержать профиль. Ширина сетки должна быть такой же, как длина линии профиля. Чтобы нарисовать профиль, сетка должна быть пересечена равномерно расположенными горизонтальными линиями, которые представляют отметки изолиний. Сетка должна простираться достаточно высоко, чтобы охватить диапазон высот горизонтальных линий, пересекаемых линией профиля.Вы можете видеть, что сетка, показанная ниже, включает диапазон высот, которые линия профиля пересекает на карте. Кроме того, сетка должна иметь дополнительную горизонтальную линию внизу и вверху для размещения частей профиля, которые проходят выше самой высокой отметки контура и ниже самой низкой отметки контура. Вот почему сетка в приведенном ниже примере идет ниже 400 футов и выше 500 футов по высоте.

    Шаг 3

    Перенести отметки изолиний с топографической карты в сетку профиля.Точка, в которой каждая горизонтальная линия пересекает линию профиля на топографической карте, определяет горизонтальную координату каждой соответствующей точки на сетке топографического профиля. Высота каждой изолинии соответствует вертикальной координате каждой соответствующей точки на профильной сетке, как показано на диаграмме ниже.

    Шаг 4

    Теперь, когда вы отметили точки высот на сетке профиля, нарисуйте плавную линию, соединяющую точки данных, как показано ниже.Обратите внимание, что концы этого профиля проходят ниже отметки контура 400 футов, но не доходят до отметки 380 футов, потому что на карте линия профиля не достигла отметки контура 380 футов. Также обратите внимание, что вершина профиля достигает пика выше 520 футов, но менее 540 футов, потому что линия профиля не пересекает контурную линию 540 футов.

    Шаг 5

    Завершенный топографический профиль и карта, с которой он был составлен, показаны ниже. Топографические профили обычно строятся без нанесения каких-либо линий на карту.Вместо этого край листа бумаги укладывается вдоль линии профиля, и данные контурной линии переносятся на край листа бумаги. С края листа данные переносятся в профильную сетку, которая находится на отдельном листе бумаги.

    Обратите внимание на топографический профиль, построенный выше, о том, что вершина холма находится выше 520 футов, но ниже 540 футов. Точно так же концы профиля ниже 400 футов, но выше 380. Это согласуется с отметками этих частей линия профиля на карте.

    Обратите внимание, что вертикальный масштаб профиля сильно отличается от горизонтального масштаба на карте. В этом примере карта покрывает 0,25 мили по горизонтали на меньшее расстояние, чем профиль покрывает 100 футов по вертикали. В результате топографический профиль сильно преувеличен по вертикали. При реальном взгляде на холм со стороны он не выглядел бы таким крутым, как на построенном нами топографическом профиле.

    Если вертикальный масштаб на топографическом профиле отличается от масштаба карты, как в этом случае, то профиль будет иметь вертикальное преувеличение .Можно рассчитать вертикальное преувеличение топографического профиля. Это дробный масштаб вертикальной оси топографического профиля, деленный на дробный масштаб карты. Например, если вертикальный масштаб профиля составляет 1: 200, а масштаб карты — 1:24 000, вертикальное увеличение будет [latex] \ displaystyle \ frac {\ left (\ frac {1} {200} \ right)} {\ left (\ frac {1} {24,000} \ right)} [/ латекс]. Чтобы разделить на дробь, вы можете инвертировать и умножить, так что это становится [latex] \ displaystyle \ left (\ frac {1} {200} \ right) \ times \ left (\ frac {24,000} {1} \ right) = \ frac {24,000} {200} = 120 [/ латекс].Топографический профиль с VE 120 был бы очень преувеличенным топографическим профилем. Это было бы так, как если бы резиновую модель ландшафта тянули в вертикальном направлении, пока она не стала в 120 раз выше, чем она есть на самом деле.

    Если вертикальный масштаб топографического профиля отличается от масштаба карты, вертикальное преувеличение должно быть указано рядом с профилем, например, VE = 10 или VE 10x, если вертикальное преувеличение составляет 10.

    Сравните профиль с топографической картой.Вы увидите, что холм круче на западной (левой) стороне, чем на восточной (правой) стороне. Это согласуется с тем, что контурные линии расположены более близко на западной стороне холма и дальше друг от друга на восточной стороне холма. Это согласуется с правилами контурных линий, которые гласят, что уклоны круче там, где контурные линии расположены ближе друг к другу, а уклоны менее крутые, где контурные линии более широко разнесены.

    Если вы начертите профиль с севера на юг через вершину холма, как вы думаете, будет ли профиль симметричным или асимметричным?

    Контрольный список для полного топографического профиля

    Правильно нарисованный топографический профиль будет иметь следующие атрибуты:

    • Топографический профиль нанесен на прямолинейный график с равномерно распределенными линиями сетки.(Вертикальные линии сетки не требуются.)
    • Линии высот нанесены вдоль левой вертикальной оси.
    • Профиль — это плавная кривая, градиент которой изменяется, а не прямые отрезки, соединяющие точки и только изгибающиеся в точках.
    • Если вертикальный масштаб профиля отличается от масштаба карты, отображается результирующая величина вертикального преувеличения.
    • Концы и любые высокие или низкие точки топографического профиля должны находиться выше или ниже линий высот, а не на них, за исключением случаев, когда конечная, высокая или низкая точка линии профиля падает прямо на контурная линия.

    Узнайте больше: топографические карты

    Используйте этот ресурс, чтобы ответить на следующие вопросы. Вы можете прекратить просмотр на отметке 4:04.

    1. Для чего нужны топографические карты и как они это делают?
    2. Каковы значения терминов топографическая карта, контурная линия, горизонтальный интервал и указательный контур?
    3. Если бы вы прошли по контурной линии, что бы произошло с вашим возвышением?
    4. Если вы идете перпендикулярно контурным линиям, что вы делаете?
    5. Что обозначают близкие контурные линии?

    Батиметрические карты

    Рисунок 3.Вулкан Лоихи, растущий на склоне вулкана Килауэа на Гавайях. Черные линии на вставке показывают поверхность суши над уровнем моря, а синие линии показывают топографию ниже уровня моря. Щелкните изображение, чтобы просмотреть его в увеличенном виде.

    Батиметрическая карта похожа на топографическую карту с контурными линиями, представляющими глубину ниже уровня моря, а не высоту выше. Числа низкие вблизи уровня моря и становятся выше с увеличением глубины.

    Килауэа — самый молодой вулкан, обнаруженный над уровнем моря на Гавайях.На склоне Килауэа находится еще более молодой вулкан под названием Лоихи. Батиметрическая карта, изображенная на рисунке 3, показывает форму Лоихи.

    Геологические карты

    Геологическая карта показывает геологические особенности региона (см. Пример на рисунке 4). Камни имеют цветовую кодировку и обозначены ключом. Разломы и складки также показаны на геологических картах. Геология накладывается на топографическую карту, чтобы дать более полное представление о геологии региона.

    На геологической карте показаны отображаемые единицы горных пород, наносимые на карту единицы отложений, которые покрывают породы, и геологические структуры, такие как разломы и складки.Картографируемая единица породы или отложений — это единица, которую геолог может последовательно распознать, проследить по ландшафту и описать так, чтобы другие люди могли распознать ее и проверить ее присутствие и идентичность. Отображаемые единицы показаны разными цветами или узорами на базовой карте географической области.

    Рис. 4. Геологическая карта региона вокруг Old Faithful, Йеллоустонский национальный парк.

    Геологические карты важны по двум причинам. Во-первых, когда геологи составляют геологические карты и связанные с ними объяснения и разрезы, они развивают теоретическое понимание геологии и геологической истории данной области.

    Во-вторых, геологические карты являются важным инструментом для практических приложений, таких как зонирование, гражданское строительство и оценка опасностей. Геологические карты также имеют жизненно важное значение для поиска и разработки геологических ресурсов, таких как гравий для прокладки дороги, по которой вы едете, масло для привода автомобиля, в котором вы путешествуете, или алюминий для создания более экономичного двигателя в вашем следующем автомобиле. Еще один ресурс, который разрабатывается на основе геологических карт, — это подземные воды, от которых многие города, фермы и фабрики используют воду.

    Основные компоненты геологических карт

    Полная геологическая карта имеет как минимум две особенности:

    • сама карта
    • легенда карты или ключ, объясняющий все символы на карте.

    Профессиональные геологические карты обычно также включают два других компонента:

    • сопутствующее объяснение горных пород или отложений
    • геологических разреза области карты.

    Легенда или ключ к геологической карте обычно печатается на той же странице, что и карта, в обычном формате.Символ для каждой породы или осадочной единицы показан в рамке рядом с ее названием и кратким описанием. Эти символы расположены в возрастной последовательности от самого старого внизу к самому младшему вверху. Геологическая эра, или период, или эпоха — геологический возраст — указаны для каждой горной единицы в ключе. Сложив блоки в возрастной последовательности от самых молодых вверху к самым старым внизу и определив, к какому интервалу геологического времени принадлежит каждая единица, устройство чтения карт может быстро увидеть возраст каждой породы или толщи отложений.Ключ карты также содержит список и объяснение символов, показанных на карте, таких как символы для различных типов разломов и складок. См. Таблицу символов геологической карты для получения изображений и обзора символов карты, включая простирания и падения, разломы, складки и обзор.

    Таблица обозначений геологической карты

    Знаки ударов и погружений
    Удар и падение — это способ представления трехмерной ориентации плоской поверхности на двухмерной карте.Удар — это компасное направление горизонтальной линии на плоскости. Все горизонтальные линии на плоскости параллельны, поэтому все они имеют одинаковое характерное направление по компасу. Падение — это угол, под которым плоскость спускается вниз от горизонтали, при максимальном наклоне, который находится под прямым углом (90º) от простирания.
    Символ карты Определение Расшифровка символа
    простирания и падения пластов, кроме горизонтальных или вертикальных
    • простирание (длинная линия) — горизонтальная линия на плоскости напластования
    • параллели простирания вблизи контактов между слоистыми породами
    • падение показывает, в какую сторону спускаются грядки
    • Угол падения, число в конце символа падения, показывает, насколько пласты наклонены вниз от горизонтали
    горизонтальные кровати
    • , поскольку пласт горизонтальный, удары ударяются во всех направлениях
    • , поскольку пласт горизонтальный, падение составляет 0%
    простирание и падение вертикальных пластов
    • простирание (длинная линия) — горизонтальная линия на плоскости напластования
    • , поскольку пласт наклонен вертикально (имеет наклон 90%), он одинаково наклоняется в любом направлении под прямым углом к ​​простиранию, поэтому линия падения показана простирающейся в обоих направлениях

    Объяснения горных пород часто даются в отдельной брошюре, прилагаемой к карте.Объяснения включают достаточно подробные описания, чтобы любой геолог мог распознать единицы и узнать, как был определен их возраст.

    Если включено, разрезы обычно печатаются на той же странице, что и геологическая карта. Они являются важным дополнением к геологическим картам, особенно если карта фокусируется на геологии коренных пород под почвой и рыхлых отложений.

    Геологические разрезы

    Геологический разрез — это вид сбоку кусочка земли.Он показывает, как различные типы горных пород слоистые или иначе конфигурируются, а также изображает геологические структуры под поверхностью земли, такие как разломы и складки. Геологические разрезы строятся на основе геологии, нанесенной на карту на поверхности, в сочетании с пониманием горных пород с точки зрения физического поведения и трехмерных структур.

    Сводка

    • Ученые-геологи регулярно используют топографические, батиметрические и геологические карты.
    • Топографические карты показывают форму ландшафта.Отметки указывают высоту над уровнем моря.
    • Батиметрические карты похожи на топографические карты объектов, обнаруженных под водой. Отметки указывают на глубину ниже уровня моря.
    • Геологические карты показывают горные породы и геологические особенности, такие как разломы и складки.

    Внесите свой вклад!

    У вас была идея улучшить этот контент? Нам очень понравится ваш вклад.

    Улучшить эту страницуПодробнее

    Совет Махараштры Решения по географии класса 7 Глава 11 Контурные карты и формы рельефа — Решения Совета Махараштры

    Правление штата Балбхарти Махараштра Класс 7 Решения по географии Глава 11 Контурные карты и формы рельефа Примечания, Упражнение в учебнике Важные вопросы и ответы.

    Государственный совет Махараштры Решения по географии класса 7 Глава 11 Контурные карты и формы рельефа

    География класса 7 Глава 11 Учебник по контурным картам и формам рельефа Вопросы и ответы

    1. Ответьте на следующие вопросы:

    Вопрос 1.
    Как можно показать распределение высоты и рельефа в регионе?
    Ответ:
    (i) При изучении различных форм рельефа на поверхности земли необходимо учитывать различные аспекты рельефа, такие как высота, рельеф, уклон, направление уклона и дренаж.
    (ii) Для этого используются карты, подготовленные с использованием методов математической съемки.
    (iii) Эти карты помогают нам понять вышеупомянутые характеристики форм рельефа.

    Вопрос 2.
    Кому пригодятся контурные карты?
    Ответ:
    (i) Контурные карты полезны альпинистам, путешественникам, солдатам, офицерам обороны.
    (ii) Можно оценить характер земли и ее форму.
    (iii) Офицеры обороны используют контурные карты для стратегического планирования.
    (iv) На контурной карте региона можно определить подходящий участок для любого проекта.

    Вопрос 3.
    Что вы понимаете под наблюдением изолиний?
    Ответ:
    (i) Контурные линии — это изолинии высоты.
    (ii) Они нарисованы путем соединения мест на одинаковой высоте.
    (iii) Они помогают идентифицировать формы суши и их высоту над уровнем моря.
    (iv) Эти линии также помогают нам понять природу и направление склона.

    Вопрос 4.
    Чем может быть полезна контурная карта фееру?
    Ответ:
    (i) Полезно выбрать тип земледелия, которое будет практиковаться в выращивании плантаций в холмистых регионах, интенсивное земледелие в низинных регионах.
    (ii) Чтобы уменьшить эрозию почвы, выкапываются траншеи в направлении, перпендикулярном склону земли.
    (iii) Вдоль таких траншей высаживаются деревья. Когда фермер выкапывает такие траншеи, он будет осторожно поддерживать уровень.

    2. Заполните пропуски соответствующими словами:

    Вопрос 1.
    Если горизонтальные линии расположены ближе друг к другу, то наклон ______.
    Ответ:
    крутой

    Вопрос 2.
    Контурные линии на карте обозначают ______.
    Ответ:
    мест на одной высоте

    Вопрос 3.
    Наклон можно понять по расстоянию между ______.
    Ответ:
    контурных линий

    Вопрос 4.
    Если расстояние между двумя контурными линиями больше, то ______ будет пологим.
    Ответ:
    уклон

    3. Определите формы рельефа на следующей карте:

    География класса 7 Глава 11 Контурные карты и формы рельефа InText Вопросы и ответы

    Модель рельефа местности представлена ​​на рис.Внимательно изучите его и ответьте на следующие вопросы:

    Вопрос 1.
    Какие формы рельефа вы видите на модели?
    Ответ:
    На модели видны горы и долины рек.

    Вопрос 2.
    Какие цвета были использованы на них?
    Ответ:
    В модели использованы желтый и синий цвета.

    Модель земной поверхности:

    Вопрос 1.
    Что все вы видели на карте?
    Ответ:
    На карте мы видим много контурных линий, показывающих холмистую местность.Мы можем видеть высоту горных хребтов, а также их уклон.

    Вопрос 2.
    Каково общее направление диапазонов, показанных на карте?
    Ответ:
    Общее направление хребтов на карте — восток-запад

    Вопрос 3.
    В каком направлении на карте расположена плоская земля?
    Ответ:
    Равнинный участок расположен восточнее.

    Вопрос 4.
    Каковы максимальные и минимальные значения линий на карте?
    Ответ:
    Максимальное значение линий на карте — 800, минимальное — 600.

    Вопрос 5.
    Что означают эти значения?
    Ответ:
    Эти линии указывают высоту региона над уровнем моря.

    Вопрос 6.
    Есть ли у карты сходство с моделью на рис. (а)? Что это?
    Ответ:
    Модель на рисунке (a) и контурная карта на рисунке (b) относятся к одному региону. Значения широты и долготы одинаковы на обеих картах.

    Вопрос 7.
    Какой рисунок дает нам дополнительную информацию и что это за информация?
    Ответ:
    Рис.(b) Контурная карта дает больше информации. Он дает нам информацию о высоте склона форм суши с широтой и долготой

    .

    Вопрос 8.
    Есть ли сходство между этой картой и схематической картой картофельного холма?
    Ответ:
    Да, как и схематическая карта картофельного холма,
    контурная карта также представляет собой трехмерную фигуру (модель земной поверхности), которая была преобразована в двухмерную карту. Карта контурных линий отображает интервалы контурных линий.

    Считайте, что вы пошли заниматься альпинизмом. Вам предстоит покорить вершину холма «А». Карта этого холма приведена ниже. Изучая контурные линии на карте, найдите сторону, с которой вы легко и безопасно достигнете вершины. Отметьте карандашом свой путь на карте.

    Ответ:
    Я буду подниматься с востока, чтобы безопасно достичь вершины, так как склон на восток пологий и не крутой. Меньшее расстояние между контурными линиями указывает на крутой уклон. Расстояние между линиями больше к востоку, что указывает на более пологий уклон с этой стороны.

    География класса 7 Глава 11 Контурные карты и формы рельефа Дополнительные важные вопросы и ответы

    Заполните пропуски:

    Вопрос 1.
    Изолинии соединяют места с такими же на карте.
    Ответ:
    высота

    Вопрос 2.
    Как правило, контурные линии не _______ друг с другом.
    Ответ:
    крест.

    Ответьте на следующие вопросы:

    Вопрос 1.
    Чем нам могут помочь контурные карты?
    Ответ:
    (i) Контурные карты помогают нам понять различные аспекты рельефа, такие как высота, рельеф, уклон, направление уклона и дренаж.
    (ii) Эти карты очень полезны альпинистам, путешественникам, солдатам, офицерам обороны и т. Д.
    (iii) Эти карты оказались очень полезными при планировании региона.

    Вопрос 2.
    Линии изолиний обычно не пересекаются. Обоснуйте.
    Ответ:
    (i) Контурные линии соединяют места с одинаковой высотой на карте.
    (ii) Следовательно, они обычно не пересекаются.

    Посмотрите на модель и карту, приведенные ниже, и ответьте на следующие вопросы:

    Трехмерная модель представлена ​​на рис.(а) В северной части модели показан бассейн реки Мула-Мута. К югу от него находится Катрадж. Хребет Дивегхат простирается с запада на восток. Далее видна часть бассейна Карха.

    Вопрос 1.
    В каком направлении лежит форт Пурандар?
    Ответ:
    Форт Пурандар находится в южном направлении.

    Вопрос 2.
    Какое направление течения реки Карха?
    Ответ:
    Река Карха течет с запада на восток.

    Вопрос 3.
    В каких частях холмы не наблюдаются?
    Ответ:
    Гряды в восточной части не наблюдаются.

    Вопрос 4.
    Какая часть модели не видна на карте? Почему?
    Ответ:
    Река Карха и направление ее течения, высота контура и наклон суши являются частью карты и не видны на модели.

    Вопрос 5.
    В каком направлении уменьшается высота хребта Катрадж-Дивегхат?
    Ответ:
    Высота хребта Катрадж-Дивегхат уменьшается с запада на восток.

    Вопрос 6.
    В каком направлении расположены более высокие холмы?
    Ответ:
    Более высокие хребты холмов расположены в южном направлении.

Добавить комментарий

Ваш адрес email не будет опубликован. Обязательные поля помечены *